103
Laws of Exponents: 1. 2. 3. 4. 5. Laws of Radicals: 1. 2. 3. 4. Laws of Logarithms: 1. 2. 3. Important Properties: 1: provided 2: 3: 4: or 5: implies that m = n 6: implies that 7: implies that M = N 8: 9: provided 10: provided QUADRATIC EQUATION Generally, an equation is said to be of quadratic form if it has the form ax 2n + bx n + c = 0, where n is an integer or a fraction; such as x 4 5x 2 + 6 = 0 and y -3 + y -3/2 + 6 = 0 Quadratic Formula: The expression b 2 – 4ac is called the discriminant 1. When b 2 – 4ac > 0, the roots are real and unequal. 2. When b 2 – 4ac = 0, the roots are real and equal (or quadratic equation is a perfect trinomial) 3. When b 2 – 4ac < 0, the roots are imaginary and unequal (complex conjugates) The roots: Sum of the roots, x 1 + x 2 = - b/a Product of the roots, x 1 ∙ x 2 = c/a

Mathematics

Embed Size (px)

DESCRIPTION

preboard reviewer for math

Citation preview

Page 1: Mathematics

Laws of Exponents:1.

2.

3.

4.

5.

Laws of Radicals:

1.

2.

3.

4.

Laws of Logarithms:1.

2.

3.

Important Properties:1: provided

2:

3:

4: or

5: implies that m = n

6: implies that

7: implies that M = N

8:

9: provided

10: provided

QUADRATIC EQUATION

Generally, an equation is said to be of quadratic form if it has the form ax2n + bxn + c = 0, where n is an integer or a fraction; such as x4 – 5x2 + 6 = 0 and y-3 + y-3/2 + 6 = 0

Quadratic Formula:

The expression b2 – 4ac is called the discriminant

1. When b2 – 4ac > 0, the roots are real and unequal.2. When b2 – 4ac = 0, the roots are real and equal (or quadratic equation is a perfect trinomial)3. When b2 – 4ac < 0, the roots are imaginary and unequal (complex conjugates)

The roots:

Sum of the roots, x1 + x2 = - b/a

Product of the roots, x1 ∙ x2 = c/a

THE BINOMIAL THEOREM

BINOMIALEXPANSION PASCAL’S

TRIANGLE(x + y)0 = 1 (x ≠ -y) 1 1(x + y)1 = x + y 1 1(x + y)2 = x2 + 2xy + y2 1 2 1(x + y)3 = x3 + 3x2y + 3xy2 + y3 1 3 3 1(x + y)4 = x4 + 4x3y + 6x2y2 +4xy3 + y4 1 4 6 4 1(x + y)5 = x5 + 5x4y + 10x3y2 +10x2y3 + 5xy4 + y5 1 5 10 10 5 1(x + y)6 = x6 + 6x5y + 15x4y2 +20x3y3 + 15x2y4 + 6xy5 + y6 1 6 15 20 15 1

This array of numbers is called the Pascal’s Triangle. Any lower row is formed by adding any two adjacent numbers of the upper row and place 1 at both ends so as to form a triangle. Pascal’s Triangle is used to easily recall the numerical coefficients of the expansion of the powers of a binomial. But for large powers of a binomial, Pascal’s Triangle becomes inconveniently to use. For such, use Binomial Theorem.

The rth term of (x + y)n = n(n-1)(n-2)…(n-r+2) xn-r+1yr-1

Page 2: Mathematics

(r – 1)!

LOWEST COMMON MULTIPLE (LCM)The lowest common multiple (LCM) of several natural numbers is the smallest natural number of which each of the given numbers is a factor. It mat be found by taking the product of all the different prime factors of the numbers, each taken the greatest number of times that

it occurs in any of those numbers.

Example: Find the lowest common multiple of 24, 10, 18, and 25.Solution:

24 = 2x2x2x3, 10 = 2x5, 18 = 2x3x3, 25 = 5x5LCM = 2x2x2x3x3x5x5 = 1800

HIGHEST COMMON FACTOR (HCF)The highest common factor (HCF) of several natural numbers is the largest natural number which is a factor of all the given numbers. It may be found by taking the product of all the different prime factors common to the given numbers, each taken the smallest number of

times that it occurs in any of those numbers. If the given numbers have no prime factors in common, the HCF is defined to be 1, in this case the numbers are said to

be relatively prime.

Example: Find the highest common factor of 24, 30, 18 and 150.Solution:

24 = 2x2x2x3, 30 = 2x3x5, 18 = 2x3x3, 150 = 2x3x5x5HCF = 2x3 = 6

PROGRESSION

Arithmetic Progression (A. P.)- a sequence of terms in which each term after the first is obtained by

adding a fixed number to the preceding term.- a sequence of terms in which any two consecutive terms has a common

difference.That is, the sequence a1, a2, a3 are in arithmetic progression if and only if:

a2 – a1 = a3 – a2

Let: a1 = first term of an A. P.an = nth term of an A. P.d = common differencen = number of termsSn = the sum of n terms

Then,an = a1 + (n – 1)dSn = n/2 (a1 + an)

Sn = n/2 [ 2a1 + (n – 1)d]

Arithmetic MeanThe arithmetic mean between two numbers is the number which when placed between

the two numbers, forms with them an arithmetic progression.

In general, for n terms, arithmetic mean (AM) = a1 + a2 + a3 + … + an

n

Geometric Progression (G.P.)- a sequence of terms in which each term after the first is found by

multiplying the preceding term by a fixed number called common ratio.- The sequence a1, a2, a3 are in G.P. if and only if:

a2/a1 = a3/a2 = rThe nth term, an

an = a1rn-1

Sum of the first n terms in G.P.Sn = a1 1-r n

1-r where a1 = first term

r = common ration = number of terms

Infinite Geometric ProgressionThe sum of terms in geometric progression can be found if the common ratio

| r |<1, -1 < r < 1

Geometric MeanThe term in between the first and last terms of the geometric sequence.

Let x = geometric mean,

a1, x2, a2 geometric progression

Then, x/a1 = a2/x common ratio

Solving for x: x2 = a1a2

x = geometric progression

Harmonic Progression

Sequence of terms whose reciprocal forms an arithmetic progression

Page 3: Mathematics

That is, a1, a2, a3…an are in harmonic progression

If 1/a1, 1/a2, 1/a3…1/an form an arithmetic progression

Harmonic Mean

Let x = harmonic mean between a and b

a, x, b in H. P.

1/a, 1/x, 1/b in A. P.

Then,

1/x – 1/a = 1/b – 1/x common difference

Solving for x:

2/x = 1/a + 1/b

2/x = a + b/ab

x = 2ab/(a + b)

Harmonic Mean (HM) = _ n 1/a1 + 1/a2 + 1/a3 + … + 1/an

RATIO, PROPORTION AND VARIATION

1. RATIOThe ratio of a number a to another number b is the fraction a/b usually as a:b (read a “is

to” b). Where a is called antecedent and b is called consequent.

2. PROPORTIONProportion is a statement that two ratios are equal. Usually written as a:b = c:d or a/b =

c/d. Where a and d are called the extremes and b and c are called the means. D is called the fourth proportional to a, b, and c. If the means of a proportion are equal, as in a/x = x/b, the number b is called the third proportional to a and x, while the number x is called the mean proportional between a and b. It is obvious that the mean proportional between a and b is equal to their geometric mean. A proportion may be altered in four different ways summarized in the tabular form below.

Basic Proportion Transformation by Transformed FormAlternation a:c = b:d

a:b = c:d Inversion b:a = d:cAddition (a+b):b = (c+d):dSubtarction (a-b):b = (c-d):d

3. VARIATION

i. Direct Variation (also direct proportion)The Five Statements Below Have Same Meaning

As x increases y increase proportionatelyy is proportional to xy is directly proportional to xy varies as xy varies directly as x

In symbols the above statements mean,y α x

In mathematical terms,y = kx

where k is called the constant of proportionality or also called the constant of variation

ii. Inverse Variation (also indirect variation)The following Statements Below Have Same Meaning

As x decreases y increase (and vice versa)y is inversely proportional to xy varies indirectly as x

In symbols the above statements mean,y α 1/x

In mathematical terms,y = k/x, (x not equal to zero)

Examples1. Boyle’s Law: “When the temperature of a confined gas is held constant, the pressure of the gas varies inversely as its absolute pressure.”2. Ohm’s Law: “The current is directly proportional to the impressed emf and inversely to the resistance

iii. Joint Variationy varies jointly as x and wIn symbols,

y α xwMathematically, Y = kxw; Warning: Not y = k(x+w)

EQUATION OF THE HIGHER DEGREE Rational integral term – a constant, or a positive integral power of any variable, or the

product of such qualities. Ex. 5, 2x4, - 6y3, 15x2y5

Degree of a term-the term Cxn, where C is a constant and n is a positive integer is said to be of degree n in terms of x.-the term Cxnyn is said to be of the degree n in terms of x, degree p in terms of y, and degree n+p in terms of x and y.

Polynomial Function

Page 4: Mathematics

-An algebraic sum of rational integral term-A series of a power of a base where he exponents are positive integer-Also called rational integral function.

f(x) = a0xn + a1xn-1 + a2xn-2 + …+ an polynomial function in degree nwhere: n = non-negative integer

a0, a1, a2,….,an are any constantsa0 ≠ 0

Zero of a function any value of the unknown x, that will make a function f(x) equal to zero also called root of f(x) = 0

Fundamental theorem every equation f(x) = 0 has at least one root there exist at least one number either real or complex which will satisfy f(x) =0

Number of rootsevery equation f(x) = 0 of degree n, has n roots and no more if root of order k is counted as k roots.

Multiple roots consider that the roots r1,r2, r3, …, rn of f(x) = 0 are equal if f(x) is exactly divisible by x – r but not by (x – r)2, then x – r is simple root of f(x) = 0. If f(x) = 0 is exactly divisible (x –r)2 but not by (x – r1)3, then r1 is called double root of f(x) = 0.

In general, if f(x) = 0 is divisible by (x – r)k but not by (x – r1)k+1, then r1 is a k – fold root of order k.

Multiplicitythe number of times that r appears as roots of f(x) = 0

Theorem of complex rootscomplex roots always occur in pair.That is if b = bi is root, then a – bi is also a root (-conjugate zeros) where a, b are real but b ≠ 0

Theorem on quadratic surd rootssurd root always occur in pairThat is, if a +√bi is a root, then a –√b is also a root where a and b are rational and √b is irrational.

Remainder theoremif f(x) is divided by (x – r), the remainder is f(r).

Factor theoremif r is a root of the equation f(x), then (x – r) is factor of f(x).

Converse of factor theoremif (x – r) is a factor of f(x), then r is a root of f(x) = 0

Depressed equationif r is a root of the equation f(x) = 0, then (x – r) is a factor of f(x). Thus, f(x) = (x – r). The equation Q(x) = 0 is called depressed equation of f(x) = 0, and the roots of Q(x) = 0 are the remaining roots of f(x) = 0

Descartes’ rule of signsLet f(x) be a polynomial with real coefficients.

a) The number of positive real roots of f(x) is either equal to the number of variations in sign of f(x), or that number diminished by a positive integer.

b) The number negative with real zeros of f(x) is either equal to the number of variations in sign of f(-x) or that number diminished by positive even integer.

Relationship Between Coefficients and Zeros of PolynomialGiven an integral rational function:

f(x) = a0xn + a1xn-1 + a2xn-2 + … + an-1x + an

The coefficients for the polynomial function in terms of its zeros can be given as:

-a1/ a0= sum of roots

a2 / a0 = sum of product of the roots taken two at a time

-a3/ a0 = sum of product of the roots taken three at a time

(-1)n an/ a0 = product of all roots Supplementary Problems

1. Determine m so that x3 – 2x2 + mx + 8 shall be divisible by x + 3. Ans. m = 92. If 3x4 = kx3 + x2 – 16 + 4 is divided by (x – 2), for what value of k will the

remainder be 8 ? Ans. k = 23. For what value of k will x = 3 be a factor of x3 + 7x2 + kx – 12? Ans. k = 884. Find the remainder when 149x1592 – 375x375 + 10 is divided by x + 1. Ans.

18775. One of the roots of 3x3 – mx2 + 23x – 14 = 0 is 2. Determine the value of m.

Answer: m = 146. 2x3 –x2 + mx + n is to be exactly divisible by x2 – 2x – 8. Determine the

values of m and n. Ans. m = -2, n = -247. The sum of the two roots of x3 + 6x + c = 0 is 2; find c. Ans. c = 208. What is the sum and product of the roots of the equation 3x4 – 2x2 + 8x – 6 =

0? Answer: sum = 0; product = -2

9. Given the equation x3 – 4x2 + 3x – 5 = 0. from the equation whose roots are :a) negative of the roots of the given equation

Page 5: Mathematics

b) thrice the roots of the given equationc) the roots of the given equation diminished by 2.

Answer:a) x3 + 4x2 + 3x + 5 = 0b) x3 + 12x2 + 27x – 135 = 0c) x3 + 2x2 – x – 7 = 0

10. The sum of the roots of 2x3 + mx2 – 5x - 3 = 0 equals twice the product of the roots. Determine m. Ans. m = -6

11. The product of the two roots of the equation x3 + 5x + 12 = 0 is 4. Find the third root. Ans. -3

12. The sum of the roots 3x2 – 2mx2 + 4 = 0 is 6. Find m. Ans. m = 9

MISCELLANEOUS QUESTIONS

1. A set of elements that is taken without regard to the order in which the elements are arranged is called:

a. combination b. sequence c. permutation d. series

2. When a logarithm is expressed as an integer plus a decimal (between 0 and 1), the integer is called the

a. Briggs Logarithm c. Napierian Logarithmb. Characteristic d. Mantissa

3. Any positive integers as 1, 2, 3, etc. is also calleda. Real Number c. Natural Numberb. Rational Number d. Irrational Number

4. The set of integers that does not satisfy the closure property under the operation ofa. addition b. subtraction c. multiplication d. division

5. An equation which is satisfied by some, but not all, of the values of the variables for which the members of the equation are not defined is called a

a. linear equation c. rational equationb. conditional equation d. irrational condition

6. An equation which is satisfied by all of the values of the variables for which the members of the equation are defined is

a. linear equation c. rational equationb. conditional equation d. irrational equation

7. Two prime numbers which differ by 2 are called prime twins. Which of the following pairs of numbers are prime twins? a. (1,3) b. (13, 15) c. (7,9) d. (9,11)

8. A relation in which every ordered pair (x,y) has one and only one value of y corresponds to the values of x is called

a. term b. function c. coordinated d. abscissa

9. Tossing a coin is generally calleda. an experiment b. an event c. an outcome d. a trial

10. A polynomial which is exactly divisible by two or more polynomials is called as:a. least common denominator b. common multiplec. factors d. binomial

11. The roots of the equation 2x2 – 13x + 20 = 0 area. real and equal b. real and unequal

c. complex and equal d. complex and unequal

12. Each of two or more numbers which is multiplied together to form a product is called a. term b. multiplier c. kilogram d. kilowatt

13. In the SI unit, the small letter k means kilo while the capital letter K meansa. Kilometer b. Kelvin c. Kilogram d. Kilowatt

14. Any number that can be expressed as a quotient of two integers (division of zero excluded) is called

a. irrational number c. rational numberb. imaginary number d. odd number

15. A rectangular array of numbers forming m rows and n columns are called asa. determinants c. elementsb. Pascal’s triangle d. none of the above

16. In the expression n√a, the letter n represents thea. power b. order c. exponent d. radicand

17. A number of the form a + bi with a and b real constants and i is square root of -1 is called

a. imaginary number c. complex numberb. radical d. compound number

18. Which of the following nonterminating decimals is rationala. 3.14159265… c. 2.470470…b. 2.71828182… d.1.141421356

19. A succession of numbers in which one number is designated as first, another as second, another as third and so on is called a

a. series c. order of numbersb. arrangement d. sequence

Page 6: Mathematics

20. An equation in which a variable appears under a radical sign is calleda. irradical equation c. irrational equationb. quadratic equation d. linear equation

21. If 1/4 and – 7/2 are the roots of the quadratic equation Ax2 + Bx + C = 0, what is the value of B?

a. -28 b. -7 c. 4 d. 26

22. If 1/4 and – 7/2 are the roots of the quadratic equation Ax2 + Bx + C = 0, what is the value of C?

a. -28 b. -7 c. 4 d. 26

23. Radicals can be added if they have the same radicand anda. exponent b. power c. order d. coefficients

24. If the roots of ax2 + bx + c = 0, are real and equal, thena. b2 – 4ac > 0 c. b2 – 4ac < 0b. b2 – 4ac = 0 d. b2 – 4ac < 0

25. The sum of the integers between 288 and 887 that are exactly divisible by 15 isa. 23,700 b. 22,815 c. 21,800 d. 24,150

26. The sum of the prime numbers between 1 and 15a. 42 b. 41 c. 39 d. 38

27. What is the sum to infinity of the sequence 1 + 1/3 + 1/9 + …a. 2/5 b. 5/6 c. 2/3 d. 3/2

31. The term free of y in the expansion of isa. 46 b. 84 c. 47 d. 49

32. If f(x) = x + 2 and g(y) = y + 2, then f[g(2)] equals x – 2

a. 6 b. 5 c. 4 d. 3

33. If x3 + 3x2 + (K + 5)x + 2 – K is divided by x + 1 and the remainder is 3, then the value of K is

a. -2 b. -4 c. -3 d. -5

34. The value of K which will make 4x2 – 4kx + 5k a perfect square trinomial isa. 6 b. 5 c. 4 d.3

35. If 3x = 9y and 27y = 81z, then is equal toa. 3/7 b. 3/5 c. 3/4 d. 3/8

.

36. A certain work can be done in as many days as there are men in the group. If the number of men in the group is reduced by 3, the job will be delayed by 4 days. The number of men originally in the group is

a. 8 men b. 10 men c. 12 men d. 14 men

37. How many terms in the progression 3, 5, 7, 9, … must be taken in order that the sum is 2,600?

a. 53 b. 52 c. 51 d. 5038. The other form of logaN = b is

a. N = ab b. N = ba c. N = a/b d. N = ab

39. In the quadratic equation Ax2 + Bx + C = 0, the product of the root is:a. C/A b. –B/A c. –C/A d. B/A

40. Two students were solving a problem that would reduce it to a quadratic equation. The first student committed an error in the constant term and found the roots to be 5 and 7 while the second student made an error in the first degree term and gave the roots as 2 and 16. if you were to check their solutions, the right equation is:

a. x2 + 12x + 35 = 0 b. x2 + 18x + 32 = 0c. x2 + 7x – 14 = 0 d. x2 – 12x + 32 = 0

41. Determine the value of k so that the equation x2 + (k-5)x + k – 2 = 0 is a perfect trinomial square.

Ans. 3 or 11

42. The expression x4 + ax3 + 5x2 + bx + 6 when divided by (x – 2) leaves the remainder 16, and when divided by (x – 1) leaves the remainder 10. Find the values of a and b.

Ans. a = - 11/3, b = 5/3

43. Given the equation x4 + x2 + 1 = 0. Which of the following is not a root?a. 1 /120° b. 1 /135° c. 1 /240° d. 1 /300°

44. The area of a square field exceeds another square by 56 square meters. The perimeter of the larger field exceeds one half of the smaller by 26 meters. What are the sides of each field?

Ans. larger field, 9m or 25/3m; smaller field, 5m or 11/3m

45. The sum of the areas of two unequal square lots is 5,200 square meters. If the lots were adjacent to each other, they would require 320 meters of fence to enclose the combined area formed by them. Find the dimensions of each lot. Ans. 60m and 40m or 68m and 24m

BINOMIAL EXPANSION

1. Solve the following equations:A) Find the value of x: (a + b)x = (a2 + 2ab + b2)x-1

B) Find the roots of the equation 4x4 + 1 = 0

Page 7: Mathematics

2. Without expanding, find the term involving x4 of (3x2 – 2x-1)8. Ans. 90,720x4

3. A) Expand to 4 terms (x2/3 – ½)x13

B) Find the 9th term in the expression of (x2 + ½)13

C) Write the first four terms and the last term of the expansion of (3/x – x/3)65

D) Find the term independent of y in the expansion of (y2 – y-1)9.Ans. 84

4. Which of the following has no middle term?a. (x + y)3 b. (a – b)4 c. (u + v)6 d. (x – y)8

5. Find the middle term of (x2 – 2y)10

Ans. -8,064x10y5

6. If the middle term in the expansion of (x + 2y)n is kx4y4, find k and n.Ans. n = 8, k = 1,120

7. If the rth term of (x2 – 2y3)n is Cx8y12, find the value of C. Ans. 1120

8. Without expanding, find the 10th term of the expansion of (S – 2t2)14

9. In the expansion of (x2 + 1/x)12

find: a) the 6th termb) the middle termc) the term involving x6

d) the term free of x

10. Find the sixth term in the expansion of (x/2 + y)9

Ans. 63/8x4y5

11. Find the term containing x26 in the expansion of (x-2 + x3)12

Ans. 66x26

12. The term containing x9 in the expansion of (x3 + 1/x)15

13. Find the coefficient of the expansion of (x2 + y)10 containing x10y5 a. 149 b. 252 c. 105 d. 10,818

AGE PROBLEMS

1. A father is twice older than his son and the sum of their ages is 48. How old is each?a. 8, 40 b. 12, 36 c. 16, 32 d. none of these

2. Maria is 36 years old, Maria was twice as old as Anna was when Maria was as old as Anna is now. How old is Anna now? Ans. 24 years old

3. The sum of the ages of the father and his son is 99. If the age of the son is added to the inverted age of the father, the sum is 72. If the inverted age of the son is subtracted from the age of the father, the difference is 22. What are their ages? Ans. 74 & 25

4. Maria is 24 years old now. Maria was twice as old as Ana when Maria was as old as Anna is now. How old is Anna now? Ans. 16 years old

5. A father is twice as old as his son and the sum of their ages is 48. How old is each?a. 8, 40 b. 12, 36 c. 16, 32 d. nota

6. Pedro is as old as Juan was when Juan is twice as old as Pedro was. When Pedro will be as old as Juan is now, the difference between their ages is 6 years. Find the age of each now. Ans. Juan, 24 years old and Pedro, 18 years old

7. I am three times as old as you were when I was as old as you are now. When you got to be my age together our ages will be 84. How old are we now?

a. 24 & 36 b. 24 & 8 c. 16 & 8 d. 18 & 27

8. The sum of the ages of two boys is four times the sum of the ages of a certain number of girls. Four years ago, the sum of the ages of the girls was one eleventh of the sum of the ages of the boys and eight years hence, the sum of the ages of the girls will be one half that of the boys. How many girls are there? Ans. 4 girls

9. In a family, there are 8 children, two of them are twins. The youngest is 3 years old and the eldest is 21 years old. Their ages are in arithmetic progression. There are three children younger than the twins. How old are the twins? Ans. 12 years

10. The sum of the ages of two men equals 99. If the inverted age of the elder is added to the age of the younger, the sum is 108. However, if the age of the younger is inverted and subtracted from the age of the older, the difference is 44. Find the age of the older man.

a. 67 b. 32 c. 53 d. 46

INTEGER AND DIGIT PROBLEMS

1. Separate 132 into 2 parts such that the larger divided by the smaller the quotient is 6 and the remainder is 13. What are the parts? Ans. 17 and 115

2. A number of two digits divided by the sum of the digits the quotient is 7 and the remainder is 6. If the digits of the number are interchanged, the resulting number exceeds three times the sum of the digits by 5. What is the number? Ans. 83

3. Six times the middle of a three digit number is the sum of the other two. If the number is divided by the sum of the digits, the answer is 51 and the remainder is 11. If the digits are reversed, the number becomes smaller by 198. Find the number. Ans. 725

Page 8: Mathematics

4. Find the number such that their sum multiplied by the sum of their squares is 65, and their difference multiplied by the difference of their squares is 5. Ans. 2 and 3

5. Three numbers are in the ratio 2:5:8. If their sum is 60, find the numbers. Ans. 8, 20, 32

6. The sum of the digits of a three-digit number is twelve. The sum of the squares of the hundreds’ digit and the tens’ digit is equal to the square of the units’ digits. If the hundreds’ digit is increased by two, the digits will be reversed. Find the number. Ans. 345

7. April 1978. The square of a number increased by 16 is the same as 10 times the number. Find the number. Ans. 8, 2

8. The sum of the digits of a three-digit number is 12. The middle digit is equal to the sum of the other two digits and the number shall be increased by 198 if its digits are reversed. Find the number. Ans. 264

9. Find three consecutive odd integers such that twice the sum of the first and the second integers plus four times the third is equal to 60. Ans. 5, 7, 9

10. The sum of the digits of a three-digit number is 12. The sum of the squares of the

hundreds digit and the tens digit is equal to the square of the units digit. If the hundreds digit

is increased by 2 and the units digit is decreases by 2, the digits of the original number will be

reversed. Find the number. Ans. 345

11. The excess of the sum of the fifth and the seventh parts over the difference of the half

and third parts of number is 259. What is the number? Ans. 1470

12. The sum of the digits of the three-digit number is 6. The middle digit is equal to the sum of the two other digits and the number shall be increased by 99 if the digits are reversed. Find the number. Ans. 132

MIXTURE PROBLEMS

1. The tank of a car contains 50 liters of alcogas 25% of which is pure alcohol. How much of the mixture must be drawn off which when replaced by pure alcohol will yield a 50-50% alcogas?

a. 16 2/3 b. 15 1/3 c. 14 d. 20

2. How much silver and how much copper must be added to 20kg of an alloy containing 10% silver and 25% copper to obtain an alloy containing 36% silver and 38% copper?

a. 14kg, 12kg b. 16kg,14kg c. 12kg,10kg d. 16kg, 18kg

3. A tank full of alcohol is emptied of one third of its content and then filled up with water and mixed. If this is done six times, what fraction of the volume (original) of alcohol remains? Ans.64/729

4. How much tin and how much iron must be added to 50 kilograms of an alloy containing 10 percent tin and 25 percent iron to obtain an alloy containing 25 percent tin and 50 percent iron? Ans. 27.5 kg(tin), 52.5 kg (iron)

5. How many liters of water must be added to 45 liters of solution which is 90% alcohol in order to make the resulting solution 80% alcohol? Ans. 5.63L

6. A 40-gram solution of acid and water is 20% acid by weight. How much pure acid must be added to this solution to make it 30% acid? Ans. 5.71 grams

7. How much water must be evaporated form 80 liters of 12% solution of salt in order to obtain a 20% solution of salt? Ans. 32 L 39. How many liters of water must be added to 100 liters of 85% sulfuric acid solution to produce 60% sulfuric acid solution? Ans. 41.67 L

8. A certain solution should contain 8% alcohol. If it was mistakenly mixed to contain 6% alcohol, how much must be drawn from a 5-liter tank and replaced by 10 percent alcohol solution to provide the proper concentration? Ans. 2.5 L

9. A certain amount of 80% sugar solution added to another amount of 40% sugar solution yields a solution that contains 14 kg of sugar. Had the amount been reversed, the solution would have contained 16 kg sugar. How much of the 80% solution was there? Ans. 10 kg

10. Ten liters of 25% salt solution and 15 liter of 35% salt solution are poured into a drum originally containing 30 liters of 10 % salt solution. What is the percent concentration of salt in the mixture?

a.19.55% b. 22.15% c. 27.05% d. 25.72%

RATE AND MOTION PROBLEMS

1. A motorist is traveling from town A to town B at 60 kph and returns from town B to town A at 30 kph. His average velocity for the roundtrip is

A. 45 kph B. 40 kph C. 35 kph D. NOTA

2. At the recent Olympic games in Montreal, Canada, a team which participated in 1600 meters relay event had the following individual speed. First runner, 24 kph, second runner, 20 kph, third runner, 22 kph and fourth runner 23 kph. What was the team’s speed. Ans. 22.149 kph

3. A troop of soldiers marched 15 km, going to the concentration camp after they were forced to surrender, at the same time that the victorious general who is supervising the

Page 9: Mathematics

“march” rode from the rear of the troop to the front and back at once to the rear. If the distance covered by the victorious general is 25 km. and both the troop and general traveled at uniform rate, how long is the troop? Ans. 8 km.

4. Two cyclists are practicing on a circular tract of circumference 276 meter. Starting at the same instant and from the same place, when they run in opposite directions they pass each other every 6 seconds and when they run the same direction the faster passes the slower at every 23 seconds. Determine their rates. Ans. F= 29 m/s, S= 17m/s

5. Two cars A and B start at the same point and at the same time and travel in opposite directions, car B traveling 20 km/hr slower than A. If they are 420 kilometers apart after 3 hours, find the rate of each. Ans. 60 kph, 80 kph

6. Two cars A and B are to race around a 1,500-meter circular track. If they will start at the same point and travel opposite directions, they will meet for the first time in 3 minutes. But if they will travel in the same direction, with the same starting point, car A will reach the starting point with car B trailing behind by 500 meters. What should be the rates of each?

Ans. 300 m/min, 200 m/min

7. A one kilometer long caravan of men is walking at a constant rate. A man from the rear ends walk towards the head and back to the rear at the instant when the caravan has covered a distance of one kilometer. Find the total distance traveled by the man. Ans. 2.414 km

8. A boy started one hour and twenty minutes earlier than a man. If the man ran at 6 kph faster than the boy and overtook the boy in 40 minutes, find the rate of each. Ans. 3 kph for the boy and 9 kph for the man

9. A man walked 24 km in time T. During the first part of this time, he walked at 6 kph and the last part at 4 kph. Had he reversed his rates, he would have walked two km more. Find the time. Ans. 5 hrs.

10. A traffic check counted 390 cars passing a certain spot on one day and 430 cars at the same spot on the second day. On the first day, there were three times as many cars going east and half as many going west on the second day. What was the total number of west bound cars for the two days?

Ans. 280 east, 540 west11. A man is sent to deliver an important package ant travels by car 75 kilometers per hour from point A to B and then by airplane to point C against a wind blowing 40 kilometers per hour. The airplane can fly 280 kilometers per hour in still air. If the package carrier takes 3 2/3 hours in going from A to C and 3 1/6 hours for the return trip, what is the total distance of travel covered by the man? D = 605 km. t1 = 1.5 hr., t 2 = 5/3 hr.

12. A motorcycle messenger left the rear of a motorized troop 8 kilometers long and rode to the front of the troop, returning at once to the rear. How far did he ride, if the troop traveled 15 kilometers during this time and each traveled at a uniform rate? Ans. 25kms.

13. An army officer made the first part of the trip on a plane which flew at the rate of 210 kilometers per hour. At the landing field, he was met by a jeep which took him the rest of the way to his destination at a rate of 40 kilometers per hour. The trip required 3 hours and 15 minutes. On his return trip, the jeep traveled at the rate of 50 kilometers per hour and the plane which he took flew at the rate of 200 kilometers per hour. The return journey required the same amount of time, but this included a minute which he spent waiting for the plane to take off. Find the total distance that he flew and the total distance that he traveled by jeep.

Ans. 532km (by plane) and 28 2/3km (by jeep)

14. A BMW car drives from A toward C at 30 miles/hr. Another car starting from B at the same time, drives towards A at 20 mi/hr. If AB = 20 miles, find when the cars will be nearest together. Ans. 24 min.

15. Two boats started their voyages is in a straight line towards each other. One has an average navigational speed of 30 km/h and the other one has an average of 20kph. Assuming that they can not avoid a collision, how long will it take before the collision occurs? How far would each boat have traveled before the collision? Ans. 4 hrs, 120 km, 80 km

16. A man traveling 40 km finds that by traveling one more km per hour, he would made the journey in 2 hours less time. How many kilometer per hour did he actually travel?

a. 4 b. 8 c. 18 d. 6

PROGRESSION

1. Two numbers differ by 40 and their arithmetic mean exceeds their positive geometric mean by 2. The numbers are

a. 45, 85 b. 64, 104 c. 81, 121 d.100, 140

2. A sets out to walk at the rate of four km per hour. After he had been walking for 2-3/4 hours, B sets out to overtake and went 4-1/2 km the first hour, 4-3/4 km the second hr., 5 km the third hr and so on gaining 1 quarter of a km. every hour. In how many hours would B overtake A? Ans. 8 hours

3. A besieged fortress is held by 5700 men who have provisions for 66 days. If the garrison loses 20 men each day, for how many days can the provisions hold out? Ans. 76 days

4. The sum of three numbers in arithmetic progressions is 60. If the numbers are increased by 2, 1, and 28, respectively, the new numbers will be in geometric progression. Find the arithmetic progression.

5. Three numbers are in arithmetic progression. Their sum is 15, and the sum of their squares is 83. Find the numbers.

Page 10: Mathematics

6. A 20-liter container is filled with pure acid. Five liters are drawn off and replaced with water; then 5 liters of the mixture drawn off and replaced with water, and so on until 5 drawings and 5 replacements have been made. Find the amount of acid in the final mixture.

7. A rubber ball is dropped from a height of 27 meters. Each time that it hits the ground it bounces to a height 2/3 of that from which it fell. Find the distance that it travels up to the time that it hits the ground for the 5th time. The total distance traveled by the ball until it comes to rest.

8. A man wishes to buy a piece of land worth 150,000 pesos. If it were possible for him to save one centavo on the first day, two centavos on the second day, 4 centavos on the third day and so on, in how many days would he save to be able to buy the land? Ans. about 24 days

9. The sum of the two numbers is 20 and their positive geometric mean is one greater than one half of their arithmetic mean. Find their difference.10. A man cuts a piece of paper 0.03 mm thick into three equal parts. Then he cuts each of these parts into three equal parts again and the process is repeated 10 times after which he piles together the pieces of paper. How thick is the pile?

11. A rich man called his seven sons. He had with him a number of pebbles, each pebble representing a gold bar. To his first son, he gave half the pebbles that he had and one pebble more. To his second son, he gave half the remaining pebbles and one pebble more. He did the same to each to his five other sons and then found out that he had one pebble left. How many pebbles were there initially? Ans. 382

12. A man receives a salary of P36,000 per annum for the first year and a 10% raise every year for ten years. What is his salary during the fifth year? Ans. P52,707.60

13. A car running at 25 kilometers per hour can cover a certain distance in 8 hours. By how many kilometers per hour must its rate be increased in order to cover the same distance in three hours less? Ans. 15km/hr

14. Find the harmonic mean 7, 1, 5, 2, 6 and 3a. 2.36 b. 2.46 c. 2.56 d.2.66

15. The 8th term of an AP is 3 while its 84th term is 273. Find the 35th term.

16. Find the sum to infinity of 1/3, 1/27,1/243… Ans. 3/8

17. In the series 1.01, 1.0, .099, .098… Find the 80th term.

18. Find the sum of 3+0.4+0.05+0.004+0.0005+… Ans. 38/11

19. An arithmetic progression starts with 1, has 9 terms, and the middle term is 21. Determine the sum of the first 9 terms.

20. A pendulum swings 24 inches for the first time. It is swinging 11/12 of its previous swing. What would be the total distance traveled when the pendulum stopped?

a. 246 b.264 c.288 d. 312

21. A small line truck hauls poles from substation stockyard to pole sites along a proposed distribution line. The truck can handle only one pole at a time. The first pole site is 150 meters from the substation and the poles are to be 50 meters apart. Determine the total distance traveled by the line truck, back and forth, after returning from delivering the 30th pole.

a. 35.0km b. 30.0km c. 37.5km d. 40.0km

22. Two positive numbers may be inserted between 3 and 9 such that the first three are in geometric progression, while the last three are in arithmetic progression. What is the sum of these two positive integers?

a.1.25 b. 12.25 c. 11.25 d.6.25

23. A man piles 150 logs in layers so that the top layer contains 3 logs and each lower layer has one more log than the layer above. How many logs are at the bottom? Ans. 17 logs

24. A body falls 16.1 meters during the first second, 48.3 meters during the second, 80.5 meters during the third second and so on. How long will it take the body to reach the ground if it was released at an altitude of 15,000 meters? Ans. 30.5 seconds

25. The 18th and the 52nd terms of an arithmetic progression are 3 and 173, respectively. The 25th term is

a. 38 b. 35 c. 28 d. 25

25. Find the number of terms of a geometric progression in which the first term is 48, the last term is 384 and the sum of the terms is 720. Ans. 4 terms

26. The sum of three numbers in A.P. is 27. If the first number is increased by 2, the second by 7, and the third by 20, the resulting numbers will be in G.P. Find the original numbers.

a. 3, 9, 15 b. 4, 9, 14 c. 5, 9, 13 d. 6, 9, 12

WORK AND DISCHARGE PROBLEMS

1. If 4 men can plow 12 hectares in 8 hours, how many men are needed to plow 24 hectares in 24 hours? Ans. 6 men

2. A garden can be cultivated by 8 boys in 5 days. The same job can be done 5 men in 6 days. How long will it take to finish the job if A) the 8 boys and 5 men will work together? B)

Page 11: Mathematics

only 6 boys ands 3 men will work together? C) two days after the 5 men were working the 8 boys arrived to help?

3. A, B and C can do a piece of work in 10 days, A and B can do it in 12 days, A and C in 20 days. How many days would it take each to do the work alone? Ans. 30, 20 ,60

4. A boat’s crew rowing at half their usual rate can negotiate 2km. down a river and back in one hour and 40 minutes. At their usual rate in still water, they would have gone over the same course in 40 min. Find their rate of rowing in still water. Ans. 32/5 km/hr

5. Two pipes running simultaneously can fill a tank in 3 hours and 20 minutes. If both pipes run for 2 hours and the first is then shut off, it requires 2 hours more for the second to fill the tank. How long does it take each pipe to fill it alone?

6. A and B can do a job in 12 days. A and C can do the same job in 18 days while B and C can do it in 24 days. How will it take A, B and C to do the job together?

7. A man can finished a certain job in three-fourths the time that the boy can; the boy can finish the same job in two-thirds the time that a girl can; and the man and the girl working only jointly can finish the job in 4 hours. How long will it take to finish the job if they all work together? Ans. 8/3 hr.

8. The intake pipe to a reservoir is controlled by a valve which automatically closes when the reservoir is full and opens again when four-fifths of the water had been drained off. The intake pipe can fill the reservoir in 4 hours and the outlet pipe can drain it in 10 hours. If the outlet pipe remains open, how much time elapses between the two instants that the reservoir is full? Ans. 13.3 hr.

9. Two brothers washed the family car in 24 minutes. Previously, when each had washed the car alone, the younger boy took 20 min. longer to do the job than the older boy. How long did it take the older boy to wash the car alone? Ans. 40 minutes

10. A swimming pool holds 54 cubic meters of water. It can be drained at a rate of one cubic meter per minute faster than it can be filled. If it takes 9 minutes longer to fill it than to drain it, find the drainage rate. Ans. 3 cu.m/min

11. One input pipe can fill a tank alone in 8 hrs. another input pipe can fill it alone in 6 hours and a drain pipe can empty the full tank in 10 hours. If the tank is empty and all the pipes are wide open, how long will it take to fill the tank? Ans.5.22 hours

12. A steel company has three blast furnaces of varying sizes. If furnaces A, B, C are used full time, 800 metric tons of steel are produced per day. If A and B are used half time and C full time, 545 metric tons are produced. If A is not used, B is used full time, and C half dime, 410 metric tons are produced. How many metric tons per day does each furnace produce?

13. Three observation planes A, B, and C, working together, can map the region in 4 hours. Planes A and B can map the region in 6 hours, planes B and C can map it in 6 hours and 40 minutes. How long would it take each of the planes working alone to map this region?

14. A pump discharging 9 gpm requires 36 hours to fill a tank. If the pump is replaced by one that will discharge 16 gpm, how long will it take to fill the tank?

a. 64 hr b. 16 hr. c. 20.25 hr d. 40.5 hr

15. Two pipes running simultaneously can fill a swimming pool in 6 hours. If both pipes run for 3 hours and the first pipe is then shut off, it requires 4 hours more for the second to fill the pool. How long does it take each pipe running separately to fill the pool? Ans. 8 & 24

16. A man and a boy can dig a trench in 20 days. It would take the boy 9 days longer to dig it alone than it would take the man. How long would it take the boy to dig alone?

a. 45 days b. 16 days c.25 days d. 4 days

17. A job can be done in as many days as there are men in the group. If the number of men is reduced by 3, the job will be delayed by 4 days. How many men are there originally in the group?

a. 6 b. 12 c. 20 d. 30

VENN DIAGRAM1. A certain part can be defective because it has one or more out of three possible defects; insufficient tensile strength a burr or diameter outside of tolerance limits. In a lot of 500 pcs:

19 have a tensile strength defect17 have a burr11 have an unacceptable diameter12 have tensile strength and burr defects

7 have tensile strength and diameter defects5 have burr and diameter defects2 have all three defects

a. how many have four defectsb. how many pcs have only a burr defectc. how many pcs have exactly two defects. Ans. 475, 2, 18

2. During the election, the total number of votes recorded in a certain municipality was 12,400 had 2/5 of the supporters of LABAN candidate stampede away from the pools and ½ of the supporters of GAD candidate behaved likewise, the LABAN candidates majority would have been reduced by 100. How many votes did the LABAN & GAD candidates actually

received? Ans. 7,000, 5,400

3. The President just recently appointed 25 Generals of the Phil. Army of these 14 have already served in the war of Korea, 12 in the war of Vietnam and 10 in the war of Japan. Therefore 4 who have served both in Korea and Japan, 6 have served both in Vietnam and Korea and 3 have served in Japan, Korea, and Vietnam. Ans. 2 generals

Page 12: Mathematics

4. A survey of 500 T.V. viewers proceed the following result: 285 watch football games

195 watch hockey games 115 watch basketball games 45 watch football and basketball 70 watch football and hockey 50 watch hockey and basketball 50 do not watch any of the 3 games,

How many watch the basketball games only?a.50 b.40 c.30 d.60

CLOCK PROBLEMS

1. At what time between 7 and 8 o’clock are the hands of the clock are A) at right angles B) straight line C) coinciding

2. The time is 3:00 o’clock and the hands of the clock are at right angles to each other. What is the nearest time of the clock such that the hands of it will be at right angles again? Ans. 32.72 minutes

3. How long will it be from the time the hour hand and the minute hand of a clock are together until they will be together again? Ans. 1 hr. and 5.45 min

4. At what time between 4 and 5 o’clock do the hands of the clock coincide? Ans. 4:21.82 o’clock5. It is exactly 3 o’clock. In how many seconds will the angle formed by the hour hand and the minute hand be twice the angle formed by the hour and the second hand? Ans. 22.4 seconds

6. It is now between 9 and 10 o’clock. In 4 minutes, the hour hand will be exactly opposite the position occupied by the minute hand 3 minutes ago. What is the time now? Ans. 9:20

7. How many minutes after 2:00 o’clock will the hands of the clock extend in the opposite directions for the first time?

a. 40.636 b. 41.636 c. 43.636 d. 42.636

8. A student left his home to attend a party one morning at past 6 o’clock and returned at past 3 o’clock. He noticed the hands of the wall clock have exchanged position. What exact time did he arrive?

a. 3:26.07 b. 3:34.62 c. 3:31.47 d. 3:32.19

9. How many times in one complete day will the hour and the minute hands coincide with each other?

a. 24 b. 23 c. 22 d. 25

RATIO, PROPORTION AND VARIATION

1. The kilowatt that can be transmitted safely by a shaft varies directly as the number of revolution it makes per minute and the cue of its diameter. If a shaft 3 centimeters in diameter making 200 revolutions per minute can safely transmit 60 kilowatt, what kilowatt can be safely transmitted by a 2-centimeter shaft making 300 revolutions per minute?

2. The time required for an elevator to lift a weight varies directly with the weight and distance through which it is lifted and inversely as the power of the motor. If it takes 30 seconds for a 10 HP motor to lift 100 lbs through a height of 50 ft, what size of motor is required to lift 800 lbs in 40 sec through the height of 40 ft? Ans. 48 HP

3. Eight men can excavate 15m3 of drainage open canal in 7 hrs. Three men can backfill 10m3 in 4 hrs. How long will it take 10 men to excavate and back fill 20 m3 in the project? Ans. 9.87 hrs.

4. A man is sent to deliver an important package and travels by car 75 kilometers per hour from point A to B and then by airplane to point C against a wind blowing 40 kilometers per hour in still air. If the package carrier takes 3 2/3 hours in going from A to C and 3 1/6 hours for the return trip, what is the total distance of ravel covered by the man?

5. A sphere 30 cm in diameter is divided into two segments. One of which is two times as high as the other. Find the volume of the bigger segment.

6. Two flywheels are connected by a belt. The radius of the flywheels are 30 in and 50 in. The small flywheel has a speed of 350 rpm. Determine the velocity of the belt in ft/sec. What would be the angular velocity of the larger flywheel?

7. A cylindrical tin can has its height equal to the diameter of its base. Another cylindrical tin can with the same capacity has its height equal to twice the diameter of its base. Find the ratio of the amount of tin required for making the two cans with covers. Ans. 0.9524

8. The diameters of two spheres are in the ratio 2:3 and the sum of their volumes is 1,260 cubic meters. Find the volume of the larger sphere.

Ans. 972 cu. M

9. If the square root of x varies directly as y and inversely as the square of z and if x = 16 when y = 24 and z = 2, find z when x = 9 and y =2. Ans. 2/3

10. If a:b = 2:3, b:c= 4:5, what is a:b:c?a. 2:3/4:5 b. 8:12:16 c. 8:12:16 d. 6:9:12

Page 13: Mathematics

ANALYTIC GEOMETRY

Distance Between Two Points P1 (x1,y1) and P2 (x2,y2) y

P2(x2,y2) d

P1(x1,y1)

o x

d = (x2 – x1)2 + (y2 – y1)2

Area of Polygon (Non-overlapping) of n-sides Given VerticesGiven vertices (x1, y1), (x2, y2), ……… (xn, yn) oriented counterclockwise

x1 x2 x3 …………x1

A =y1 y2 y3 …………y1

+ + +

A = ½ [ (x1y2 + x2y3 + x3y4 ……. + xny1) – (y1x2 + y2x3 + y3x4 ……. + ynx1)]

Division of Line SegmentLet P(x, y) be a point on the line joining P1(x1, y1) and P2(x2, y2) and located in such a way that segment P1P is a given fraction k of P1P2, that is P1P = kP1P2.

y P2(x2,y2)

P(x,y) x = x1 + k (x2 – x1)y = y1 + k (y2 – y1)

P1(x1,y1)

o x

If k = ½, then formula above becomes a midpoint formulax0 = ½ (x1 + x2) ; y0 = ½ (y1 + y0)

Angle Between Two Concurrent LinesLet and be the inclinations of lines L1 and L2 respectively and let be the angle between the two lines

y L1 = - ;m1 = tan , m2 = tan

L2 tan = tan ( - ) tan = tan - tan __ 1 + tan tan tan = m2 – m1 1 + m1m2

SUPPLEMENTARY PROBLEMS

1. A point P(x, 3) is equidistant from points A(1, 5) and B(-1, 2). Find x. Ans. ¾

2. Find the locus of points P(x, y) such that the distance from P to (3, 0) is twice its distance to (1, 0). Ans. 3x2 – 3y2 – 2x – 5 = 0

3. Find the length of the segment joining the two midpoints of the sides of the triangle if the length of the third side opposite to it is 30 cm. Ans. 15 cm.

4. A line from P(1, 4) to Q(4, -1) is extended to a point R so that PR = 4PQ. Find the coordinate of R. Ans. R(13, -16)

5. Two vertices of a triangle are (0, -8) and (6, 0). If the medians intersect at (9, -3), find the third vertex of the triangle. Ans. (-3, -1)

6. The area of a triangle with vertices (6, 2), (x, 4) and (0, -4) is 26. Find x.Ans. – 2/3 and 50/3

7. Find the length of the median from A of a triangle ABC given vertices A(1, 6), B(-1, 3) and C(3, -3). Ans. 6

8. If the midpoint of a segment is (5, 2) and one endpoint is (7, -3), what are the coordinates of the other end? Ans. (3, 7)

9. Given vertices of a triangle ABC :A(1, 5),B(-1, 1) and C(6, 3). Find the intersection of the median. Ans.(2, 3)

10. Find the inclination of the line 2x + 5y = 10. Ans. 158.2

Locus – the curve traced by an arbitrary point as it moves in a plane is called locus of a point.

– the locus of an equation is a curve containing only those points whose coordinates satisfy the equation.

EQUATION OF A STRAIGHT LINE

Line – is a locus of points which has constant slope.

Theorems :

Page 14: Mathematics

Every straight line can be represented by a first-degree equation. The locus of an equation of the first degree is always a straight line.

General Equation of a Line

Ax + By + C = 0 ; A, B, C are constants ; A and B, not zero at the same time

Standard Equation of a Line

1. Two Point Form Y P2(x2,y2)

P(x,y) By similarity of triangles

y – y1 = y2 – y1 (x – x1)

P1(x1,y1) x x2 – x1

y

2. Point-Slope Form P(x, y)In (1) replacing y2 – y1 by m,

x2 – x1

y – y1 = m (x – x1)

x

m = tan

0 180

3. Slope-Intercept Formy = mx + b (0, b)

where m = slope P(x,y)

b = y intercept

4. Two- Intercept Formx + y = 1 P2(0, b)

a b P(x, y)

where a = x intercept

b = y intercept P1(a, 0)

5. Normal Equation of a Straight Line

Given = normal intercept N(cos,sin)

= segment from the origin

perpendicular to the required line sin

= normal angle cos

= inclination of the normal intercept

From the point slope form :

y – y1 = m (x – x1) where x1 = cos , y1 = sin

mL = -1 / tan

y - sin = (-1/ tan ) (x–cos )

Simplifying, xcos + ysin =

Reduction to Normal Form :

Given the line Ax + By + C = 0

The normal form is :

A x + B y + C = 0

A2 + B2 A2 + B2 A2 + B2

Note : The sign of the radicand must be chosen such that the last term will become negative since > 0.

Special Cases of a Straight Line

A. Equation of the x – axis: y = 0 Equation of a horizontal line : y = b where b is a constant

(1)

y

x

y

x

y

x

Page 15: Mathematics

B. Equation of the Y-axis : x = 0 Equation of a vertical line : x = a where a is a constant

SUPPLEMENTARY PROBLEMS:

Find the equations of the line/s satisfying the given conditions.

1. Passing through (1, -2) and perpendicular to the line through (2, -1) and (-3, 2)Ans. 5x – 3y – 11 = 0

2. With x intercept of 5 and passing through (3, 4) Ans. 2x + y – 10 = 0

3. Passing through (-3, 4) and with equal intercepts Ans. x – y + 7 = 0 and x + y – 1 = 0

4. Making an angle of 45 with the x-axis and passing through (2, 3) Ans. x – y – 1 = 0

5. With slope -12/5 crosses the first quadrant and forms with the axes a triangle with perimeter of 15. Ans. 5x + 12y – 3 = 0

6. Passing through (7, -4) and at a distance of 1 unit from the point (2, 1)Ans. 4x + 3y – 16 = 0 ; 3x + 4y – 5 = 0

7. Passing through the midpoint of the segment joining the points (1, 3) and (5, 1) and parallel to the line 2x – y + 5 = 0 Ans. 2x – 3y – 5

8. Find the value of parameter k so that the line 3x – 5ky + 5 = 0a) will pass through (0, 1)b) will be parallel to x + 2y = 5c) will be perpendicular to 4x + 3y = 2d) has the y-intercept equal to 3

Ans. a) 1 b) –6/5 c) 4/5 d) 1/3

9. Find the equations of the lines parallel to the line x + 2y – 5 = 0 and passing at a distance 2 from the origin Ans. x + 2y + 25 = 0 and x + 2y - 25 = 0

10. Find the equation of the perpendicular bisector of the segment joining (2, 5) and (4, 3).Ans. x – y + 1 = 0

11. Given vertices of a triangle ABC, A(2, 0); B(3, -2) and C(7, 5)a) find the equation of the median from Ab) find the equation of the altitude from Bc) the intersection of medians from B to C

Ans. a) x – 2y – 2 = 0 b) x + y – 1 = 0 c) (4, 1)

12. Find the normal intercept and the normal angle of line 5x+12y–39 = 0Ans. =3, = 67.38

y

Distance Between Parallel Lines L1

Let the parallel lines be given by the equations :

L1 : Ax + By + C1 = 0 L2

L2 : Ax + By + C2 = 0

The distance between the two

lines is given by the formula

d = C2 – C1_

A2 + B2

Sample Problems:

1. Find the distance from point (3, -1) to the line 3x – 4y – 3 = 0Solution :

Here, A = 3, B = -4, C = -2 P0(x0, y0) (3, -1)

Using the formula d = Ax0 + By0 + C = 3(3) + (-4)(-1) – 3

A2 + B2 + 32 + 42

d = 2 units (the point (3, -1) and the origin are on the opposite side of the line)

2. Find the distance between parallel lines 8x + 15y + 18 = 0 and 8x + 15y + 1 = 0.Solution :

A = 8, B = 15, C2 = 18, C1 = 1

D = C2 – C1 = 18 – 1 = 1 unit Answer

A2 + B2 82 + 152

Distance from a Point to a Line

The directed distance from a point P(x0,y0) to a line Ax + By + C = 0 is given by the formula:

d = Ax0 + By0 + C

A2 + B2

where the sign of the radical is chosen to be the opposite that of C.

Remarks:

1. If d > 0, the origin and P lie on opposite sides of the given line.2. If d < 0, the origin and P lie on the same side of the line. Notes:

Regardless of the location of the point P0(x0, y0), the distance being always positive the formula can be expressed using the absolute value as:

d =Ax0 + By0 + C

d

Page 16: Mathematics

A2 + B2

Line Through the Intersection of Two Lines

Let Ax + By + C = 0 and

Dx + Ey + F = 0 be two intersecting lines, where A, B, C, D, E and F are constants

and A = B 0, E = F 0.

The equation of the family of lines passing through the intersection of the two given lines is given by,

(Ax + By + C) + k (Dx + Ey + F) = 0

where k is an arbitrary constant.

INTERCEPT OF A CURVE x-intercept – directed distance from the origin to the point where the curve crosses the x-

axisTo find the x intercept of a curve, set y = 0, then solve for x.

y-intercept – the directed distance from the origin to the point where the curve crosses the y-axis

to find the y- intercept of a curve, set x = 0, then solve for y.

SYMMETRY If the equation of a curve does not change upon replacement of y by –y, then the

locus is symmetric with respect to the x-axis.f(x, -y) = f(x,y) =0

If an equation of a curve does not change upon replacement of x by –x, then the locus is symmetric with respect to the y-axis

f(-x,y) = f(x,y) = 0 If an equation of a curve does not change upon replacement of x by –x and y by

–y, then the locus is symmetric with respect to the origin.f(-x, -y) = f(x, y) = 0

ASYMPTOTE - a straight line which the curve f(x, y) = 0 approaches indefinitely near as its tracing point approaches to infinity.

To find the vertical asymptote, solve the equation for y in terms of x and set the linear factors of the denominator equal to zero.

To find the horizontal asymptote, solve the equation for x in terms of y and set the linear factors of the denominator equal to zero.

CIRCLECircle is the locus of a point which moves so that it is always equidistant from a fixed point.Note: fixed point is called the center Fixed distance is called the radius

Equation of a Circle

In normal form

Consider a circle of radius r with center at C(u, k)Let P(x, y) be a point in the circle

y

P(x,y) By Pythagorean Theorem r y–k (x – h)2 + (y – k)2 = r2 standard form

C(h,k)Center at the origin C(0, 0)

x2 + y2 = r2

x0

General Form

Expanding the form (x – h)2 + (y – k)2 = r2 becomesx2 + y2 – 2xh – 2ky + h2 + k2 – r2 = 0

This is of the form:x2 + y2 + Dx + Ey + F = 0 general formwhere D, E, F are constants not all zero at a time.

Note: By equation of coefficients:-2h = D ; h = -½ D abscissa of center-2k = E ; k = -½ E ordinate of centerh2 + k2 – r2 = F ; r = (h2 + k2 – F)

Radical Axis of Two CirclesConsider the two non-concentric circles

x2 + y2 +D1x + E1y + F1 = 0x2 + y2 +D2x + E2y + F2 = 0

The equation:x2 + y2 +D1x + E1y + F1 + k (x2 + y2 +D2x + E2y + F2) = 0represent a circle for any value of k except for k = -1

if k = -1, the equation of the family of circles above becomes:(D1 – D2) x + (E1 – E2) y + (F1 – F2) = 0This represents a straight line called the RADICAL AXIS of two circles.

Properties of the Radical Axis

x–h

Page 17: Mathematics

A. If two circles intersect at two distinct points, their radical axis is the common chord of the circles.

Common Chord

Condition for OrthogonalityThe two non-concentric circles : x2 + y2 + D1x + E1y + F1 = 0

x2 + y2 + D2x + E2y + F2 = 0, meet at right angles (orthogonal) if : D1D2 + E1E2 = 2(F1 + F2)

B. If two circles are tangent, their radical axis is the common tangent to the circles at their point of tangency.

Radical Axis

C. The radical axis of two circles is perpendicular to their line of centers.

Radical AxisAB – line of centers

A B

D. All tangents drawn to two circles from a point on their radical axis have equal lengths.

y Radical Axis P

T1 & T2 are points of T1 tangency

T2

PT1 = PT2

x

Supplementary Problems1. Find the center and radius of the circle whose equation is x2 + y2 – 4x –6y –12 = 0 (ECE Board Problem – Oct 1981) Ans: C(2, 3) r = 5

2. Find the area of the circle whose equation x2 + y2 = 6x – 8y(ECE Board Problem – Mar. 1981) Ans: 25 sq. units

3. Find the equation of the circle whose center is at (3, -5) and whose radius is 4 units.Ans: (x – 3)2 + (y + 5)2 = 16

For Problems 4 – 9, determine the equation of the circle given the following conditions4. Passes through the point (2, 3), (6, 1) and (4, -3) Ans: x2 + y2 – 10y = 0

5. Center on the y – axis, and passes through the origin and point (4, 2). Ans: x2 + y2 – 10y = 0

6. Passes through the points of intersection of the circles x2 + y2 = 5, x2 + y2–x + y = 4, and through the point (2, -3) Ans: x2 + y2 –2x + 2y –3 = 0

7. Center on the line x – 2y –9 = 0 and passes through the points (7, -2) and ( 5, 0) Ans: x2 + y2 – 10x + 4y +25 = 0

8. Circumscribe the triangle determine by the lines x – u – 8 = -y and y = -1.Ans: x2 + y2 –8x + 2y + 8 = 0

9. Given the endpoints of the diameter (5, 2) (-1, 2) Ans: x2 + y2 – 4x – 4y – 1 = 0

10. Find the equation of the line tangent to the circle x2 + y2 – 8x – 8y + 7 = 0 at the point (1, 0)

Ans: 3x + 4y – 3 = 0

PARABOLAThe locus of a point that moves in a plane such that its distance from a fixed point

equals its distance from a fixed line.

Notes: Fixed point is called focus Fixed line is called directrix Axis – the line passing through the focus and perpendicular to the directrix Vertex – The midpoint of the segment of the axis from the focus to the directrix. Latus rectum – a segment passing through the focus and perpendicular to the axis of the

parabola. Focal distance – distance from vertex to focus = a

y

x0

y

x0

y

x0

Page 18: Mathematics

Standard Equations of Parabola

A. Vertex at V(h,k), Vertical Axis(x-h)2 = 4a(y-k)

if a is positive (+a) ----- concave upwardif a is negative(-a) ----- concave downward

Notes:

1. Equation of axis : x=h y axis2. Focus : F(h,k + a) 3. End of Latus Rectum L F R

L(h-2a, k+a)R(h+2a, k+a)

4. Equation of Directrix V(h,k)y = k-a

directrix 0 x

B. Vertex at V(h,k), Horizontal Axis(y-k)2 = 4a(y-k)

if a is positive (+a) ----- concave to the rightif a is negative (-a) ----- concave to the left

Notes:1. Equation of the axis: y=k directrix2. Focus: F(h+a, k) y L 3. Ends of Latus Rectum:

L(h+a, k+2a) V F axisR(h+a, k-2a) (h,k)

4. Equation of Directrixx = h-a R

0 a>0 x

C. Vertex at the Origin, Vertical Axisx2 = 4ay

if a is positive (+a) ----- concave upwardif a is negative (-a) ----- concave downward

Notes: y1. Axis : the y axis axis 2. Focus: F(0,a)3. Latus Rectum: /4a/ F

Ends: L(-2a,a) L(-2a,a) R(2a,a)

R(2a, a)4. Equation of directrix

y = -a V(0,0)

directrix

D. Vertex at the Origin, Horizontal Axisy2 = 4ax

if a is positive (+a) ----- concave to the rightif a is negative (-a) ----- concave tot he left

Notes: y1. Axis : the x-axis directrix2. Focus: f(a,0) L 3. Latus Rectum = 4a

Ends: L(a,2a) V F R(a,-2a) x axis

4. Equation of Directrix x = -a R

Remarks:1. The vertex and focus always lie on the axis of the parabola.2. Focus is always located on the concave side of the parabola.

General Equations of Parabola

1. Vertical Axis Ax2 + Dx + Ey + F = 0, E or A must not be zero

2. Horizontal Axis Cy2 + Dx + Ey + F = 0, D or C must not be zero

Supplementary Problems1. Find the vertex, focus, and end points of the Latus Rectum of each of the following

parabolasa. 4y2 – x + 2y = 0

Ans: V(-1/4, -1/4), F(-3/16, -1/4) EL(-3/16, -1/4 1/8)

b. 2y2 – 5x + 3y - 7 = 0 Ans: V(-13/8, -3/4 ), F(-1, -3/4 ), EL(-1, -3/4 5/4 )

2. Find the equation of the parabola determined by the given conditionsa. focus at (-11/4, 1) and the endpoint of latus rectum is (-11/4, 5/2) Ans: y2 + 3x – 2y + 7 =

0

b. vertex at (1, -1) and focus at (1, -3/4) Ans: x2 – 2x – y = 0

c. vertex at (0, 3) directrix x = -1 Ans: y2 – 4x – 6y + 9 = 0

Page 19: Mathematics

d. axis vertical, vertex (-1, -1) and passing through (2, 2) Ans: x2 + 2x – 3y – 2 = 0

3. A chord passing through the focus of the parabola y2 = 16x has one end at the point (1, 4). Where is the other end of the chord?Ans: (4, 8)

4. Find the equation of the line tangent to the parabola x2 + 2x + 3y – 1 = 0 Ans: 2x + 3y – 1 = 0

5. Find the equation of the circle that passes through the vertex and the endpoints of latus rectum of the parabola y2 = 8x. Ans: x2 + y2 – 10x = 0

6. Find the equation of parabola whose axis is horizontal, vertex is on the y – axis and which through (2, 4) and (8, -2) Ans: y2 = 20y –18 +100 = 0, y2 = 4y + 2x + 4 =0

7. An arch in the form of parabolic curve, with a vertical axis is 60 m, across the bottom. The highest point is 16 m above the horizontal base. What is the length of a beam placed horizontally across the arch 3m below the top? Ans: 26 m

8. Assume that water issuing from the end of a horizontal pipe, 25 ft. above the ground describe a parabolic curve, the vertex of the parabola being at the end of the pipe, the flow of water has curve outward 10 ft. beyond a vertical line through the end pipe, how far beyond this vertical line will the water strikes the ground? Ans: 17. 68 ft.

ELLIPSEEllipse is the locus of a point P(x, y) in a plane which moves such that the sum of its

distances from two fixed points is constant.

Notes: The two fixed points are called foci. Major axis – the segment cut by the ellipse on the line containing the foci

- a segment joining the two vertices of an ellipse of length equal to I2aI

Diameters– the chords of an ellipse that pass through the center Vertices – the endpoints of the diameter through the foci

- the endpoints of the major axis Latus Rectum – the segment cut by the ellipse passing through the foci and

perpendicular to the major axis Eccentricity – measure the degree of flatness of an ellipse

Standard Equation of Ellipse

Center at C(h, k), Horizontal Major Axis y

(x – h)2 (y – k)2

------------ + ----------- = 1 a2 b2 Notes: b 1. Major axis : y = k V1 F1 C(h,k) F2 V2

2. Minor axis ; x = h 3. Vertices : V1(h-a, k) b

V2(h+a, k)4. Foci : F1(h-c, k)

F2(h+c, k) 0

Center C(h, k), Vertical Major Axis y V1

F1

(x – h)2 (y – k)2

–––––––– + –––––––– = 1 a2 b2

Notes: 1. Major axis : x = h C(h,k)2. Minor axis ; y = k F2

3. Vertices : V1(h, k+a) V2(h, k-a)

4. Foci : F1(h, k+c) V2

F2(h, k-c) 0 x

Center at the Origin, Horizontal Major Axis

(x)2 (y)2

------- + ------ = 1 y a2 b2

Notes:5. Major axis : x = axis6. Minor axis ; y = axis V1 F1 F2 V2 5. Vertices : V1(-a, 0) C(0,0)

V2(a, 0)7. Foci : F1(-c, 0)

F2(c, 0)

c ca a

c

c

a

a

b b

c ca a

Page 20: Mathematics

Center at the Origin, Vertical Major Axis V1

(x)2 (y)2 ------- + ------ = 1 F1

a2 b2

Notes: 8. Major axis : y = axis C(0,0)9. Minor axis ; x = axis 10. Vertices : V1(0, a) V2(0. -a) F2

11. Foci : F1(0, c) F2(0, -c) V2

General Remarks1. Vertices and foci lie on the major axis2. IaI is the distance from the center to the vertex3. IcI is the distance from the center to the foci ( focal distance)4. The ellipse is symmetrical to the major, minor axes and the center.

Important Relations

1. a > b, a > c2. a2 = b2 + c2

3. e = eccentricity = c/a < 14. Latus Rectum, LR = 2b2/a

General Equation of an Ellipse

Ax2 + Cy2 + Dx + Ey + F = 0 where A C but of the same sign

Supplementary Problems:1. In the ellipse below determine the following: a) Center b) Vertex c)Foci d) Major Axis e) Major Axis f) Latus Rectum g) Eccentricity

A. 25x2 + 16y2 – 50x + 32y – 1559 = 0B. 144x2 + 169y2 +864x – 23,760 = 0Ans: A. a) C(1, -1); b) V1(1, 9), V2(1, -11); c) F1(1, 5), F2(1, -7)

d) 20 e) 16 f) 12.8 g) 0.6 B. a) C(-3, 0); b) V1(-16, 0), V2(10, 0); c) F1(-8, 0), F2(2, 0) d) 26 e) 24 f) 288/13 g) 5/13

2. In each of the following find the equation of ellipse satisfying the given conditions

A. center at (0, 0), focus at (3, 0), and b = 1Ans: x2 + 4y2 = 4

B. center at (1, 0) focus at (1, 3) e= 3/2Ans: 4y2 + y2 = 8x

C. focus at (0, -1), (-4, -1), a = 6Ans: x2 + 3y2 + 4x + 6y +1 = 0

D. center at (-1/2, 2) a = 5/2 b = 2Ans: 16x2 + 25y2 + 16x + 4 = 100y

E. center at (0, 0), vertex (0, 4) e = ½Ans: 4x2 + 3y2 – 48 = 0

3. A satellite orbits around the earth in an ellipse orbit of eccentrically of 0.80 and semi – major axis of length 20,000 km. If the center of the earth is at one focus, find the maximum altitude (apogee) of the satellite. Ans: 36, 000 km

4. Find the equation of the locus of a point which moves so that the sum of its distance from (-2, 2) and (1, 2) is 5. Ans: 16x2 + 25y2 + 16x – 100y +4 = 0

5. Find the eccentricity of an ellipse whose major axis is thrice a long as its minor axisAns: 2 2/3

6. What is the quadrilateral formed by joining the foci of an ellipse to the endpoints of the minor axis? Ans: rhombus

7. Find the distance of the point (3, 4) to the foci of the ellipse whose equation 4x2 + 9y2 = 36

Ans: 3 5

8. An arch in the form of a semi – ellipse has a span at 45 m and its greatest height is 12m. There are two vertical supports equidistant from each other and the ends of the arc. Find the height of the support.

Ans: 8 2 m.

9. Determine the locus of a point P(x, y) so that the product of the slopes joining P(x, y) to (3, -2) and (-2, 1) is –6. Ans: 6x2 + y2 – 6x +y – 20 = 0

10. What is the area of the ellipse whose equation is 25x2 + 16y2 = 400. Ans: 20 sq. units

HYPERBOLA

Hyperbola is the locus of point P(x, y) in a plane which moves such that the difference of its distances from two fixed points is a positive constant.

Notes: The two fixed points are called foci Transverse axis – a line segment joining the two vertices of hyperbola

- the length of the transverse axis is I 2a I

c

c

a

a

b b

Page 21: Mathematics

Conjugate Axis – the perpendicular bisector of the transverse axis.- the length of the conjugate axis is I2b I

Center – point of intersection of transverse and conjugate axis Central Rectangle – the rectangle whose area is (2a) (2b) and whose diagonals

are asymptotes of hyperbola Vertices – the endpoints of the transverse axis Asymptotes of the hyperbola – two intersecting lines containing the diagonal of

the central rectangle–To find the equation of the asymptote, set the right side of the equation of hyperbola in standard form to zero then solve for y.

Central Circle – the circle of radius c with center at the center of the hyperbola circumscribing the central rectangle

Equilateral Hyperbola – hyperbola whose transverse axis equals its conjugate axis

Conjugate Hyperbolas – hyperbolas whose transverse axis of one is the conjugate axis of the other

Standard Equations of Hyperbola

Center at C(h,k), Horizontal Transverse Axis

(x – h) 2 _ (y – k) 2 = 1 y a2 b2

Asymptote

Notes: 1. Transverse axis: y = k 2. Conjugate axis: x = h

transverse3. Vertices: V1 (h – a, k) axis

V2 (h + a, k) conjugate axis4. Foci: F1 (h – c, k)

F2 (h + c, k)5. Asymptotes: y – k = b/a ( x – h) 0 x

Center at C(h, k), Vertical Transverse Axis

Asymptote(y – k)2 _ (x – h)2 = 1 Transverse axis

a2 b2

Notes: Conjugate axis

1. Transverse axis: x = h2. Conjugate axis: y = k3. Vertices: V1 (h, k + a)

V2 ( h, k – a)4. Foci: F1 (h, k + a)

F2 ( h, k – c)5. Asymptotes: y – k = a/b( x – h)

Center at the Origin, Horizontal Transverse Axis

x 2 _ y 2 = 1 ya2 b2

Notes:1. Transverse axis: x–axis2. Conjugate axis: y–axis3. Vertices: V1 (-a, 0) x

V2 (a, 0)4. Foci: F1 (-c, 0)

F2 (c, 0)5. Asymptotes: y = b/a x

Center at the origin , Vertical Transverse Axis

y 2 _ x 2 = 1 ya2 b2

Notes:

1. Transverse axis: y – axis 2. Conjugate axis: x – axis3. Vertices: V1 (0, a)

V2 ( 0, -a)4. Foci: F1 (0, c)

F2 (0, -c)5. Asymptotes: y = a/b x

General Remarks

1. Vertices and foci are on the transverse axis2. IaI is the distance from the center to the vertex3. IcI is the distance from the center to the focus.4. The hyperbola is symmetrical to the transverse and conjugate axis and to the center.

V1

V2

F1

F2

C

V1 V2F1 F2

C

V2

V1

F1

F2

C

V1 V2F1 F2

C

Page 22: Mathematics

General Relations

1. c > a, c > b ( a = b or a < b or a > b) If a = b, then the hyperbola is called equilateral hyperbola

2. c2 = a2 + b2

3. Length of Latus Rectum = 2b 2 a4. Eccentricity, e = c/a > 1

General Equation of HyperbolaAx2 + Cy2 + Dx + Ey + F = 0 where A and C are of opposite signs

Supplementary Problems1. Find the center, vertices, foci and asymptotes, transverse axis, conjugate axis, latus

rectum and eccentricity of hyperbola below.a. 9x2–16y2+18x+64y=91b. 16x2–4y2–62x+24y+92=0c. 25x2–16y2=400

Ans. a. C(–1,2); V(–12, 2); F(–15/2, 2); asymptote (3x+4y–5=0, 3x–4y+11=0); TA = 4; CA = 3;LR = 9/4; e = 5/4

b. C(2,3); V1(2,1),V2(2,7); F(2,3+20 ), F2(2, 3–20 ); asymptote(2x–y–1=0, 2x+y–7=0); TA = 8; CA = 4;LR = 2; e = 5/2

c. C(0,0); V1(–4,0),V2(4,0); F(–41 ,0), F2(41 ,0); Asymptote ( y = 5/4 x ); TA = 8; CA = 10;LR = 25/2; e = 41/4

2. Find the equation of the hyperbola satisfying the conditions given in each casea. Center(3,–1); vertex(1,–1); focus(0,–1)

Ans. 5x2–4y2–30x–8y+21=0

b. vertices at(0,4) and (4,4); foci at (5,4) and (–1,4)Ans. 5x2–4y2–20x+32y–64=0

c. Center at (1,1), vertex(1,3), eccentricity=2Ans. x2–3y2–2x +10 =0

d. Directrices: y=4; asymptotes: y=3/2xAns. 9y2–324x2–208 = 0

e. Asymptotes: 3y=4x ; foci (6,0)Ans. 400x2–225y2–5184=0

f. Foci(0,0), (0,10); asymptote: x+y=5Ans. 2x2–2y2+20y–25=0

g. Asymptotes: x+y=1 and x–y=1 and passing through (–3,4) and (5,6)Ans. x2–y2–2x–2=0

h. Axes along the coordinates axes, passing through (–2,5)Ans. 4y2–5x2=19

i. Vertices at (0, 4) passing through (–2,5)Ans. 4y2–9x2 = 64

3. Find the eccentricity of a hyperbola whose transverse axis and conjugate axis are equal in length Ans. e= 2

THE CONIC SECTIONS (a summary)A conic section is the locus of a point which moves such that its distance from a fixed

point called focus is in constant ratio and called eccentricity to its distance from a fixed straight line called directrix.

A. General Form of a Quadratic Equation in x and yAx2 + Cy2 + Dx + Ey + F = 01. Ellipse : A C, same signs2. Circle : A = C, same signs3. Hyperbola : A and C have opposite signs

B. Eccentricity e = c/a1. Circle : e = 02. Parabola : e = 13. Ellipse : e < 14. Hyperbola : e > 1

Note: The circle, parabola, ellipse and hyperbola are called conic sections (or conics) because

any one of them can be obtained geometrically by cutting a cone with a plane.

Page 23: Mathematics

Circle Ellipse Parabola Hyperbola

If the cutting plane is perpendicular to the axis of the cone, the section is a circle. If the cutting plane is making an angle ( other than 90o) with the axis of the cone, the

section is an ellipse. If the cutting plane is parallel to one of the elements of a cone, the section is a parabola If the cutting plane is parallel (but not coincident) to the axis of the cone, the section is a

hyperbola.

In each of the cases, the cutting plane should not pass through the vertex of the cone, otherwise the section hat will be formed is a degenerate conic.

Degenerate Conic ( one point, one line, two lines) is a conic formed if the cutting plane is passing through the vertex along one of its elements

Principal Axis of a Conic – is the line through the focus and perpendicular to the directrix

Diameter of a Conic – the locus of the midpoints of a system of parallel chords.Direction: Encircle the letter corresponding to the correct answer.1. The sum of the digits of a two-digit number is 11. If the digits are reversed, the resulting number is seven more than twice the original number. What is the original number? a. 38

b. 53 c. 83 d. 442. A metal washer 1-inch in diameter is pierced by ½-inch hole. What is the volume of the washer if it is 1/8 inch thick.

a. 0.074 b. 0.047 c. 0.028 d. 0.0823. If a regular polygon has 27 diagonals, then it is a,

a. nonagon b. pentagon c. hexagon d. heptagon4. Find the probability of getting exactly 12 out of 30 questions on a true or false question. a.

0.12 b. 0.08 c. 0.15 d. 0.045. Find the area bounded by the curve and the x-axis

a. 18 units2 b. 25 units2 c. 36 units2 d. 30 units2 6. It is a measured of relationship between two variables.

a. Function b. Relation c. Correlation d. Equation7. A central angle of 45 degrees subtends an arc of 12 cm. what is the radius of the circle? a.

15.28 cm b. 12.82 cm c. 12.58 cm d. 15.82 cm8. Two posts, one 8 m and the other 12 m high are 15 cm apart. If the posts are supported by a cable running from the top of the first post to a stake on the ground and then back to the top of the second post, find the distance from the lower post to the stake to use minimum?

a. 6 m b. 8 m c. 9 m d. 4 m9. A regular octagon is inscribed in a circle of radius 10. Find the area of the octagon.

a. 228.2 b. 288.2 c. 238.2 d. 282.2

10. The volume of the two spheres is in the ratio 27:343 and the sum of their radii is 10. Find the radius of the smaller sphere.a. 5 b. 4 c. 3 d. 6

11. Find the approximate change in the volume of a cube of side “x” inches caused by increasing its side by 1%.a. 0.30x2 in3 b. 0.02 in3 c. 0.1x3 in3 d. 0.03x3 in3

12. The time required for the examinees to solve the same problem differ by two minutes. Together they can solve 32 problems in one hour. How long will it take for the slower problem solver to solve a problem?

a. 5 minutes b. 2 minutes c. 3 minutes d.4 minutes13. If a = b, then b = a. This illustrates which axiom in Algebra?

a. Transitive axiom b. Replacement axiomc. Reflexive axiom d. Symmetric axiom

14. Find the distance of the directrix form the center of an ellipse if its major axis is 10 and its minor axis is 8.a. 8.5 b. 8.1 c. 8.3 d. 8.7

15. A regular hexagonal pyramid has a slant height of 4 cm and the length of each side of the base is 6 cm. Find the lateral area.a. 62 cm2 b. 52 cm2 c. 72 cm2 d. 82

16. At the surface of the earth 9 = 9.806 m/s2. Assuming the earth to be a sphere of radius 6.371 x 106 m, compute the mass of the earth.a. 5.12 x 1024 kg b. . 5.97 x 1023 kgc. 5.97 x 1024 kg d. . 5.62 x 1024 kg

17. The perimeter of an isosceles right triangle is 6.6824. Its area isa. 4 b. 2 c. 1 d. ½

18. Determine the vertical pressure due to a column of water 85-m high.a. 8.33 x 105 N/m2 b. 8.33 x 104 N/m2

c. 8.33 x 106 N/m2 d. 8.33 x 103 N/m2

19. a 40-gm rifle with a speed of 300 m/s strikes into a ballistic pendulum of mass 5 kg suspended from a cord 1 m long. Compute the vertical height through which the pendulum rises.

a. 28.87 cm b. 29.42 cm c. 29.88 cm d. 28.45 cm20. What is the area of the largest rectangle that can be inscribed in a semi-circle of radius 10?

a. b. 100 units2 c. 1000 units2 d. 21. The amount of heat needed to change solid to liquid is

a. condensation b. cold fusionc. latent heat of fusion d. solid fusion

22. Mr J. Reyes borrowed money from the bank. He received from the bank P1,842 and promise to repay P 2,000 at the end of 10 months. Determine the simple interest.

a.19.45% b. 15.70% c. 16.10% d. 10.29%23. Find the length of the vector (2,4,4)

a. 7.00 b. 6.00 c. 8.50 d. 5.1824. According to this law, “The force between two charges varies directly as the magnitude of each charge and inversely as the square of the distance between them.”

a. Law of Universal Gravitation b. Coulomb’s Law

Page 24: Mathematics

c. Newton’s Law d. Inverse Square Law25. A loan of P 5,000 is made for a period of 15 months, at a simple interest rate of 15 %, what future amount is due at the end of the loan period.

a. P 5,637.50 b. P5,937.50 c. P 5,900.90 d. P5,842.54

26. If .

a. b. c. d.

27. The integral of any quotient whose numerator is the differential of the denominator is the ____________.

a. cologarithm b. product c. logarithm d. derivative28. Find the nominal rate, which if converted quarterly could be used instead of 12% compounded semi-annually.

a. 14.02% b. 21.34% c. 11.29% d. 11.83%29. Evaluate the expression (1 + i2 )10 where I is an imaginary number.

a. 1 b. 0 c. 10 d. -130. A VOM has a selling price of P 400. If its selling price is expected to decline at a rate of 10% per annum due to obsolence, what will be its selling price after 5 years?

a. P 213.10 b. 249.50 c. 200.00 d. 236.2031. If then what is the value of angle A.

a. 90 b. 120 c. 100 d. 140

32. Find the sum of the rots of a. -1/2 b. -2 c. 2 d. ½

33. In a box there are 25 coins consisting of quarters, nickels and dimes with a total amount of $ 2.75. If the nickels were dimes, the dimes were quarters and the quarters were nickels, the total amount would be $ 3.75. How many quarters arethere?

a. 12 b. 16 c. 10 d. 534. A point moves so that its distance from the point (2,-1) is equal to its distance from the axis. The equation of the locus is.

a. b.

c. d. 35. You loan from a loan firm an amount of P 100,000 with a rate of simple interest of 20% but the interest was deducted from the loan at the time the money was borrowed. If at the end of one year you have to pay the full amount of P 100,000 what is the actual rate of interest.

a. 25.0% b. 27.5% c. 30.0% d. 18.8%36. It is a polyhedron of which two faces are equal polygons in parallel planes and the other faces are parallelograms.

a. Tetrahedron b. Prism c. Frustum d. Prismatoid37. A railroad is to be laid –off in a circular path. What should be the radius if the track is to change direction by 30 0 at a distance of 157.08 m?

a.150 m b. 200 m c. 250 m d. 300 m

38. A 200-gram apple is thrown from the edge of a tall building with an initial speed of 20 m/s. What is the change in kenetic energy of the apple if it strikes the ground 50 m/s? a. 130 Joules b. 210 Joules c. 100Joules d. 82 Joules39. A machine costs P 8, 000 and an estimated life of 10 years with a salvage value of P 500. What is its book value after 8 years using straight-line method?

a. P 2,000 b. P 4,000 c. P 3, 000 d. P 2, 50040. The distance between the points AB defined by and is equal to

a. cos A b. 1 c. d. 41. What nominal rate, compounded semi-annually, yields the same amount as 16% compounded quarterly?

a. 16.64% b. 16.16 c. 16.32 d. 16.00%42. If – find x.

a. 10 b. 13 c. 12 d. 1143. The point of intersection of the planes and is at

a. b. c. d. 44. To compute the value of n factorial, in symbolic form (n!); where n is large number, we use a formula called

a. Richardson-Duchman Formula b. Diophantine Formulac. Stirling’s Approximation d. Matheson’s Formula

45. A boat can travel 8 miles per hour in still water. What is it velocity with respect to the shore if it heads East of North?

a. 6,743 b. 8,963 c. 5,400 d. 4,58846. What is the distance in cm between two vertices of a cube which are farthest from each other, if an edge measures 8 cm?

a. 13.86 b. 16.93 c. 12.32 d. 14.33

47. If then the value of x is

a. 1/3 b. 1/2 c. 1/4 d. 1/548. The energy stored in a stretched elastic material such as a spring is

a. mechanical energy b. elastic potential energyc. internal energy d. kinetic energy

49. Given the points (3, 7) and (-4, -7). Solve for the distance between them.a. 15.65 b. 17.65 c. 16.65 d. 14.65

50. What rate of interest compounded annually is the same as the rate of interest of 8% compounded quarterly?

a. 8.48% b. 8.42% c. 8.24% d. 8.86%

ANSWERS 1A 2A 3A 4B 5C 6C 7A 8 A 9 D 10 C 11D 12D 13D 14C 15C 16C 17B 18A 19D 20B

21C 22D 23B 24B 25B 26C 27C 28D 29D 30D 31A 32C 33D 34A 35A 36B 37D 38B

Page 25: Mathematics

39A 40C 41C 42C 43B 44C 45D 46A 47B 48B 49A 50C

Differential Equation - an equation involving differential coefficient or differentials.

Consider the equation :

(d2y / dx ) + P (x) =Q (x )

* When an equation involves one or more derivatives with respect to a particular variable, that variable is called independent variable. For the given equation, the independent variable is x.

* If a derivative of a variable occurs, that variable is a dependent variable. For the given equation; dependent variable is x.

Types of Differential Equation:

1. Ordinary Differential Equation – an equation which all differential coefficient have reference to a single independent variable.

2. Partial Differential Equation – an equation which there are two or more independent variables and partial differential coefficients with respect to any of them.

Order of Differential Equation:- The order of the highest derivative appearing in the equation.

Degree of Differential Equation:- The power to which the highest - order derivative is raised , if the equation is written as a

polynomial in the unknown function .

Linearity of a Differential Equation

- A differential equation is linear if it has the form:

Pn(x) dny + Pn-1 (x) dn-1y +……+ P1(x) dy + Po(x)y = Q(x) dxn dxn-1 dx

where: y – the unknown function

x – the independent variable

Q(x) , Pn(x), Pn-1(x)…P0(x) - presumed known and depend only on the variable x

* Differential equations that cannot be reduced or put in this form are nonlinear.

Sample Problems:

Determine the order, degree, linearity, unknown function, and independent variable of the differential equation.

1. y’’’ – 2xy ‘ = ln x +2

2. 5x (d2y / dx2 ) 9x3 (dy / dx) tanx (y) = 0

3. (d2b / dp2 ) + p (db / dp)2 = 0

Answers:1. third order , first degree, linear (P3(x) =1 , P1(x) = -2x , P2(x ) = P0 (x) =0 , Q(x) =

lnx +2, the unknown function is y, and the independent variable is x.2. second order , first degree, linear, unknown function is y, and the independent

variable is x.3. second order , second degree , nonlinear ( one of the derivatives is raised to a power other than the first) unknown function is b and its derivatives, and the independent variable is p.

Supplementary Problems:Determine a)order b) degree c) linearity d) unknown function and e)independent variable for the following differential equations.

1. (y’’’)3 - 5x(y’)2 = e-x + 1 2. 5y d2z / dy + 3y2 (dz/dy) - (siny) z = 0

3. (d4x / dy4)5 + 7(dx / dy)10 + x7 –x5 = y 4. ty’’ + t2y’ - (sin t ) y = t2 – t + 1

Families of Curves:Families of curves may be represented by equation involving parameters. If the

constants of this equation is treated as an arbitrary constant, the result is called differential equation of the family represented by equation.

Sample Problems :1. Obtain the differential equation of the family of:

a). straight lines with slope and y- intercept equal.

Solution:

y = mx +b - the slope intercept form

In these case, m = b

y = mx+m - the constant to be eliminated

differentiating dy = m

dy/dx = m substituting to the original equation

y = (dy / dx) (x+1) Answer

Page 26: Mathematics

b). circles with center on the x-axis

y

x

Solution:C( h,0) the arbitrary constants are h and r, therefore we must

(x-h)2 + y2 = r2 differentiate the equation two times.2(x-h) + 2yy’ = 0 first differentiation

(x-h) + yy ‘ = 0 x+yy’ = h 1+yy’’+ (y ‘)2 = 0 Answer

Supplementary Problems:For each of the following, obtain the equation of the family of plane curves.

1. Straight lines whose distance is a from the origin. Ans. (xy’ –y ) 2 = a2 ( 1+ (y’)2)

2. Parabolas with foci at the origin and axis Ox.Ans. y( y’’)2 + 2xy’ –y =0

3. Circles tangent to the x-axis.Ans. [ 1+ (y’ )2 ]3 = [ yy ’’ +1 + (y’)2 ]2

4. All tangents to the parabola y2 =2xAns. 2x (y’ )2 –2yy’ + 1 = 0

Solution of Differential Equations

A function y = f(x) is a solution of differential equation if it is identically satisfied when y and its derivatives are replaced throughout by f(x) and its corresponding derivatives.

A differential equation of order n will, in general, possesses a solution involving n arbitrary constants. This solution is called general solution. It will be necessary to assign specific values to these arbitrary constants in order to meet the prescribed initial conditions.

First Order Equations

1. First Order : Variables Separable

A first order differential equation can be solved by integration if it is possible to collect all y terms with dy and all x terms with dx. That is, if it is possible to write it in the form:

f(y) dy + g(x) dx = 0 ,Then the general solution is

f(y) dy + g(x) dx = Cwhere C is an arbitrary constant.

Sample Problems:1. Find the general solution of the following differential equation

a. y’ =(x+1) / y b. ds/dt = s2 + 2s + 2

2. Obtain a particular solution which satisfies the given initial condition.dy/dx = 3x3 / y , y = 4 when x = 1

Solution:1. a). The equation maybe written in the form

dy/dx = (x+1)/y

ydy = (x+1) dx

The solution is

ydy = (x+1) dx

½ y2 = ½ x2 + x + C

y2 – x2 – 2x = C Ans.

b. Separate the variables to obtainds

= dt s2 + 2s + 2

which is the same asds

= dt (s + 1)2 + 1

The solution isds

= dt (s + 1)2 + 1 C + t = arctan (s+1)

s = tan (C+t) – 1 Answer

2. The variables are separable, which can be written in the form;

y dy = 3x3 dx

integrating both sides

Page 27: Mathematics

y dy = 3x3 dx

2/3 y3/2 = ¾ x4 + C this is the general solution

if y = 4, x = 1

C = 2/3 (4)3/2 – ¾ (1)4

C = 55/12

2/3 y3/2 = ¾ x4 + 55/12

or simply

8 y3/2 – 9 x4 = 55 Answer

Supplementary Problems:1. Obtain the general solution of the following:

a. xy’ + y = 0 Ans. xy + C = 0

b. x dx = y dy = 0 Ans. x2 + y2 = C

c. x3 dx + (y + 1)2 dy = 0 Ans. (y + 1)3 + ¾ x3 = C

d. ds/dt = t2 / s2 + 6s + 9 Ans. (s + 3)3 – t3 = C

e. y dy + (y2 + 1) dx Ans. ln (1 + y2) + 2x

2. Obtain the particular solution satisfying the given initial conditions.a. dy/dx + 2y = 3 x=0, y=1 Ans. ln (3 – 2y) + 3x = 0

b. dy/dx = sec y tan x x=0, y=0 Ans. sin y + ln cos x = 0

2. First Order : Homogeneous EquationPolynomials in which all terms are of the same degree such as x2 + y2 and x2 sin (y/x)

are called homogeneous polynomial.

Equations with homogeneous coefficient.Consider

M (x,y) dx + N (x,y) dy = 0 1If the coefficients M and N are of the same degree in x,y , they are called homogeneous

functions.

Let y = Vx, and dy = Vdx + xdV, substitute this to equation 1 to make the variables separable.

Sample Problem:1. Solve y’ = (y + x )/ x

Solution: Let y = Vx; dy = Vdx + xdV

dy/dx = (y + x)/ x = (Vdx + xdV ) / dx = (Vx + x) / x

or simply

x (dV/dx) = 1

V = ln x + C

V = ln C x *

But v = y/x

Therefore, y = x ln C x

* The ln of a constant is still a constant.

Supplementary Problems:Find the general solution:

1. y’ = 3xy / y2 – x2 Ans. (y2 + 2x2)3 = Cy2

2. y’ = (x2 + y2 ) / 4xy Ans. 3y2 - x2 = C x

3. y’ = y / ( x + xy) Ans. – 2 x/y + ln y = C

3. First Order : Linear EquationA differential equation of first order, which is also linear, can be written in the form:

dy/dx + P(x)y =Q (x) 1

To obtain the general solution we must find a function = (x) such that if the equation is multiplied by this function derivative of the product y. This function is called the integrating factor .

= e Pdx

And the solution is

y ePdx = Q ePdx dx + C 2

Sample Problem: 1. Obtain the general solution of the differential equation.

x dy + y dx = sin x dxSolution:

The differential equation must be first reduced in the form of equation 1, hence,

dy/dx + y/x = (sin x) / x

P(x) = 1/x ; Q(x) = (sin x) / x

Solving for the integrating factor,

= edx/x

= eln x

= x

Page 28: Mathematics

using the formula,

yx = sinx /x (x) dx + C

simplifying,

xy + cos x = C Answer

Supplementary Problems:Obtain the general solution:

1. dy/dx + x3y = 0 Ans. y = c e (-x^4)/4

2. e-2ydx + 2 (xe2y – y) dy Ans. x e2y = y2 + C

3. dT/d = cos + y cot Ans. T = c sin - cos

4. y’ = x – 2y cot 2x Ans. 4y sin 2x = c + sin 2x – 2x cos 2x

5. y’ = x3 2xy; when x = 1, y = 1 Ans. 2y = x2 – 1 + 2 e(1-x²)

3. Bernoulli’s EquationStandard Form:dx/dy + y P(x) = yn Q(x)provided, n 0, 1Integrating factor = ePr (x) dx

Where: P(r)(x) = (1-n) P(x) Q(r)(x) = (1-n) Q(x)

z = y1-n

General Solution: z = Qr (x)dx + c

Sample Problem:Solve dy / dx – 2y / x = 4x3 y3

Solution:dy / dx + y (-2/x) = y3 (4x3)

n = 3

z = y1-3 z = y –2

P(x) = -2 / x

Pr (x) = (1-n) Px

Pr (x) = (1-3) Px

Pr (x) = -2 (-2/x) = 4/x

Q (x) = 4x3

Qr (x) = (1-n) Q(x)

Qr (x) = (1-3) (4x3)

Qr (x) = -8x3

= e Pr (x) dx = e (4/x) dx + e 4 lnx = x4

General solution:z = Qr (x)dx + c

(y-2)(x4) = x4 (-8x3) dx + c

y-2x4 = -8 x7 dx + c

y-2x4 = - x8 + c

x4 = (c –x8) y2

4. First Order: Exact equationsAny equation that can be written in the form

M(x,y)dx + N(x,y)dy = 0

And we have the property

M N ------- = ------- is said to be an exact equation.y x

The technique is to find a function f(x,y) such that

f’(y) – the common term of f(x,y) and N(x,y) g’(y) – the common term of f(x,y) and M(x,y)

Sample Problem:Determine whether the equation 2xy dx + (1+x2) dy = 0 is exact. If so, obtain the general

solution.Solution:

Given in the equation that

M = 2xy, N = 1+ x2

M / y = N / x = 2x The equation is exact

F = 2xy dx + f(y)

F = x2y + f(y)

F/ y = x2 + f’(y) = x2 + 1

f’(y) = 1

f(y) = y + c

x2y + y + c = 0 Answer

Reduced polynomial in x

Page 29: Mathematics

Supplementary Problems:1. (2x ey + ex) dx + (x2 +1) ey dy = 0 Ans. ex + ey(x2 +1) = c

2. (2xy + y2) dx + (x2 +2xy - y) dy = 0 Ans. 2x2y +2xy2 – y2 = c

3. (x + sin y) dx + (x cos y – 2y) dy = 0 Ans. ½ x2 + x sin y – y2 = c

4. dy + (y- sin x) / x dx = 0 Ans. xy + cos x = c

Elementary Applications:1. Law of Exponential Change (Growth /Decay)

The rate of which the amount of a substance changes, is proportional to the amount present or remaining at any instant.

Q = amount of substance at any time t

dQ / dt = rate of change of the amount Q

dQ / dt Q

dQ / dt = kQ

Q = Qo ekt k > 0 – exponential growth

k < 0 – exponential decay

Sample Problem:1. A certain radioactive material is known to decay at a rate proportional to the amount present. If initially there is 50 mg of the material present and after 2 hrs. it is observed that the material has lost 10% of its original mass. Determine mass of the material after 4 hours?

Solution:

Given: Qo = 50 mg

Q(2) = (50 mg) (1-0.1)

Q(2) = 45 mg

Solving for k,

45 = 50 e2k

k = -0.053

amount of material at any time t

Q = 50 e -0.053 t

In the problem t = 4 hrs.

Q = 50 e -0.053 t

Q = 40.5 mg. Answer

2. Newton’s Law of CoolingThe time rate of change of the temperature of a body is proportional to the temperature difference between body and its surrounding medium.

T = temperature of the body at any time t

To = initial temperature

Tm = temperature of the surrounding medium

dT / dt = time rate of change of the temp. of the body

dT / dt = - k (T – Tm)

(dT / dt) + kT = kTm , k is always positive, dT / dt = negative for cooling

T = Ce-kt + Tm or T = (To – Tm) e-kt + Tm

Sample Problem:A body at a temperature off 36oC is placed outdoors where the temperature is 60oC. If

after 5 mm, the temperature of the body is 40oC. How long will it take to reach a temperature of 45oC?

Solution:Given: Tm = 60oC

To = 36oC

T @ 5 mm = 40oC

Solve for k:

40 = (36-60) e –k(5) + 60

k = 0.03646

Solving for t at T = 45oC

45oC = (36-60) e –0.03646 t + 60

t = 12.9 min. Answer

3. Simple Chemical ConversionIn certain reactions in which a substance A is being converted into another substance the time of change of the amount x of unconverted substance is proportional to x.

Let x = xo the uncovered substance at to = 0, then the amount x at any time t > 0, is given by the differential equation as dx/dt = - kx

The proportionality is chosen to be –k because x is decreasing as time increases,from x = ce-kt but x = xo at t = 0, hence

xo= ce-k(1) c = xo

X = xoe-kt

Sample Problem:

Page 30: Mathematics

Suppose that a chemical reaction proceed such that the time rate of change of the unconverted substance is proportional to the amount of it. If half of substance A has been converted at the end of 10 sec. Find when 9/10 of the substance will have been converted.

Solution:Given: x = ½ xo

½ xo = xo e k(-10)

k = 0.069

when 9/10 is converted only 1/10 is unconverted

x = 1/10 x1/10 xo = xo e –0.069 t

solving for t

t = 33 sec. Answer

4. Dilution / Flow Problem:The tank initially hold Vo (gal., liter,etc.) of solution that contains So (lb.,kg.,N) of a substance. Another solution containing a substance at CI = lb/gal.; N / L is poured into the tank at Ri = (gal/min. , liter/min.) while simultaneously, the well-stirred solution leaves the tank at the rate Ro ( gal/min.)

In general, dS/dt = So / [Vo + (Ri , Ro) t] = Ci Ri Ri > Ro , then Vo will overflowRi = Ro , Vo = cRo < RI , then Vo

Let S = amount of substance present in the tankdS/dt = rate of change of substance S

Sample Problem:A 50 gal. tank contains 10 gal. of fresh water. At t = 0, a brine solution containing 1 lb.

Of salt per gallon is poured into the tank at the rate of 4 gal/min. , while the well-stirred mixture leaves the tank at the rate of 2 gal/min,a.) find the amount of time required for overflow to occur, b.) Find the amount of salt in the tank at the amount of overflow.

Solution:At t = 0, Vo = 0 , ci = 1, Ri = 4,Ro = 2

The volume of brine in the tank at any time t equals to

Vo + Ri t - Ro t = 10 + 4t – 2t

= 10+2toverflow will occur if the volume of the brine in the tank equals to the volume of tank now.

T = ½ (50-10) = 20 min.

b.) dQ/dt + Q/10+2t = 4

dQ/dt + 2Q/10+2t = 4

integrating factor = e (dt / 5+t) = e ln (5+t) = 5+t

Q (5+t) = 4 (5+t) dt

Q (5+t) = 4 [ 5t +1/2 t2] + c

Q = 20t +12 t2 + c / 5+t

At t = 0 Q = a = 0

0 = [20(0) + 2(0)2 + c] / (5+t)

c = 0; now

Q = 20t + 2 t2 / 5+t

Req’d: Q =? At t = 20 min.

Q = [20(20) + 2(20)2 / (5+20)

Q = 48 lb. Answer

5. Orthogonal TrajectoriesGiven a family of curves given by f(x,y,c) = 0The curves that intersects a family f(x,y,c) = 0 at right angles, whenever they do intersect is given by the family of another curve g(x,y,k) and are called orthogonal trajectories. This two curves are said to be orthogonal to each other, because each point of intersection, the slopes of the curves are negative reciprocals to each other.

Let Mdx + Ndy = 0 where M and n are f(x,y)

dy/dx = - M/N

Now the DE of the orthogonal trajectories is dy/dx = N/M

Sample Problem:Find the orthogonal trajectories of all circles whose center is at the origin.

Solution:

x2 + y2 = c2

Differentiating implicitly, 2xdx + 2ydy = 0

Mixture Vo ,So

Ci , Ri

OutgoingCo , Ro

ds/dt = Ci Ri - Co Ro

Co = S / [Vo + (Ri , Ro) t]

Page 31: Mathematics

Simplifying

dy/dx = - x/y = - M/N

N/M = dy/dx the D. E. of trajectories

y/dy = x/dx then integrating

y = kx Answer

Supplementary Problems:1. A body at an unknown temperature is placed in a room which is held at a constant

temperature of 30oC and after 10 min., the temperature of the body is 0oC and after 20 min., the temperature of the body is 15oC. Find the expression for the temperature at any time t. Ans. T = - 60 e –0.069t + 30

2. At exactly 10:30 pm, a body at a temperature of 50oF is placed In an oven whose temperature is kept at 150oF, If at 10:40, the temperature of the body is 75oF, at what time will it reach 100oF. Ans.10:54 p.m.

3. Find the required for a radioactive substance to disintegrate half of its original mass if three quarters of it are present after 8 hrs. Ans. 19.3 hrs.

4. After 2 days, log of a radioactive material is present. Three days later, 5g. is present. How much of the chemical was present initially, assuming the rate of disintegration is proportional to the amount present. Ans. 15.87 g.

5. If a population of a country doubles in 50 years, in how many years will it treble under the assumption that the rate of increase is proportional to the no. of inhabitants? Ans. 79

6. A cylindrical tank contains 40 gal. of a salt solution containing 2 lb. of salt per gallon. A salt solution of concentration 3 lb/gal flows into the tank at 4 gal/min. How much salt is in the tank at any time t if the well-stirred solution flows at 4 gal/min? Ans. 120 - 40e –t/10

7. An inductance of 1 henry and a resistance of 2 ohms are connected in series with an emf of 100e-t volts. If the current is initially zero, what is the maximum current attained? Ans. 25A

8. Find the orthogonal trajectories of x2=cy. Ans. 2y2+x2=c

DIFFERENTIAL CALCULUS

LIMIT OF FUNCTIONS

Definitions The limit of a function of x or f(x) as x approaches a is L; written as

lim f(x) = Lxa

lim f(x) = L if and only if, for any chosen positive number ,xa however small, there exists a positive number such that, whenever 0 <

x-a < , then (x) – L <

Theorems on Limits

1. Limit of a constant c

lim c = cxa

2. Limit of a variable xlim x = axa

3. Limit of sum of two functionslim [ (x) + g(x) ] = lim (x) + lim g(x) xa xa xa

4. Limit of product of two functionslim [ (x) g(x) ] = lim (x) [ lim g(x) ]xa xa xa

5. Limit of a quotientlim (x) = lim (x)xa g(x) xa

lim g(x) xa

6. Limit of a radicallim n (x) = n lim (x)

xa xa

Limit to Infinity or ZeroGiven a constant c and variable x1. lim cx = + for positive c

x = - for negative c2. lim c/x = 0

x

3. lim x/c = + for positive cx = - for negative c

4. 4. lim c/x = + for positive cx0 = - for negative c

L’Hospitals Rule

If the functions f(x) and g(x) are continuous in an interval containing x = a, and if their derivatives exist and g’(x) 0 in this interval (except possibly at x = a), then when f(a) = 0 and g(a) = 0

lim f(x) = lim f ’(x) xa g(x) xa g ’(x)Provided that the limit on the right side exists.

Evaluation of LimitsLet lim N(x) = L

xa D(x)where N (x) and D (x) are polynomials in x

a = any real number L = limit

Page 32: Mathematics

Methods of Evaluation

1. direct substitution ( obvious limit )2. rationalizing N (x) or D (x)3. expanding N(x) or D (x)4. combining terms in N (x) or D (x)5. factoring N (x) or D (x)6. applying L’Hospital’s Rule

Limits to Infinity of a Fraction N(x) D(x)

Let lim N(x) = L x D(x)

1. If the degree of the numerator N (x) is less than the degree of denominator D (x), then L = 0.

2. If the degree of N (x) equals the degree of D (x) thenL = coefficient of N (x) with highest degree coefficient of D (x) with highest degree

3. if the degree of N (x) > D (x), then L =

INTERMEDIATE FORMS

A. 0/0 or / ( L’Hospital’s Rule is applicable)B. - , 0. ( L’Hospital’s Rule is not directly applicable)C. 00, 0, 1

Note:If the evaluated function turned out to be in the form of those in B and C, change the

form of the given function to obtain an evaluated function in the form of that in A.

The Indeterminate Form 0. If f(x) approaches zero and g(x) approaches infinity as x approaches a (or x

) , the product (a). g(a) is undefined and will be of the form 0. .If the limit f(x) . g(x) exists as x a (or as x ), it may be found by writing the

product as a fractionlim (x) g(x) = lim (x) = lim g(x)xa xa 1/g(x)

xa 1/f(x)

Then apply L’Hospital’s Rule The Indeterminate Form -

If (x) and g(x) both increase without bound as x a (or x ), the difference (a) – g(a) is undefined and will be of the form - If the limit of (x) – g(x) exists as x a (or x ), then by algebraic means.

lim [ (x) – g(x)] = lim 1 /g(x) - 1 /f(x) __xa xa 1___

g(x) f(x)

The form 00, 0, 1

If the limit of (x)g(x) exists as x a (or as x ), then by logarithm, lim (x)g(x) =y can be evaluated

xa

Let L = lim (x)g(x) xa

Taking the logarithm of both sidesln L = ln lim (x)g(x) = lim (x)g(x) lim g(x) ln (x) = k

xa xa xa

In L = k , then elnL = ek or L = ek

Supplementary Problems

Evaluate the limits given below1. lim sin 5x x0 x Ans: 52. lim 1 – cos 2 Ans: 03. lim (1 + 1/x)3x

x Ans: e3

4. lim 1 /3 – 1 / x

x3 x – 3 Ans: 1/9

5. lim sin 0 Ans: 1

6. lim (1/x – 1/3sinx) x0 Ans: 07. 7. lim (x+1) ln x

x0 Ans: 18. 8. lim x csc 5x x0 Ans: 1/5

9. ECE Board exam 1987Evaluate lim x 3 – 2x + 9 x 2x3 – 8 Ans: ½

10. ECE Board Exam 1987Evaluate lim 2x 4 – 2x 3 + 9x 2 – x + 7 x x3 – 8 Ans:

Interpretation of Derivative1. Derivative as Slope

y y=(x)

Tangent Line

Po(xo,yo)

Page 33: Mathematics

0 x

Definitions

The tangent to the curve with equation y = f(x) at Po(xo, yo) is the line through Po(xo, yo) with slope f’(xo)Slope of Tangent Line = tan = dy /dx = f’(xo)

The normal to the curve with equation y = f(x) at Po(xo, yo) is the line through Po which is perpendicular to the tangent line at Po

Slope of Normal Line = 1 Slope of tangent line

= - 1 dy/dx= - 1 f’(xo)

ANGLE BETWEEN TWO CURVES (Angle of Intersection) – is defined as the angle between their tangents at their point of intersection.To determine the angle of intersection of two curves, f(x) and g(x)1. Solve the equations simultaneously to find the points of intersection.2. Find the slopes m1 = f’ (xo) and m2 = g’ (xo)

Then the acute angle of intersection is given by

m1 – m2

Tan = --------------- 1 + m1m2

2. Derivative as Rate of Change dy /dx = lim y / x

x0 The value of the derivative of the function is the instantaneous rate of change of the function with respect to the independent variable.

Rectilinear Motion – motion in a straight line Assumption:

1. Motion will always be assumed to take place along straight line, although the object in motion may go either direction.

2. The body in motion is idealized to be a point or a particle.

Let S = displacement of a particle at any time t

Speed (v) – of a particle moving along a curve is the absolute value of the time rate of change of the displacement (or distance), measured along the curve, of the point with reference to some fixed point on the curve.

V = ds/ dt Acceleration (a) – of a moving point is the time rate of change of the velocity of

the point a = dv/ dt = d2s/dt2

Differentiation FormulasLet u, v be any functions of x n = any integer C= any constant

I. Basic Formulas

1.

2.

3.

4.

5.

6.

7.

8.

9.

10.

11.

Page 34: Mathematics

12. (The Chain Rule)

13.

II. Differentiation of Trigonometric Function

1.

2.

3.

4.

5.

6.

III. Differentiation of Inverse Trigonometric Function

1.

2.

3.

4.

5.

6.

IV. Differentiation of Logarithmic Functions

1.

2.

3.

V. Differentiation of Exponential Functions

1.

2.

3.

VI. Differentiation of Hyperbolic Function

1.

2.

3.

4.

5.

6.

VII. Differentiation of Inverse Hyperbolic Function

1.

2.

3.

4.

Page 35: Mathematics

5.

6.

Supplementary Problems:1. Find the slope of the curve y = x + 2x-2 at point (1, 2).Ans: -32. Find the point on the curve y = x2 – 6x + 3 where the tangent is horizontal. Ans: P(3, -6)3. At what point on the curve y = x4 + 1 is the normal line parallel to

2x + y = 5?Ans: P(1/2, 17/16)4. Find the point on the curve y = 7x – 3x2 + 2 where the inclination of the tangent is

450.Ans: P(1, 6)5. Find the point where the normal to y = x + x1/2 + 1 at (4, 7) crosses the

y- axis. Ans: P(0,10 1/5)6. The tangent to y = x3 – 6x2 + 8x at (3, -3) intersects the curve at another point. Determine

this point. Ans: P(0, 0)7. For the curve y = x2 + x, at what point does the normal line at (0, 0) intersect the tangent

line at (1, 2)? Ans: P(3/10, -1/10)8. Find the angle of intersection between the curves y = x2 + 2 and

y = x + x-1 + 1. Ans: = 63.409. A boy 3 ft. tall walks away from alight which is on top of post 7.5 ft. high. Find the rate of

change of the length of his shadow with respect to his distance from the lamp post. Ans: 2/3 ft/ft

10. Find the rate of change the area of an equilateral triangle with respect to the side of this triangle when the latter is 2 ft. Ans: 3 ft2/ft.

11. If the particle moves according to the law S = t2 – t3 + 3, find the velocity when acceleration is zero.Ans: -1/3

12. If the motion of the body is described by S = 3t5 – 30t2 + 5, when will the acceleration be zero? Ans: 1

13. Two particles have position at time t given by the equation S1=t3 + 6t2 – 7t + 1 and S2 = 2t3

– 3t2 – t + Find their position when they have the same acceleration. Ans: S1 = 61; S2 = 25

14. Find y’ and y”, given x3y + xy3 = 2 and x = 1. Ans: y’ = -1;y” = 015. Find the equation of the tangent and normal to x2 + 3xy + y2 = 5 at (1, 1).Ans: Tangent: x + y – 2 = 0, Normal: x = y

Other Applications of DerivativesIncreasing and Decreasing Functions:

A function y = f(x) is said to be increasing if its value increases as y increases A function y = f(x) is said to be decreasing if y decreases as x increases

Given a function f(x) differentiable in the interval a x b1. If f’(x) > 0, then f(x) is increasing2. If f’(x) < 0, then f(x) is decreasing3. If f’(x) = 0, then f(x) is stationary

Concavity, Critical Points, Inflection Points Concave Upward

The graph of a function is said to be concave upward if the function is decreasing then increasing.

Concave DownwardThe graph of a function is concave downward if the function is increasing then decreasing.

Maximum PointA point where the function from increasing to decreasing and the function is said to have a relative minimum value.

Critical PointThe point at which y’ = 0 and value of x at this point critical value.

Inflection PointA point at which the curve changes its direction of concavity.

First Derivative TestSubstitute in the expression for the first derivative a value slightly less than and then a value slightly greater than the critical value under consideration.

1. If f’(x) changes from positive to negative as x increases through the critical value, then the critical is a maximum point.

2. If f’(x) changes from negative to positive as x increases through the critical value, then the critical is a minimum point.

3. If f’ (x) does not changes sign, the critical is neither a maximum or a minimum point. It is the point of inflection with horizontal tangent.

maximum point y’= 0

Inflection y = f(x) Point

y’=0 minimum point

x=a

concave concavedownward upward

Second Derivative Test1. The function y = f(x) has a maximum value at x = a if f’(a) = 0 and f”(a) < 0. The

curve is concave downward at that point.

Page 36: Mathematics

2. The function y = f(x) has a minimum value at x = a if f’(a) = 0 and f”(a) > 0. The curve is concave upward at that point.

3. If at f’(a) = 0, f”(a), then his test fails. Use the first derivative test.

Third Derivative TestA function y = f(x) has an inflection point at x = a if f”(a) = 0 and f”’(a) 0.

Supplementary Problems1. Find the maximum, minimum and inflection point of the curve

y=-x3 + 2x2 – x +2 Ans:Max.(1, 2); Min. (1/3, 50/27); Inflection (2/3, 52/27)2. Find the value of k such that the curve y = x3 – 3kx2 + 5x – 10 Ans: k = 13. Determine the equation of the parabola y = ax2 + bx + c passing through (2, 1) and be

tangent to the line y = 2x + 4 at point (1, 6) Ans: y = -7x2 + 16x –34. What curve of the form y = ax3 + bx2 + cx + d will have critical points at (0, 4) and (2, 0).

Ans: y = x3 – 3x2 + 45. Determine a, b, c and d so that the curve y = ax3 + bx2 + cx + d will have horizontal

tangents at the points (1, 2) and (2, 3) 6. Find the equation of the line normal to the curve y = 3x5 – 10x3 + 15x + 3 at its point of

inflection. Ans: x + 15y – 45 = 07. Find a such that the curve y = 2x3 – 3ax2 + 12x – 1 will have are of its critical points where

x = 2 Ans: a = 38. Find the all values of x where the curve y = x2 – 2x + 5 is increasing. Ans: x > 1

MAXIMA AND MINIMA APPLICATIONSStep in Solving Problems Involving Maxima and Minima1. Identify the quantity to be maximized or minimized2. Use the information in the problem to eliminate all quantities so as to have a function

variables. Determine the possible domain of this function.3. Differentiate this function with respect to the variable whose maximum/minimum value is

to be determined4. Equate the derivative of step 4 to zero and solve for the unknown.

Supplementary Problem1. ECE Board Exam 1973

An open rectangular tank with square base is to have a volume of 10 cu. m and the material for the bottom is to cost per square meter and that for the sides 6 cents per square meter. Find the height of the tank if the coil of making the tank is to be a minimum. Ans: 2.5 cm.

2. ECE Board Exam Feb. 1973In problem no. 1 above, find the most economical dimension for the tank. Ans: 2m x 2m x 2.5m

3. ECE Board Exam 1981Divide 60 into two parts so that the product of one part and the square of the other is a maximum. Ans: 20, 40

4. ECE Board Exam April 1988

Find the altitude of the largest circular cylinder that can be inscribed in a circular cone of radius r and height h. Ans: 1/3 h

5. ECE Board Exam 1989Find the greatest volume of a right circular cylinder that can be inscribe in a sphere of radius r. Ans: r = 2.418 r3

6. ECE Board Exam Nov. 1995Find the radius of a right circular cylinder that can be inscribe in a cone of a radius R and height H. Ans: r = 2/3 R

7. ECE Board Exam Feb. 1978The sum of the two numbers is 36. What are these numbers if their product is to be the largest possible. Ans: 18 and 18

8. ECE Board Exam Feb. 1978A square sheet of galvanized iron 100cm x 100cm will be used in making an open top

container by cutting a small square from each corners and bending up the sides. Determine how large the square should be cut from each corner in order to obtain the largest possible volume. Ans: 16.67cm x 16.67

9. ECE Board Exam, Mar. 1989A rectangular field containing a given area is to be fenced off along a straight river. If

no fencing is needed along the river, what should be the dimension of the field so that least amount of fencing materials will be used? Ans: L = 2W.

10. ECE Board Exam, Nov. 1989Find the minimum volume of a right circular cylinder that can be inscribe in a sphere having a radius equal to r.

11. EE Board Exam , April 1981A telephone company agrees to put up a new exchange for 100 subscribers or less

at a uniform change of P40 each. To encourage more subscriber the company agrees to deduct 20 centavos from their uniform rate for each subscriber in excess of 100. If the cost to serve each subscriber is P 14, what number of subscriber would give the telephone company the maximum net income. Ans: 115

12. Find the equation of the tangent line to the curvey = x3 – 3x2 + 5x = 2 that has the least slope Ans: 2x – y + 3 = 0

13. Find the area of the largest rectangle with sides parallel to the coordinate axes which can be inscribe in the bounded by x2 = 28 – 4 and x2 = y – 4.

Ans: 64 sq. units14. An isosceles trapezoid is 6 cm long on each side. How long must be the longest side if

the area is maximum. Ans: 12 cm.15. Find the dimension of the right circular cone of minimum volume which can be

circumscribed about a sphere of radius 8 cm. Ans: radius = 82 cm ; height = 32 cm.16. Find the dimension of the cylinder of maximum lateral area which can be inscribe in a

sphere of radius 62 cm. Ans: radius = 6 cm. ; height = 12 cm.17. Find the ratio of the volume of the right circular cylinder of maximum volume to that of the

circumscribing cone. Ans: Vcyl/Vcone = 4/918. Find the equation of the line passing through the point (3, 4) which cuts from the first

quadrant of a triangle of minimum area. Ans: 4x + 3y – 24 = 0

Page 37: Mathematics

19. Find the dimension of the right circular cone of maximum volume which can be inscribe in a sphere of radius 12 cm. Ans radius=82 cm ; height = 16cm.

20. Find the area of the largest rectangle that can be inscribe in an ellipse 9x2 + 4y2 = 36. Ans: 12 square units.

RELATED RATESIf a quantity x is a function of time t, the time rate of x given expressed as dx/dt.When two or more time varying quantities are related by an equation, the relation between their rates of change may be obtained by differentiating both members of the equation with respect to time t.

Supplementary Problems:1. ECE Board Exam, Sept. 1986

A baseball diamond is a square, 27 m on each side. The instant a runner is halfway from home to first base, he is giving towards first base at 9 m/s. How fast is his distance from the second base changing at this instant? Ans: -4.025 m/s

2. ECE Board Exam , Sept. 1983A boat is being towed to a pier. The pier is 20 ft. above the boat. The remaining length of the rope to be pulled is 25 ft. It is being puled at 6 ft. per second. How fast does the boat approaches the pier? Ans: 10 ft/s

3. ECE Board Exam, March 1981Given a conical funnel of radius 5 cm and height 15 cm. The volume is decreasing at the rate of 15 cu. cm/s. Find the rate of change in height when the water is 5 cm from the top. Ans: 0.43 cm/s

4. ECE Board Exam, April 1998 / ECE Board Exam, Oct. 1985Sand is pouring from a hole at the rate of 25 cu, ft. per second and is forming a conical pile on the ground. If the conical formation has an altitude always ¼ of the diameter of the base, how fast is the altitude increasing when the conical pile is 5 ft. high? Ans: 1/12.75 ft/s increasing

5. ECE Board Exam, Feb. 1978A helicopter is rising vertically from the ground at a constant rate of 4.5 m/s. When it is 75 m off the ground, jeep passed beneath the helicopter travelling in a straight line at a constant speed of 80 kph. Determine how fast the distance between them changing after 1 second? Ans: 10.32 m/s (increasing)

6. ECE Board Exam, Feb. 1977Two boats starts at the same point. One sail due east starting 10 A.M. at a constant rate of 20 kph. The other sail due south starting 11 A.M. at a constant rate of 9 kph. How fast are they separating at noon?Ans: 21.49 kph.

7. ECE Board Exam, August 1976A dive bomber loss altitude at a rate of 400 mph. How fast is the visible surface of the earth decreasing when the bomber is 1 mile high? Ans: 2792

8. A man lifts a bag of sand to a scaffold 30 m above his head by means of a rope which passes over a pulley on the scaffold. The rope is 60 m long. If he keeps his end of the

rope horizontal and walks away from beneath the pulley at 4 m/s, how fast is the bag rising when he id 22.5 m away?Ans: 2.4 m/s

9. Water is passing through a conical filter 24 cm deep and 16 cm across the top into a cylindrical container of radius 6 cm. At what rate is the level of water in the cylinder rising if when the depth of the water in the filter is 12 cm its level is falling at the rate of 1 cm/min? Ans: 4/9 cm/min

10. A particle starts at the origin and travel up the line y = 3 x at a rate of 5 cm/sec. Two seconds later, another particle starts at the origin and travels up the line y = x at the rate of 10 cm/s. At what rate are they separating 2 seconds after the last particle started? Ans: 0.37 ft/s

11. A particle travels along a parabola y = 5x2 + x + 3. At what point do its abscissa and ordinate change at the same rate? Ans: P(0, 3)

12. At a certain instant the semi major axis and semi minor axis of an ellipse are 12 and 8 respectively and the semi major axis is increasing ½ unit each minute. At what rate is the semi major axis decreasing if the area remains constant? Ans: 1/3 unit/min.

13. A clock hands are 1 and 8/5 inches long respectively. At what rate are the ends approaching each other when the time is 2’o clock? Ans:0.095 in/min.

14. An elevated train on a track 30 ft above the ground crosses a street at the rate of 20 ft. per sec. At the instant that the car approaching at the rate of 30 ft/s and the car are separating 1 sec later/ Ans: 2.67 ft/sec

Differential ApproximationApproximation of Error

If y = f(x), then dy = f’(x) dxdx = change or error in xdy = change or error in ydx/x = relative error in xdy/y = relative error in ydx/x (100) = percentage error in xdy/y (100) = percentage error in y

Supplementary Problems:1. What is the maximum allowable error in the edge of a cube to be used to contain 10

cubic meters if the error in the volume is not to exceed 0.015 cubic meter? Ans: 0.00108

2. The semi major axis and semi minor axis of an elliptical plate is measure to be 8 cm and 6 cm respectively. If there is an approximate error of 0.01 cm and .02 cm in measuring error in computing for the area. Ans: 0.628

3. The altitude of a certain circular cone is the same as the radius if the base and is measured as 12 cm with a possible error f 0.04 cm. Find approximately the percentage error in the calculated value of the volume. Ans: 1%

4. Find the allowable percentage error in the radius of a circle if the area is to be correct to within 5%. Ans: 2.5%

Page 38: Mathematics

5. What is the percentage error made in the computed surface area of a sphere if the error made in measuring the radius is 3%.Ans: 6%.

CURVATURE

y tangent

y = (x)

P(x,y) R C(h,k), Center of Curvature

NormalCircle of

Curvature 0 x

Definition:- The curvature K of a curve y = f(x), at any point P(x, y) on it , is the rate of

change indirection (that is, the angle of inclination - of the tangent line per unit arc length S.

K = d/ds = lim /S s0

y” - x”K = -------------------- or K = ----------------------

[1 + (y’)2]3/2 [1 + (x’)2]3/2

Notes: If K 0, the point P is on the arc that is concave upward If K 0, the point P is on the arc hat is concave downward

The curvature K is given by:

g’ h” - g” h’K = ------------------------ [ (g’)2 + (h)2]3/2

In Polar form, r = f()Where r’ = dr/d , r” = d2r / d2

r2 + 2(r’)2 – rr” K = -------------------------

[r2 + (r’)2]3/2

Radius of Curvature- the radius curvature R for a point P on the curve is the reciprocal of its curvature

at that point

R = 1/KNotes:

If R 0, the curve is concave upward If R , the curve is concave downward

Circle of Curvature or Oscillating Circle- The circle of curvature of a curve at a point P on it is the circle of radius R lying

on the concave side of the curve and tangent to it at P

y

C y = (x)

R

P

x

Center of CurvatureThe center of curvature for a point P(x, y) of a curve y = f(x) is the center C(h, K) of the

circle of curvature at P

y” [1 + (y’)2] 1 + (y’)2

h = x - ----------------- k = y - -------------- y” y”

EVOLUTE- The evolute of a curve is the locus of the center of curvature of the given curve.

Supplementary Problems:1. Find the curvature of ech of the following curves

a) y = sin x Ans: -1

Page 39: Mathematics

b) x2 = 12y at x = 6 Ans: -12. Find the point of maximum curvature of the curve y = ln x Ans: (1/2 2, -1/2 ln 2)3. Find the radius of curvature of the curve of x3 + xy2 – 6y2 = 0 at point (3, 3) Ans: 554. Find the equation of the circle of curvature of the parabola y2 = 12x at point (3, 6) Ans: (x

– 15)2 + (y + 6)2 = 2885. Find the center of the curvature of x3 + xy2 – 6y2 = 0 at (3, 3) Ans: C(-7, 8)

MATHEMATICS MODULE 4

1. The hypotenuse of a right triangle is 34 cm. Find the lengths of the two legs if one leg is 14 cm longer than the other.

A. 15 and 29 cm.B. 16 and 30 cm.C. 17 and 31 cm. D. 18 and 32 cm.

2. The area of a rhombus is 132 sq. m. if its shorter diagonal is 12 m, find the longer diagonal.

A. 20 mB. 22 mC. 36 mD. 28 m

3. One side of the parallelogram is 10 m and its diagonals are 16 m and 24 m respectively, find its area.

A. 158.7 sq. mB. 120 sq. mC. 96 sq. mD. 192 sq. m

4. The diameter of two spheres is in the ratio two is to three and the sum of their volumes is 1260 cu. m. Find the volume of the larger sphere in cu. m.

A. 980B. 972C. 960D. 938

5. The side of the triangle are 5, 7 and 10 respectively. Find the radius of the circumscribed circle.

A. 5.39 mB. 6.40C. 7.20 D. 4.80

6. The volume of a sphere is 36π cu. m. The surface area of this sphere in sq. m is:A. 25πB. 42πC.36π

D. 54π

7. How many side does a polygon has if the sum of the interior angles is 2520o?A. 14B. 16C. 12D. 10

8. The first term of an arithmetic progression is 3 and the 15th term is 45. Find the sum of the first 15 terms.

A. 260B. 360C. 460D. 560

9. The first term of an arithmetic progression is -2 and the sum of the first 11 terms is 88. The common difference is:

A. 4B .3C. 2D. 1

10. An ellipse with major axis 8 and a minor axis 6 is revolved about its minor axis. Find the volume of the solid revolution.

A. 201.06B. 150.80C. 1608.50D. 1206.37

11. Find the eccentricity of an ellipse with major axis 8 and minor 6.A. ¾B.4/2C. 4/3D. 0.66

!2. Find the length of the latus rectum of an ellipse with major axis 8 and minor axis 6.

A. 3.5B. 4.5C. 4D. 5

94. Find the smallest number which when you divide by 2, the remainder is 1; when you divide by 3, the remainder is 2; when you divide by 5, the remainder is 4 and which when you divide by 6, the remainder is 5.

A. 39B. 49C. 59D. 69

Page 40: Mathematics

95. A man bough 20 pcs of assorted calculators for P20, 000. These calculators are of three types namely:

a. programmable at P3,000/pcb. scientific at P1,500/pcc. household type

How many programmable calculators did the man bought?A. 5B. 2C. 13D. 15

96. Two ferryboats ply back and forth across a river with constant but different speeds, turning at the riverbanks without loss time. They leave opposite shores at the same instant, meet for the first time at 900 meter from one shore and meet for the second time 500 meters from the opposite shore. What is the width of the river?

A. 2000B. 2200C. 2020D. 2002

97. How much lead must be added to n alloy which is 50% tin and 25% lead to make an alloy which is 60% tin and 20% lead?

A. 0B. 1 kgC. 2 kgD. 3 kg

98. A pipe can fill up a tank with drain open in 3 hrs. If the pipe runs with the drain open for 1 hr. and then the drain is closed, it ill take 45 more minutes for the pipe to fill up the tank. If the drain will be closed right at the start of filling how long will it take for the pipe

to fill up the tank?A. 1.1 hrs.B. 1.125 hrs.C. 1.25 hrs.D. 1.3 hrs.

99. Ding can finish the job in 8 hrs. Tito can do it in 5 hrs. If Ding wok for 3 hrs. and then Tito was asked to help him finish it, how long Tito will have to work with Ding?

A. 25 hrsB. 25/13 minC. 1.923 hrsD. 30 hrs

100 Find the volume of the solid generated by revolving the area bounded by x = y2 and x = 2- y 2 about the y – axis.

A. 8 pi/3B. 16 pi/3C. 10 pi/3D. 7 pi/3

1 B 21.C 41.B 61.D 81.A 2. B 22.A 42.C 62.D 82. C 3. A 23.B 43.A 63.B 83.4. B 24.B 44.B 64.C 84.A5. A 25.D 45.D 65.A 85.D6. C 26.A 46.C 66.B 86.A7. B 27.A 47.A 67.D 87.B8. B 28.A 48.A 68.C 88.B9. C 29.C 49.B 69.B 89.A10 A. 30. 50.C 70.D 90.B11. D 31.C 51.B 71.A 91.B12. B 32.B 52.C 72.D 92.A13. D 33.D 53.A 73.A 93.A14. D 34.B 54.A 74.A 94.C15. C 35.B 55.A 75.C 95.B16. A 36.A 56.C 76.A 96.B17. D 37.C 57.C 77.A 97.A18. B 38.B 58.C 78.A 98.B19. A 39.C 59.B 79.D 99.C20. D 40.A 60.A 80.C1 00.B

MATHEMATICS MODULE 2

1. When the corresponding elements o two rows of determinant are proportional, then the value of the determinants is:

a. Multiplled by the ratiob. Zeroc. Unknownd. One

2. When two rows are interchanged in position, the value of the determinant will be:a. Unchangedb. Becomes zeroc. Multiplied -1d. Unpredictable

3. If every element of a row (or column) are multiplied by a constant k, then the value of the determinant is:

a. Multiplied by –nkb. k to the nc. Multiplied by kd. Anyone of the above may be true

4. If the quadratic equation ax2 + bx + c = 0, when b2 is equal than 4ac. then the root are:

a. Equalb. Real and unequalc. Imaginaryd. Extraneous

Page 41: Mathematics

5. In the quadratic equation ax2 + bx + c = 0, when b2 is greater than 4ac, then the root are:

a. Equalb. Real and unequalc. Imaginary d. Extraneous

6. In the quadratic equation ax2 + bx + c = 0, if r1 + r2 represent the roots, the r1 = r2

is equal to:a. b/ab. c/ac. –b/ad. –c/a

7. They are equation whose members are equal only for certain (for possibly) no values of unknown.

a. Conditional equationb. Inequalitiesc. fix equationd. Temporary equation

8. Roots which are equal to zero are called the:a. Trivial rootsb. Identicalc. Symmetricd. Rational

9. When all x are replaced by y and all y are replaced by x and the equation remains the same, then the equation is said to be:

a. Equivalent b. Identicalc. symmetricd. Rational

10. The number 0.123123123 … is:a. Irrationalb. Surdc. Transcendentald. Rational

11. To eliminate the surd, we multiply it by its ______________:a. Squareb. Cubec. Reciprocald. Conjugate

12. The letter D in the Romans Numerals is equivalent to:a. 50b. 500c.5000d.50000

13. A statement which is accepted without proof:

a. Postulateb. Lemmac. Theoremd. corollary

58. There is mo change in the motion of the body unless a resultant force is acting on it. This law is known as:

a. the law of Inertiab. Third law of Newtonc. Law of resistanced. Doppler’s principle

59. The energy which body possess by virtue of its positions, configuration or internal mechanism is called ________.a. potential energyb. Kinetic energyc. Mechanical energyd. electrical energy

60. If the mass of the body is expressed in grams and the velocity in cm/sec, the kinetic energy is expressed in:a. Ergsb. Joulesc. Coulombsd. Slugs

61. Energy is given to a body or systems of bodies when work is done upon it. In this process there is merely a transfer of energy from one body to another. In such transfer, no energy is created or destroyed; it merely changes from one to another. This statement is known as:

a. The law of conservation of energyb. Energy transformationc. Coulomb’s lawd. law of power

62. The deformation of elastic body is directly proportional to the applied force, provided that the elastic limit is not exceeded. This theory is known as:

a. Hooke’s lawb. Young theoryc. Keppler’s lawd. Bulk modulus

63. If an external pressure is applied to a confined fluid, the pressure will be increased at every point in the fluid by the amount of external pressure. This theory is known as:

a. Pascal’s lawb. Hydraulic lawc. Hydrostatic law d. Brangg’s law

Page 42: Mathematics

64. A body wholly or partially submerged in a fluid experiences an upward force equal to the weight of the fluid displaced. This theory is known as:

a. Archimedes principleb. Boyle’s lawc. Third law of Newtond. Fluid theory

65. If the temperature of a confined gas does not change the product of the pressure and volume is constant. This statement is known as:

a. Boyle’s lawb. Young’s lawc. Gay lussac lawd. Charles law

66. At any two points along a streamline in an ideal fluid in steady flow, the sum of the pressure, the potential energy per unit volume, and the kinetic energy per unit volume has the same value. This concept is known as:

a. Bernoulli’s theoremb. Fluid theoryc. Hydraulic theoremd. Pascal theorem

Answers:

1. B 21 B 41.A 61.A2. C 22.B 42.A 62.A3. C 23.B 43.C 63.A4. A 24.C 44.B 64.A5. B 25.B 45.D 65.A6. C 26.B 46.C 66.A7. A 27.A 47.B8. A 28.C 48.B9. C 29.D 49.A10.D 30.C 50.D11.D 31.B 51.D12.B 32.A 52.A13.A 33.B 53.D14.C 34.B 54.C15.A 35.A 55.A16.D 36.A 56.C17.B 37.D 57.B18.D 38.B 58.A19.A 39.A 59.A20 A 40.D 60.A

MATHEMATICS MODULE 1

1. The solid formed by revolving the ellipse about is minor axis is called ________ .

a. spheroidb. oblate spheroidc. prelate spheroidd. ellipsoid

2. When two planes intersect with each other the amount of divergence between the two planes is expressed by measuring the:

a. polyhedral angleb. plane anglec. reflex angled. dihedral angle

3. If the product of the slope of any two straight lines is negative. One of this are said to be:

a. parallelb. skewc. perpendiculard. Non-intersecting

4. The logarithm of 1 to any base is:a. zerob. onec. infinityd. intersecting

5. The maximum displacement of vibration from the equilibrium is called:a. frequencyb. speedc. amplitude d. period

6. If the velocity of the body is doubled,a. The kinetic energy is quadrupledb. the kinetic energy is halvedc. the potential energy is halvedd. the potential energy is doubled

7. A body traveling to a circle with constants speed: a. Is accelerated b. Has constant velocityc. Does no move

8. Ivory soap floats in water because:a. All matter has mass

Page 43: Mathematics

b. The density of ivory soap is unityc. The specific gravity of ivory soap is greater than that of waterd. The specific gravity of ivory soap is greater than that of water

9. When two wave of the same frequency, speed and amplitude traveling in opposite directions are superimposed:

a. Destructive interference always resultsb. Constructive interference always resultsc. Standing waves are producedd. The phase difference is always zero

10. The velocity of a wave is:a. The product of the frequency and wavelengthb. Distance of the crest to the next crestc. Always equal to 186.00 miles per. sec.d. The ratio of the frequency to wavelength

11. Acceleration is:a. the same as velocityb. the same as displacementc. the rate of change of velocityd. always zero

12. Mass is the quantitative measure oa. inertiab. gravityc. weightd. momentum

13. A sequence of number where every term is obtain by adding all the preceding terms of a square number series such as 1, 5, 14, 30, 55, 91 …is called _________.

a. Triangular number b. Tetrahedral numberc. Euler’s number

98. Square root of the product of two terms of a geometric progression.a. Median b. Meanc. Geometric meand. Geometric term

99. Which of the following cannot be probability?a. 0.1b. 0c. 1d. 0.232323

100. If a is the number of times that an event will take place and b is the number of times that it will not take place, then the probability that it will take place is:

a. a/bb. b/ac. a/(b+a)d. ab(a+b)

Answers:

1. B 21. C 41. A 61. D 81. C2. D 22. A 42. C 62. C 82. C3. C 23. A 43. C 63. B 83. D4. A 24. A 44. A 64. D 84. C5. A 25. A 45. D 65. A 85. A6. D 26. B 46. D 66. A 86. B 7. A 27. A 47. D 67. A 87. B8. D 28. A 48. 68. A 88. A 9. C 29. A 49. A 69. A 89. B10. A 30. A 50. B 70. B 90. D11. C 31. A 51. A 71. B 91. D12. A 32. B 52. A 72. D 92. A13. D 33. B 53. A 73. C 93. A14. A 34. C 54. A 74. A 94. B15. A 35. D 55. B 75. B 95. A16. A 36. A 56. A 76. C 96. C17. A 37. D 57. B 77. B 97. C18. A 38. D 58. A 78. A 98. C19. A 39. A 59. B 79. B 99. C20. A 40. B 60. A 80. A 100.C

PLANE GEOMETRY

Definitions of Terms

Axiom – a statement of truth of which is admitted without proof.Theorem – a statement of truth which must be established by proof.Corollary – a statement of truth of which follows with little or no proof from a theorem.Postulate – In construction or drawing of lines and figures of which is admitted without proof.Hypothesis – part of a theorem which is assumed to be true.Conclusion – part of a theorem which is to be proved.Converse of a Theorem – another theorem wherein the hypothesis and conclusion of the first are reversed, i.e. the hypothesis becomes the conclusion and the conclusion becomes the hypothesis.

CIRCLE

Circle – locus of points which are at the same distant from a point within, called the center.Diameter – a line passing thru the center, terminating at both ends on the circle.Radius – a line drawn from the center to the circle

Page 44: Mathematics

Arc – a part of the circle.Chord – a line joining two points on a circle.Secant – an indefinite line intersecting the circle in two points.Tangent – an indefinite line touching a circle at only one point.Segment – position of a circle between chord and its arc.Sector – position of a circle between two radii and arc.Inscribed angle – an angle whose vertex is a point in the circle, the sides of which are chords.Central angle – an angle whose vertex is at the center of the circle the sides of which are radii.Concentric circles – circle having the same center with unequal radii.Tangent circles – circles tangent to the same line at the same point.Inscribed circle – (in a polygon) when the sides of the polygon are tangent to it.Circumscribed circle – (about a polygon) when it passes through the vertices of the

polygon.Circular ring/annulus – area included between two concentric circles of unequal radii.

POLYGON

Polygon – a plane closed by broken linesRegular polygon – polygon whose angles are equal and all of whose sides are equalSimilar polygon – polygon whose corresponding angles are equal and their

corresponding sides are proportional.Center of a polygon – common center of its inscribed and circumscribed circle.Diagonal of a Polygon – line joining any two non-consecutive vertices.Apothem (of a regular polygon) – the perpendicular line drawn from the center of the inscribed circle to any one of the sides. It is the radius of the inscribed circle.

Classifications of Polygon

Number of Sides Polygon3 triangle4 quadrilateral5 pentagon6 hexagon7 heptagon8 octagon9 nonagon10 decagon11 undecagon12 dodecagon

Trapezoid – a quadrilateral with only two sides of which are parallel.Parallelogram – a quadrilateral whose opposite sides are parallel.Rhombus –a parallelogram with equal sides and oblique angles.Rectangle – a parallelogram whose angles are right angles.Square – a rectangle with equal sides.

Isosceles trapezoid – one whose non-parallel sides are equal.Isometric figures – figures whose parameters are equal.

Properties of Plane Figures

1. The exterior angle of a triangle is greater either non-adjacent interior angle and is equal to their sum.

= +

2. The diagonal of a parallelogram divides the parallelogram into two congruent triangles.

ACD ABC

3. The opposite sides and opposite angles of a parallelogram are equal.

AB = CD ADC = ABCAD = BC BAD = BCD

4. The diagonals of a rhombus are perpendicular to each other and bisect the angles through which they pass.

ACBDACD = ACB = ½ ABCADB = CDB = ½ ADC

5. The diagonals of a parallelogram bisect each other.

AX = XC = ½ AC X BX = XD = ½ BD

A B

C D

d

A B

C D

A B

C D

A B

C D

C

Page 45: Mathematics

6. The median of the hypotenuse of a right triangle have equal distances from the three vertices.

Mc AMc = BMc = CMc

7. The line segment which joins the midpoint C

D EDE AB

A B DE = ½ AB

8. The median of a trapezoid is parallel to the bases and equal to one-half of their sum.EF AB

EF = (AB+CD)/2

9. The intersection of the three angle bisectors meet at a common point is called incenter, which is equidistant from the three sides of a triangle. The inscribed circle is called incenter.

OP = OQ = OR

10. The intersection of the three perpendicular bisectors meet at a common point called circumcenter, which is equidistant from the three vertices of the triangle. The circle whose center is 0 touching the three vertices is called circumcenter.

OB = OA = OC

11. The intersection of the three altitudes of a triangle meet at a common point called orthocenter.

12. The three medians of any triangle meet at a common point which is two-thirds of the distance

from each vertex to the midpoint of the opposite side. The point of intersection is called the centroid.

2/3 BR = BM2/3 CP = CM2/3 AQ = AM

13. The altitude upon the hypotenuse of the right triangle is the mean proportional between the segments of the hypotenuse.

CD = (AD)(DB)

14. A central angle is measured by its intercepted arc.

15. The inscribed angle is measured by one-half of the intercepted arc.

ABC = ½ AC

16. An angle formed by two chords intersecting within a circle is measured by one half of the sum of the arcs intercepted by it and its vertical angle.

(PA)(PB) = (PC)(PD)

A B

B

CD

A

FE

AB

Q

CP

R

o

C

o

B

A

B

P

B

Q

CA

R C

M

AC

A B

C

D

C

B

A

A

C

BP

D

Page 46: Mathematics

17. The product of one entire secant and its external segment equals the product of the other entire secant and its external segment.

(AD) = (AB)(AC)

18. The tangent line is a mean proportional between the entire secant and its external segment.

A1/A2 = (L12) / (L2

2)

19. The sum of interior angles of any polygon of n sides is

A + B + C… = (n–2)180

20. Interior angle ( i) and exterior angle ( 0 ) of a regular polygon of n sides is

I = [(n–2)/n] 180o = 360 / n

Formulas for Plane Figures

I. Area of Triangles

I.0 Given sides a, b, and c(Hero’s formula) a b

A = s(s-a)(s-b)(s-c)Where:

s = ½ (a+b+c) c

2. Given base b and altitude h

A = ½ bh

Hbb h b

b

3. Given: equilateral triangle of side sA = 3 s2

4

s s

s

4. Given: two adjacent sides and the included angleA = ½ bc sin A = ½ ac sin A = ½ ab sin

b a

c5. Given: at least two angles and a side

h

C

A

L1

B

D

L2

A2

i

A1

F

0

E D

C

BA

Page 47: Mathematics

A = a 2 sin sin 2 sin

A = b 2 sin sin 2 sin

A = c 2 sin sin 2 sin

6. Given: sides a, b, and c and inscribed in a circle of radius RA = abc 4R

R

7. Given: sides a, b, and c and circumscribed about a circle of radius rA = rs

Where:s = a + b + c

2

II. Area of Parallelogram1. Given: base b and altitude h

A = bh b2. Given: sides a and b and their included angle

A = ab sin

a

III. Area of a TrapezoidGiven: bases a and b and altitude h

A = ½ (a+b) a

h

b

IV. Area of cyclic quadrilateral (Bramaguphta’s Formula)Given: sides a, b, c, and d

A =(s-a)(s-b)(s-c)(s-d)Where:

s = ½ (a+b+c+d)A+C = 180B+D = 180

V. Area of RhombusGiven: diagonals d1d2

A = ½ d1d2

VI. Area of Trapezium1. Given: diagonals d1 and d2 and their included angle

A = ½ d1d2 sin

VII. Area of Regular Polygon1. Given : n sides, each of length s.

A = ½ ns2 cot (/n)2. Given: n sides, and apothem a.

A = na2 tan (/n)3. Given: n sides inscribed in a circle of radius R.

A = ½ nR2 sin (2/n)

VIII. Area (A) and Perimeter (P) of a Circle1. Given: radius r and diameter d

A = r2 = d 2 r 4

P = 2r = d

dIX. Area of Annulus

Given: circle of radii r1 and r2

a r b c

a b

r

c

BD

C

A

cb

a

d1

d2

d

d1 d2

d1 Ad2

R

a

a

Page 48: Mathematics

A = (r12 – r2

2) r1 Where r1>r2 r2

X. Area of a sector of a circleGiven: radius r and

s = r rA = ½ r2 s

XI. Area of a segment of a circleA = ½ r2 ( – sin) r

Where is in radius r

XII. Area and Circumference of EllipseGiven: major axis a and minor axis b

A = abC = 2 (a2+b2)/2

a

b

XIII. Area of a parabolic segmentGiven: base b and height h

A = 2/3 bh

h

b

Supplementary Problems

1. Bisectors of the 3 angles of a triangle meet at a common point called the __________

a. orthocenter b. centroidc. incenter d. circumcenter

2. The perpendicular bisector of the sides of a triangle pass through a common point called the __________

a. orthocenter b. centroidc. incenter d. circumcenter

3. Which of the following is not a property of a circle?a) through 3 points not in the straight line one circle and

only 1 can be drawnb) a tangent to a circle is perpendicular to the radius at the

point of tangency and conversely.c) an inscribed angle is measured by ½ of the intercepted

arc. d) the arc of 2 circles subtended by equal central angle are

equal.4. Which of the following is not a property of a triangle?

a) the sum of the 3 angles of the triangle is equal to two right triangles.

b) the sum of the 2 side of the triangle is less than the 3rd side

c) if the 2 sides of the triangle are unequal, the angles opposite are unequal.

d) the altitude of a triangle meets in a point

5. The radius of the circle inscribed in a polygon is called asa) internal radius b) radius of gyration c) apothem d) hydraulic radius

6. A polygon with 12 sides is called as a) bidecagon b) dodecagon c) nonagon d) pentedecagon

7. A polyhedron having bases 2 polygons in parallel plane and for lateral faces triangles or trapezoid with 1 side lying on 1 base and the opposite vertex or side lying on the other base of the polyhedron is

a) pyramid b) conec) prismatoid d) rectangular parallelepiped

8. An angle greater than a straight angle but less than 2 straight angles is called asa) complement b) supplementd) complex d) reflex

9. A part of a circle is often called asa) sector b) cordb) arc d) segment

10. An angle whose vertex is appoint on the circle and whose sides are cords is known asa) interior angle b) vertical angle

Page 49: Mathematics

c) acute angle d) inscribed angle11.Two angles whose sum is 360 degrees are said to be

a) supplementary b) complimentaryb) elementary d) explementary

12. All circles having the same center but with unequal radii are called as

a. eccentric circles b. concentric circlesc. inner circles d. Pythagorean circles

13. A circle is _________ outside the triangle if it is tangent to one side and the other two sides prolonged.

a. inscribed b. escribedc. circumscribed d. tangent

14. A triangle having three sides of unequal length is known asa. equilateral triangle b. scalene trianglec. isosceles triangle d. equiangular triangle

15. In a proportion of four quantities, the first and the fourth terms are referred to as the

a.means b. extremesc. denominators d. axiom

16. A statement the truth of which is admitted without proof isa. theorem b. corollaryc. postulate d. axiom

17. The part of the theorem which is assumed to be true is thea. corollary b. hypothesisc. postulate d. conclusion

18. In Geometry,the construction or drawing of lines and figures, the possibility of which is admitted without proof is called the:

a. corollary b. theoremc. postulate d. hypothesis

19. A statement the truth of which follows with little or no proof from the theorem isa. corollary b. axiomc. postulate d. conclusion

20. A polygon is ______ when no side, when extended, will pass through the interior of the polygon.

a. convex b. equilateralc. isoperimetric d. regular

21. A circle is said to be _______ to a polygon having the same perimeter with that of the circle

a. congruent b.isoperimetricc.proportional d. similar

22. The intersection of the sphere and the plane through the center is thea. great circle b. small circlec. poles d. polar distance

23. Points that lie on the same plane are said to bea. collinear b. coplanarc. dihedral d. parallel

24. What kind of a quadrilateral is always formed by connecting the midpoints of the consecutive sides of a quadrilateral? Ans. parallelogram

25. The angles of a pentagon are in the ratio 3: 3: 3: 4: 5. Find the largest angle. Ans. 150deg

26. The legs of a triangle are in the ratio 3: 3 and its area is 108 sq. cm. Find the length of the legs. Ans. 12 cm., 18 cm

27. Find the side of a regular octagon inscribed in a circle of radius 19 cm.

28. The upper and lower bases of a trapezoid are 6cm and 12cm respectively, and the altitude is 4cm. The non-parallel sides of the trapezoid are produced until they meet at P. Find the altitude of the triangle whose vertex is P and whose base is the lower base of the trapezoid. Ans. 8cm

29. A secant and a tangent are drawn to circle from the same point. If the internal segment is 1cm longer than the external segment, and if the tangent is 6cm long, find the length of the secant. Ans.145 cm.

30. Two parallel chords of a circle are 16cm and 30 cm long respectively, and the distance between them is 23cm long. Find the distance each is from the center, and radius of the circle. 15cm and 8cm., r = 17 cm.

31. The diagonals of a rhombus have the ratio of 3:4. The area of the rhombus is 96 sq.cm. Find a side of the rhombus. Ans. 10cm

32. The areas of two similar triangles are to each other as 9:25. The perimeter of the first is 36 cm. What is the perimeter of the second triangle? Ans. 60cm.

33. The side of a rhombus is 13cm, and one of its diagonals is 24cm. Finds its area. 120 sq.cm.

34. In a circle whose radius is 10cm, a chord bisects the radius at right angles. Find the area of the smaller of the two segments into which the chord divides the circle. 25/3(2 - 33)sq cm.

35. From a point P, exterior to circle C, the tangents are drawn to the circle. The distance of P from the center of C is 4cm. The length of each tangent is 2cm. Find the diameter of the circle. Find the area bounded by the tangents and the arc between them.Ans. d = 43 cm, A = (43 - 2) sq. cm.

Page 50: Mathematics

36. Find the area of the folded triangle shown below.

a) 15 b) 20 c) 25 d) 4037. The area of a triangle whose angles are A = 69.159˚, B = 34.246˚, C = 84.595˚ is 680.60 m2. The longest side is

a) 15.387 b) 52.431 c) 37.853 d) 64.97438. Find the area of a decagon that can be inscribed in a circle with radius 10 cm.

a)11.387459 b)10.066446 c)37.853527 d)64.97413639. Find the area of a regular 5-pointed star that can be inscribed in a circle with radius 10 cm. a) 112.257 b) 56.129 c) 114.535 d) 124.43140. Find the radius of the largest circle that can be inscribed in the triangle with sides a = 8cm, b = 15cm, and c= 17cm Aa) 3 b) 1.8 c) 30 d) 8.541. Find the radius of the smallest circle that circumscribe the triangle in the previous problem. a) 8.5 b) 10.2 c) 9.51 d) 7.75 42. Chords AB = 12 cm and BC = 8cm of a circle forms 120˚. Find the radius of the circle.

a) 10 b) 12.164 c) 14.742 d) 24.634

43. The radius of the smallest circle that can be circumscribe a right triangle of sides a, b, and c.

a. a + b + c     b. a + b – c 2 2

C. a – b + c d. c 2 244. The radius of the largest circle that an be inscribed in a right triangle of sides a, b and c.

a. a + b + c     b. a + b – c 2 2

c. a – b + c d. c 2 2

Find CD.

45. Find the area of the cyclic quadrilateral.

AP = 50cm, BP = 28cm, DP = 56cm, Ө = 30degrees

a) 456 b) 627 d) 364 d)52546. The internal angle of a polygon is 150˚ greater than its external angle, how many side has the polygon?

a) 9 b) 6 c) 7 d) 847. The 30˚ angle of a right triangle is bisected. Find the ratio of which opposite side is divided.

a) 1:2 b) 1:3 c) √2:2 d) 1:448. A certain angle has a supplement 5 times the compliment. Find the angle.

a. 67.5˚ b. 58.5˚c. 30˚ d. 27˚

49. Find the supplement of an angle whose compliment is 62 degreesa. 30˚ b. 28˚c.152˚ d. 118˚

50. The sum of the interior angles of a polygon is 540 degrees. Find the number of the sides.a. 5 b. 6 c. 8 d. 11

51. Six lines are situated in the plane so that no two are parallel and no three are congruent. How many points of intersection are there?

A. 13 b. 14 c. 15 d. 1652. The area of a square field exceeds another square by 56 square meters. The perimeter of the larger field exceeds ½ of the smaller by 26 m. What are the sides of each field? Ans. larger field, 9m or 25/3m; smaller field, 5m or 11/3m

53 . The sum of the areas of two unequal square lots is 5,200 square meters. If the lots were adjacent to each other, they would require 320 meters of fence to enclose the combined area formed by them. Find the dimensions of each lot. Ans.60m and 40m, or 68m and 24m.

Page 51: Mathematics

54. Six lines are situated in the plane so that no two are parallel and no three are congruent. How many points of intersection are there?

a. 13 b. 14 c. 15 d. 1655. A triangle is inscribed in ac ircle such that one side of the triangle is a diameter of the circle. If one of the acute angles of the triangle has a measure of 60˚ and the opposite side of that angle has a length 12, then the nearest value of the radius of the circle is.

a. 6.93 b. 1.93 c. 9.6 d. 5.856. The hypotenuse of a right triangle is 34 cm. Find the length of the two legs if one leg is 14 cm longer than the other.

a. 18 and 32 cm b. 15 and 29 cmc. 17 and 32 cm d. 16 and 30 cm

57. The sum of the interior angles of a polygon is 540 degrees. Find the number of the sides.a. 5 b. 6 c. 8 d. 11

58. A circle of radius 6 has half its area removed by cutting off a border of uniform width. Find the width of the border.

a. 22 b. 135 c. 375 d. 17659. A circle is inscribed in a 3, 4, 5 right triangle. How long is the line segment joining the point of tangency of the “3-side” and the “5-side”?

a. 1.28 b. 1.35 c. 1.46 d. 1.7960. Two figures having equal perimeter are said to

a. congruent b. isoperimetricc. equal d. similar

61. A right triangle whose length and side may be expressed as ratio of integral unitsa. isosceles triangle b. scalene c. pedal triangle d. primitive triangle

62. The perpendicular bisector of the sides of a triangle intersect at the point known as thea. orthocenter b. cicumcenterc. centroid d. incenter

63. The bisectors of the 3 angles of a triangle meet at a common called thea. circumcenter b. centroidc. orthocenter d. incenter

64. An isosceles trapezoid ABCD, AB=5, BC=AD = 3 and CD = 8. Find the length of the diagonals.

a. 7.5 b. 8 c. 7 d. 665. The diagonals of rhombus have length 4 and 6 inches. Find the area of the region inside the rhombus but outside the circle that is inscribed in a rhombus.

a. 3.3 b. 3.4 c. 2.9 d. 2.85

66. Two equilateral triangles, each with 12-cm sides, overlap each other to form a 6-point “Star of David”. Determine the overlapping area, in sq-cm.

a. 36.64 b. 41.57 c. 28.87 d. 49.88

67. Let D be the set of vertices of a regular dodecagon (12 sided plane polygon). How many triangles may be constructed having D as the vertices?

a. 220 b. 120 c. 240 d. 180

68. A group of children playing with marble place 50 pieces of the marble inside the cylindrical container with water filled to a height of 20 cm. If the diameter of aech marble is 1.5 cm and that of the cylindrical container 6 cm, what would be the new height of water inside the cylindrical container after the marbles were placed inside?

a. 23.125 b. 24.125c. 26.125 d. 25.125

69. A horizontal cylindrical tank with hemispherical ends is to be filled with water to a height of 762mm. If the total inside length of the cylinder is 3,600mm, find the volume of water in cubic meters that will have to filled into the tank up to the required height. If one edge of a cube and the volume of the cube respectively in sq.cm. and cu.cm.

a. 864 and 1728 b. 684 and 1728c. 864 and 1728 d. 864 and 1729

70. A pyramid with square base has an altitude of 25 cm. If the edge of the base is 15cm, calculate for the volume of the pyramid.

a. 1885 b. 1875 c. 1785 d. 1958

71. If a right circular cone has a base radius of 35 cm and an altitude of 45cm, solve for the total surface area in square cms.

a.10116 b. 10117 c. 11117 d. 12117

72. Two circles of radius 4 and 6.93 are placed in a plane so that their circumference intersect at right angles. Find the area of the interlapping region.

a. 14.81 b. 13.92 c. 14.18 d. 15.1

73. One side of a regular octagon is 2. Find the area of the region inside the octagon.a. 3.91 b. 13.92 c. 14.18 d. 15.1

SOLID GEOMETRY

Definition of Terms:Polyhedron – a solid bounded by planes.Regular Polyhedron – polyhedron whose faces are congruent regular polygons and

whose polyhedral angles are regular polygons and whose polyhedral angles are equal.

There are only five regular polyhedron:Tetrahedron – 4 facesHexahedron – 6 facesOctahedron – 8 facesDodecahedron – 12 facesIcosahedron – 20 faces

Cube – a polyhedron whose six faces are all squares

Page 52: Mathematics

Rectangular Parallelepiped – polyhedron whose six faces are all rectangles.

Prism – polyhedron of which two faces are equal polygon in parallel planes and the other faces are equal parallelogram.

Pyramid – a polyhedron of which one faces is a regular polygon and other faces are triangles which have a common vertex.

Regular Prism – prism whose lateral edges are perpendicular to its bases.

Regular Pyramid – pyramid whose base is a regular polygon and whose altitude passes through the center of base.

Slant height – altitude of lateral faces.

Section – polyhedron is the plane figure formed by a plane passing through the solid.

Convex Polygon – a polygon whose each interior angle of which is less than 180.

Frustum of Pyramid – section of the pyramid between the base and a section parallel to the base.

Cone – a solid bounded by a conic surface and a plane intersecting all the elements.

Dihedral Angle – the divergence of two intersecting planes.

Sphere – a solid bounded by a surface all points of which are equidistant from a point called center.Great circle – the intersection and a plane passing through the center.

Small circle – the intersection of a sphere and a plane not passing through the center.

Quadrant – one-fourth of a great circle.

Zone – portion of sphere bounded by a spherical polygon and the plane of its sides.

Lune – portion of a sphere lying between two semi – circles of great circle.

Spherical pyramid – portion of sphere bounded by a spherical polygon and the plane of its sides.

Spherical Sector – portion of sphere generated by the revolution of circular sector about any diameter of the circle of which the sector is apart.

Spherical segment - portion of sphere included between two parallel lines.

Spherical wedge – portion bounded by a lune and the planes of two great circles.Focus – a solid formed by revolving a circle about a line not intersecting it.

Formulas in Solid Mensuration

1. Cube with edge s:

Volume :

Surface Area :

2. Rectangular parallelepiped with edges a, b, c and diagonal D:

Volume :

Surface Area :

Diagonal :

3. Volume of a prism with base B and altitude h:

4. Volume of a pyramid with base B and altitude h:

5. Volume of a prismatoid with bases b and B, midsection M and altitude h:

Page 53: Mathematics

Prismoidal Formula:

6. Sphere of radius r or diameter D:

Volume : or

Surface Area : or

7. Right circular cylinder with radius r and altitude h:

Volume :

Lateral Area :

8. Right circular cone with radius r and altitude h:

Volume :

Lateral Area :

(l = slant height)

9. Frustum of a right circular cone with base radii r and R and altitude h:

slant height l:

Volume :

Lateral Area :

10. Frustum of a pyramid with bases b and B and altitude h:

Volume :

Lateral Area :

where l = slant height

p = perimeter of base b

P = perimeter of base B

11. Area Z of a zone with altitude h on a sphere of radius R:

12. Spherical segment of one base and altitude h on a sphere of radius R:

Volume :

Total Area :

13. Spherical segment of two bases with radii a and b and altitude h on a sphere

of radius R:

Volume :

Total Area :

14. Spherical Cone = a spherical sector having only one conical surface

Page 54: Mathematics

Volume : or

where V = volume of spherical sector

Z = area of the zone which forms the base of the sector

R = radius of the sphere

h = altitude of the sphere

15. Ellipsiod

V = 4/3 abc ba

For oblate spheroid cV = 4/3 ab2

For prolate SpheriodV = 4/3 a2 b

16Paraboloid h

V = ½ a2 h

a

17. Ungula V = 2/3 r2h hS = 2 rh

r

Supplementary Problems:

1. The slant height of a regular hexagonal pyramid is 9 cm. and a side of the base is 6 cm. Find the volume of the inscribe cone. Ans. 27 6 cu. cm.

2. A rectangle, 4 cm wide and 9 cm long is revolved about its longer side. Find the total surface area and the volume of the cylinder generated. Ans. T = 104 sq. cm. ; V = 144 cu. cm

3. A solid wooden cone is cut into two parts, the cut being parallel to the base and halfway between the base and the vertex. Find the ratio of the weights of the two parts. Ans. 7 : 1

4. Find the area of a zone of one base, if the base is a circle of radius 6 cm and is 8 cm from the center of the sphere. Ans 40 sq. cm.

5. Three spheres of radii a, 2a, and 3a are melted and formed into a new sphere. Find the surface of this sphere. Ans. 24 36 a2

6. A sphere whose radius is 10 cm is cut into three parts by parallel planes 8 cm and 6 cm. from the center respectively, and on the opposite sides of the center. Find the total surface of the part in the middle. Ans. 380 sq. cm.

7. In a frustum of a regular pyramid the bases are squares with sides 6 cm and 12 cm respectively. If the lateral area of the frustum is just half of its total area, what is the volume?

Ans: 420 m3

8. The volume of two similar polyhedron are 64 and 125 cu cm. respectively. If the total surface of the first is 112 square centimeters, find the total surface area of thesecond.Ans.380sq.cm

9. Find the total surface and the volume of a tetrahedron whose edges are equal to a.Ans: T = 3 a2

,V = (2/12) a3

10. A sphere of radius 5 cm and a right circular cone of bass radius 5 cm and height 10 cm stand on a plane. Find the position of a plane that cuts the two solids in equal circular sections.

Ans: d = 2 cm and h = 10 cm

11. A cylindrical tin can has its height equal to the diameter of its base. Another cylindrical tin can with the same capacity has its height equal to twice the diameter of its base. Find the ratio of the amount of tin required for making the two cans with covers. Ans. 0.9524

12. Find the volume of the frustum of a regular square pyramid whose base edges are 4 cm and 10 cm and whose slant height is 5 cm. Ans. 208 cu cm

13. Find the volume of a prism having an altitude of 13 cm and a rectangular base of 8 cm long and 4 cm wide. Ans.416 cm3

Page 55: Mathematics

14. The base of a right prism is a rhombus whose sides are each 10 cm long and whose shorter diagonal is 12 cm. Find its volume if the altitude is 8 cm. Ans. 768 cu cm

15. The diameter of two spheres are in the ratio 2:3 and the sum of their volumes is 1,260 cu m. Find the volume of the larger sphere. Ans. 972 cu mIf the volume of a cube is 625 m3, find the length of the diagonal. Ans. 14.8 m

16. Find the volume of a regular triangular pyramid whose slant height is 17 cm and whose altitude is 15 cm.Ans. 960 cu cm

17. Find the capacity in liters of a pail in the form of a frustum of a circular cone if the radii of the bases are 10 and 15 cm and the depth of the pail is 36 cm. Ans. 17.9 L

18. The space occupied by the water in a reservoir is the frustum of a right circular cone. Each axial section of this frustum has an area of 28 sq m and the diameter of the upper and lower bases are in the ratio 4:3. If the reservoir contains 148 /3 cu m, find the depth of the water in the reservoir. Ans. 4m

19. The lateral surface area of a right circular cylinder is 330 sq cm. If its altitude is 20 cm, find the diameter of its base. Ans. 16.5 cm

20. A reservoir is in the form of the frustum of an inverted square pyramid with upper base 24 m, lower base edge 18 m and altitude 9 m. How many hours will it require for an inlet pipe to fill the reservoir if water flows in at the rate of 5,000 liters per minute? Ans. 13.32 hr

21. A horizontal cylindrical tank with diameter of 0.60 m and 3.66 m long is filled with water to a depth of 0.46 m. Find the number of liters of water in the tank. Ans. 851 L

22. A sphere is inscribed in aright circular cone with altitude 15 cm. If the slant height of the cone is equal to the diameter of its base, find the surface area of the sphere. Ans. 100 sq cm

23. Use the prismoidal formula to find the volume of the common part of two cylinders, each with a radius of 6 cm, which intersect at right angles. Ans. 1,152 cu cm

24. If the radius of a sphere is inscribed by 6 cm, its volume is multiplied by 27. Find the radius of the sphere. Ans.3 cm

1. Find the slope of x2y = 8 at point (2, 2)a. –2 b. 2 c. 8 d. 4

2. If y = x lnx, find y”a. 1/x b. ln x c. 1/lnx d. x

3. Evaluate the limit (x - 4)/(x2 – x – 12) as x approaches 4a. 1/7 b. 0 c. infinity d. indeterminate

4. Evaluate the limit lim x – sin x x 0 x3

a. 1/3 b. ¼ c. 1/5 d. 1/65. Find the maximum point on the curve y = x3 – 3x2

– 9x + 5a. (1, 15) b. (3, -22) c. (-1, 10) d. (-3, 21)

6. Determine the velocity of a body which moves according to the law S = 2t3 – t2 + 4 where S is displacement in ft. and t is time in sec at t = 1sec.

a. 2ft b. 6ft. c. 4ft. d. 10ft.7. Find the equation of the tangent to the curve y = x4 – x2 + 2 at x = -1.

a. 2x + y = 0 b. 2x – y = 1 c. 2x – y = 0 d. y + x = 18. Determine the altitude of the largest circular cylinder that can be inscribed in a right circular cone of radius 6 inches and of height 15 inches.

a. 12 inches b. 8 inches c. 5 inches d. 10 inches9. Given a cone of radius R and of altitude H, what percent is the volume of the largest

cylinder which can be inscribed in the cone to the volume of the cone?a. 49% b. 44% c. 50% d. 60%

10. Find the most economical proportion for a box with an open top and a square base.a. b = h b. b = 4h c. b = 3h d. b = 2h

11. The 5m picture hung on a wall so that its bottom edge is 4m above an observer’s eye. How far should the observer stand from the wall so that the angle subtended by the picture at the eye is a maximum.

a. 4.9 b. 5.5 c. 6.7 d. 7.212. What is the largest area of a rectangle that can be inscribed within an ellipse whose

major diameter is 10ft and whose minor diameter is 6ft.a.32.50 sq. ft. b. 28.00 sq. ft c. 30.00 sq. ft d. 34.00 sq.ft

13. The hands of the tower clock are 4 ½ ft and 6 ft long respectively. How fast are the ends approaching at 4 o’clock in ft per minute?

a. – 0.246 b. – 0.203 c. –0.264 d. –0.25614. A man on the wharf (pier) is pulling a rope tied to a raft at a time rate of 0.60 m/sec if

the hands of the man pulling the rope is 3.66m above the level of the water, how fast is the raft approaching the wharf when there are 6.10m of rope out?

a. –0.75 m/sec b. –0.55 m/sec c. –0.45 m/sec d. –0.65 m/sec15. A balloon is rising vertically over a point A on the ground at the rate 15 ft/sec. A point

B on the ground level is with the same horizontal plane as A and 30 ft from it, when the balloon is 40 ft from A, at what rate is its distance from B changing?

a. 12 ft/sec b. 15 ft/sec c. 18 ft/sec d. 21 ft/sec16. What is the percentage error made in the computed surface area of a sphere if the

error made in measuring the radius is 3%a. 3% b. 4% c. 5% d. 6%

17. What is the allowable error in measuring the edge of the cube that is intended to hold 8 cu.m. if the error of the computed volume is not to exceed 0.03 cu.m.?

a. 0.002 b. 0.003 c. 0.0025 d. 0.00118. Find the radius of curvature of the curve y = x3/3 at x = 1.

a. 2 b. /3 c. 3/ d. /219. Find the area bounded by the curves y = 4x – x2, y = 0, x = 1, x = 3.

a. 23/3 b. 22/3 c. 21/4 25/3

Page 56: Mathematics

20. Find the volume generated by revolving the region bounded by y = x2 and y = x about the y= axis.

a. π/8 b. π/10 c. π/6 d. π/1221. If the first derivative of a function is constant, then the function is said to be:

a. constant b. linear c. sinusoid d. exponential22. Three sides of the trapezoid are each 8 cm long. How long is the 4th side when the

area of the trapezoid has the greatest value?a. 16 cm b. 12 cm c. 15 cm d. 10 cm

23. Water is running out from a canonical funnel at a rate of 2 cu. in. per second. If the radius of the top of the funnel is 4 inches and the altitude is 8 inches, find the rate at which the water level is dropping when it is 2 inches from the top.a. 2/9 in/sec b. -2/9 in/sec c. -3/2 in/sec d. 5/9 in/sec

24. Find the area bounded by the parabola x2 = 8y and its latus rectum.a. 10.67 sq. units b. 32 sq. units c. 48 sq. units d. 16.67 sq. units

25. What theorem is used to solve for centroid?a. Pappus b. Varignon’s c. Castiglliano’s d. Pascal’s

26. Find the area (in sq. units) bounded by the parabola x2 – 2y = 0 and x2 + 2y = 8. a. 11.7 b. 4.7 c. 9.7 d. 10.7

27. Find the equation of the curve at every point of which the target line has a slope of 2x.

a. x =-y2 + C b. y = -x2 + C c. y = x2 + C d. x = y2 + C28. The rate of change of a function of y with respect to x equals 2 – y, and y = 8 when x

= 0. Find y when x = ln 2b. 5 b. 2 c. –5 d. –2

29. Solve the differential equation (x2 + y2)dx + 2xydy = 0

c. 3xy2 + x3 = C b. 2xy2 = C c. 3xy + 2 = C d. 3x2 + 2y = C30. Solve the differential equation y” – 5y’ + 6y = 0

d. Y = Ae2x + Be3x b. Y = Ae-2x + Be-3x

c. Y = Ae5x + Be3x d. Y = Ae2x + Be5x

31. Find the equation f the family of orthogonal trajectories of the system of parabolas y2 = 2x + C.

e. y = ce-x b. y = ce2x c. y = cex d. y = ce-2x

32. The rate of change of a certain substance is proportional to the amount of substance is 10 grams at the start and 5 grams at the end of 2 minutes, find the amount of substance remaining at the end of 6 minutes.

f. 1.25 grams b. 2.67 grams c. 3.46 grams d. 2.98 grams33. The rate of population growth of a country is proportional to the number of

inhabitants. If the population of a country now is 40 million and is expected to double in 25 years, in how many years will the population be 3 times the present?

g. 39.6 yrs b. 39.5 yrs c. 37.9 yrs d. 36.9 yrs34. Water at 100C is transferred to a room which is at constant temperature of 60C.

After 3 minutes the water temperature is 90C, find the water temperature after 6 minutes.

h. 82.5C b. 85.2C c. 80C d. 75C

35. A body at 90C cools in 10 mins to 70C in a room temperature of 25C. When will its temperature be 40C?

i. 39.8mins b. 38.8mins c. 36.8mins d. 34.7mins36. Evaluate ln (3 + j4)

j. 1.77 + j0.843 b. 1.61 + j0.927 c. 1.95 + j0.112 d. 1.46 + j0.10237. Find the value of (1 + i)12 where i is an imaginary number.

k. –64 b. 64 c. 4 d. 4i38. Simplify the expression i1997 + i1999

l. 0 b. –1 c. 1 + i d. 1- i39. Express e0.32 + j0.56 in rectangular form

m. 1.167 + j0.732 b. 1.193 + j1.163 c. 1.452 – j0.315 d. 1.684 – j1.462

40. Evaluate cos (0.492+j0.942)n. –1.032 + j0.541 b. 1.302-j0.504 c. 3.12+j1.54 d. 1.48+j0.01

41. Evaluate the value of log (-5)o. 5+j log e b. 5+j log e c. log 5+j log e d. log 5+j log e

42. Find the Laplace transform of t3 e4t

p. 6/(s+4)4 b. 6/(s-4)4+ c. 6/(s-2)2 d. 6(s+4)4

43. Find the Laplace transform of (1 – e-at)q. 1/[s(s+a)] b. 1/(s2 + a2) c. 1/[s(s-a)] d. 1/(s+a)2

44. In complex algebra, we use a diagram to represent a complex plane commonly called:

a. De Moivre’s diagram b. Argand diagramc. Venn Diagram d. Funicular diagram

45. When the corresponding elements of two rows of a determinant are proportional, then the value of the determinant is:

a. unknown b. one c. zero d. multiplied by the ratio46. A sequence of number where the succeeding term is greater than the preceeding

term.a. isometric series b. divergent seriesc. convergent series d. dissonant series

47. In any square matrix, when the two elements of any two rows are exactly the same, the determinant is:

a. unity b. positive integer c. zero d. negative integer48. Which of the following cannot be an operation of matrices?

a. Subtraction b. multiplication c. addition d. division49. Convergent series is a sequence of decreasing numbers or when the succeeding

term is ___________ than the preceding term.a. ten times more b. equal c. greater d. lesser

50. Find the length of the vector (2, 4, 4).a. 8.75 b. 7.00 c. 6.00 d. 5.18

51. Find the value of (1 + i)5, where i is an imaginary number.a. 1 – i b. 1 + i c. -4(1 + i) d. 4(1 + i)

52. Find the equation of the family of orthogonal trajectories of the system of the parabolas y2 = 2x + C.

a. y = Ce-x b. y = Ce2x c. y = Cex d. y = Ce2x

Page 57: Mathematics

53. What is the quotient when 4 + 8i is divided by i3.a. 8-4i b. 8+4i c. -8+4i d. -8-4i

54. Evaluate the expression (1+i2)10, where i is an imaginary number.a. -1 b. 10 c. 0 d. 15

55. Solve for x in (x+yi) (2+4i) = 14 + 8i. a. 3 b. 4 c. 14 d. 8

TRIGONOMETRY

Trigonometry – the branch of mathematics that deals with the solution of triangles.

Angle – the space between two line meeting at a point called vertex.

Kinds of Angles:

1. Acute angle – an angle which measures between 0 to 90

2. Right angle – an angle measuring exactly 90

3. Obtuse angle – an angle which measures between 90 to 180

4. Straight angle – an angle measuring exactly 180

5. Reflex angle – an angle greater than 180 but less than 360

Two General Classes of Triangles

1. Right Triangle – a triangle with a right angle

2. Oblique Triangle – a triangle without a right angle

Oblique Triangles can be further classified as:

1. Acute Triangle – a triangle whose all angles are acute.

2. Obtuse Triangle – a triangle with one obtuse angle.

The Pythagorean Theorem

- states that the sum of the squares of the legs is equal to the square of the

hypotenuse.

Pythagorean Triple – three positive integers satisfying the Pythagorean principle.

Example: 3, 4, and 5; 5, 12, and 13; 20, 21 and 29; 8, 15, 17; 7, 24, 25; etc.

Supplementary Problems:

1. A storm broke a tree 50 ft high so that its top touched the ground 30 ft from the foot of the

tree. What is the height of the part standing? Ans. 16ft.

2.

3. How far from the center of a circle is its chord 8 inches long if its radius is 5 inches. Ans.

3”

4. For what positive value of x will the following lengths be sides of a right triangle 2x + 1, 5x

– 1, 8x – 3? (The last being the longest) Ans. X = 1

I. Circular Functions

1.SinA=y/r 4. CotA=x/y

2.CosA=x/r 5.SecA=r/x

3.TanA=y/x 6.CscA=r/y

II. The Relation Among Functions

1. TanA=SinA/CosA

2. CotA=CosA/SinA=1/TanA

3. SecA=1/CosA

4. CscA=1/SinA

5. Versed SinA=1-CosA

6. Coversed SinA=1-SinA

P(x,y)

Page 58: Mathematics

7. ExsecantA=SecA-1

III. Pythagorean Identities

1. Cos2A+Sin2A=1

2. 1+Tan2A=Sec2A

3. Cot2A+1=Csc2A

IV. Sum and Difference of Angles Identities

1. Sin(A+B)=SinACosB+CosASinB

2. Sin(A-B)=SinACosB-CosASinB

3. Cos(A+B)=CosACosB-SinASinB

4. Cos(A-B)=CosACosB+SinASinB

5.

6.

V. Double Angle Identities

1. Sin2A = 2SinACosA

2. Cos2A = Cos2A-Sin2A

= 2cos2A-1

= 1-2Sin2A

3.

VI. Complementary Angle Identities

SinA = Cos(90-A)

CosA = Sin (90-A)

TanA = Cot(90-A)

CotA = Tan(90-A)

SecA = Csc(90-A)

CscA = Sec(90-A)

VII. Half Angle Identities

1.

2.

3.

VIII. Product of Sine and Cosine Indentities

1. 2SinACosB=Sin(A+B)+Sin(A-B)

2. 2CosASinB=Sin(A+B)-Sin(A-B)

3. 2CosACosB=Cos(A+B)+Cos(A-B)

4. 2SinASinB=Cos(A-B)-Cos(A+B)

IX. Sum and Difference of Sine and Cosine Identities

1. SinA+SinB=2Sin ½ (A+B)Cos ½ (A-B)

Page 59: Mathematics

2. SinA-SinB=2Cos ½ (A+B)Sin ½ (A-B)

3. CosA+CosB=2Cos ½ (A+B)Cos ½ (A-B)

4. CosA-CosB=2Sin ½ (A+B)Sin ½ (A-B)

X. Sine Law

1.

2. A+B+C = 180

XI. Cosine Law

1. a2 = b2+c2-2bcCosA

2. b2 = a2+c2-acCosB

3. c2 = a2+b2-2abCosC

4. A+B+C = 180

XII. Tangent Law

1.

2.

3.

4. A+B+C = 180

5.

Properties of Graph of Circular Functions

Consider the function

y = a sin (bx + c) + d

c/b

2/b

1) amplitude a – highest value of the function in standard form, a property is true

for sine, cosine functions only

2) Period 2/b – for sine, cosine, secant and cosecant functions.

/b – for tangent and cotangent

– distance of one complete wave of the function

3) Phase Shift c/b – property true for any functions

– distance the graph is shifted to the right or to the left from its

standard position

– if c < 0, the graph is shifted to the right

– if c > 0, the graph is shifted to the left

a

d

Page 60: Mathematics

4) Vertical translation d – true for all functions

– defined as the distance the graph is shifted upward or

downward

– if d > 0, the graph is shifted upward

– if d < 0, the graph is shifted downward

Sample Problem:

Determine the maximum value and the period of the function f(x) = –3 sin(/4 x + 5) - 7

Solution:

a = 3

d = –7

The graph has an amplitude of 3, but it is shifted downward by 7 units. Therefore the

maximum value of the function is 3 – 7 = –4.

Period = 2/(/4) ; since b = /4

Period = 8 Answer

Supplementary Problems

1. If tan y = 1/3 and tan m = ½, y and m being acute, find y + m.

2. If tan A = ¾ and Sin B = 12/13, A being greater than 180 and B is obtuse , find

a) Sin (A + B)

b) Cos (A + B)

c) Tan (A + B)

3. Find the value of Cos (A + B) if tan A = ¾ and csc B = 13/15, both angles are acute.

4. If tan x + tan y = 3 and csc y = 2, x and y are acute, find x.

1 – tan x tan y\

5. Tan x = 3, x is acute. Find

a) sin 2x

b) tan (x + 45)

6. If tan (x + y) = 2 and tan x = 1, find tan y.

7. Given the acute angles A and B, sin A = 3/5 and sin B = 12/13, find the value of

sin (A + B) + cos (A + B).

8. Find the value of tan (A + 2B) if cot A = tan B = 2.

9. Find sin 2A if tan A = -5/12 and cos A is negative.

10. Evaluate sin x(cot x/2 + tan x/2).

11. Given A, an angle between 0 and 360 such that sin A = -4/5 and whose tangent is

positive, construct A and find the value of sin 2A – cot ½ A.

12. Find the value of cos 2x – sin (90 + x) if tan x = -3/4 and sin x is positive.

Find all values of x less than 360 satisfying the equations below.

13. 5sin 2x – 25cos x = 10 – 4sin x

14. cot 2x – tan x = 1

15. sin 2x + sin x = 0

16. (ECE Board Exam Nov. 4, 1995)

1 – cos x sin x

–––––––– + –––––––– Ans: 2cscx

sin x 1 – cos x

17. ECE Board Exam April 3, 1993

Solved for in the equation: sin 2 = cos Ans: 300 and 1500

18. ECE Board Exam April 3, 199

Given the relations P = Asin + Bcos and

Q = Acos - Bsin , derive another equation showing the relationship

between P, Q, A and B not involving any of the trigonometric functions of angle

Ans: P2 + Q2 = A2 + B2

19. (ECE Board Exam April 6, 1991)

Simplify cos A + cosB + sin A + sin B Ans: 0

Sin A - sin B cos A - cos B

20. (ECE Board Exam April 6, 1991)

Simplify 2sin B cos B – cos B Ans: 2 sin B – 1

1 – sin B + sin2 B –cos2 B 1 – sin B

Page 61: Mathematics

21. (ECE Board Exam April 6, 1991)

Simplify sin2 150 + sin2 750 Ans:1

22. (ECE Board Exam April 6, 1991)

Simplify sin 0 0 + sin 1 0 + sin 2 0 + . . . + sin 90 0 Ans: 1

Cos 00 + cos 10 + cos 20 + . . . + cos 900

23. (ECE Board Exam April 16, 1991)

Simplify: sin20o + sin21o + sin22o + . . . sin290o Ans: 45.5

Angles of Elevation and Depression

Angle of elevation – is the angle made with the horizontal by the line of sight from an

observer to an object on the higher level than the observer.

Object

Line of sight

Angle of elevation

Observer Horizontal Line

Angle of depression – the angle made with the horizontal by the line of sight from an

observer to an object of lower level that the observer.

Horizontal Line

Observer

Angle of depression

Line of sight

Object

Supplementary Problems:

1. Two buildings 450 ft. and 600 ft. in height are opposite each other. From the roof of the

higher building, the angle of depression of the edge of the roof of the lower building is

38º40’. How wide is the street? Ans. 187.45ft

2. A tower and a monument stand on a level ground. The angles of depression of the top of

the monument viewed from the top of the tower are 13º and 31º, respectively; the height

of the tower is 145 m. Find the height of the monument. Ans. 89.3

3. A flagpole 25m high stands on the top of a tower, which is 10.5 m high. At what distance

from the base of the tower will the flagpole subtend an angle of 3º20’.

Ans. 34.59 m

4. The angle of depression of a barge from the top of a lighthouse is 16º. After the barge

has traveled 100 m toward the lighthouse, the angle of depression of the barge is 20º.

Find the height of the lighthouse. Ans. 135.147

5. From a boat directly south of a lighthouse, the angle of elevation of the top of the

lighthouse is 32º. From a second boat lying directly east of the first and 150 ft. from it, the

angle of elevation of the top of the lighthouse is 24º. Find the height of the lighthouse.

Ans. 95.181ft

6. If in right triangle eight times the product of the legs equals the square of the hypotenuse,

find the angles of the triangle.

Ans. [ArcTan (4 + 215)], [ArcTan ( 2/4 + 215)], 90º

Page 62: Mathematics

7. Two ladders, one of which is twice as long as the other, rest on the floor and reach the

same vertical height on the wall. The shorter ladder makes an angle of 60º with the floor.

What angle does the longer ladder make with the floor? Ans. 25.66º

8. From a point midway between two objects, one being three times as tall as the other, the

angle of elevation of the taller is twice the angle elevation of the shorter. Find this angle

of elevation. Ans. . 30 , 60

9. A man standing at a certain point on a level filed determines the angle of elevation of the

top of a tower 50 feet high, he then finds that by going 90 feet nearer the tower, the angle

of elevation is increased by 45º. At the first observation how far was he from the tower,

and what was the angle of elevation. Ans. 108.443ft., 27.753

10. A garage is 12 feet high and fixed on its top is a flagpole 15 feet high. On the opposite

side of the street from the garage at a given point, the garage and the flagpole subtend

equal angles. How wide is the street? Ans. 36ft.

11. The angles of a triangle are in the ratio 3:4:5. Express the ratio of the sine of the smallest

angle. Ans. 2/2

12. Two parallel chords of a circle of radius 8 inches are on the same side of the center and 5

inches apart. One subtends a central angle twice as large as the other. Find the length of

the shorter chord.

13. A man standing at a certain point on a level field determines the angles of elevation of the

top of a tower 50 feet high. He then finds that by going 2/3 of the distance to the base of

the tower, toward it, the angle of elevation has been doubled. At the first observation,

how far was he from the tower, and what was the angle of elevation?

14. A and C, the bases of two towers AB and CD standing on a horizontal plane, are 120 feet

apart. The angle of elevation of D as observed from A is double the angle of elevation of

B as observed from C. From a point midway between A and C the angles of elevation of

B and D are complementary. Find the height of the towers.

Ans. AB = 40 ft., CD = 90 ft.

15. (ECE Board Exam Nov. 4, 1995)

The angle which the line of sight to the object makes with the horizontal is above the

eye of the observer. Ans. Angle of Elevation

16. (ECE Board Exam Nov. 4, 1995)

The angle which the line of sight to object makes with the horizontal is below the eye of

an observer. Ans. Angle of Depressio

Solutions of Oblique Triangles

C C

b a b

a

A c B A c B

Two General Forms of Oblique Triangle

A triangle has three angles and three sides. If we are given the measures of three out of

this six principal parts, at least one of which is a side, we can possibly find the measures of

the other three parts.

Case I. Given two angles and one side.

Page 63: Mathematics

Use Law of Sines: B

a = b = c c a

sin A sin B sin C

A b C

Case II. Given two sides and an included angle.

Use Law of Cosines:

a2 = b2 + c2 – 2bc cos A

b2 = a2 + c2 – 2ac cos B

c2 = a2 + b2 – 2ab cos C

Case III. Given two sides and the angle opposite one of them.

This is known as the ambiguous case, you can use sine law and examine the possibility

of no solution, one solution or two solutions.

Case IV. Given three sides.

Use cosine law, to solve for each angle.

A = cos-1 b2 + c2 – a2

2bc

B = cos-1 c2 + a2 – b2

2ac

C = cos-1 a2 + b2 – c2

2ab

Sample Problem.

1. Two angles of a triangle are 5215 and 5930, and the shortest side is 38.46 ft. long.

Find the length of the largest side.

Solution :

Using Sine Law (given 2 angles and any side)

Note : The shortest side is opposite of the smallest angle 38.46 ft. is opposite of

5215.

The longest unknown side must be opposite of the largest angle.

Largest angle is the third angle :

180 – (5215 + 5930) = 68.25

Thus :

a = b__

Sin A Sin B

a = longest side, A = 68.25

b = shortest side = 38.46 ft. B = 5215

Now, a = b SinA

SinB

= (38.46)(Sin 68.25 o )

Sin5215’

= 45.178 ft. Answer

2. The sides of a triangle are 3, 5 and 7. Find the largest angle.

Solution :

Using Cosine Law

a2 = b2 + c2 – 2bcCosA

here, a = longest side = 7 ; b = 3; c = 5

A = largest angle unknown

Then, b 2 + c 2 – a 2 = Cos A

Page 64: Mathematics

2bc

Cos A = 3 2 + 5 2 – 7 2 = -0.5

2(3)(5)

A = Cos-1(-0.5)

A = 120 Answer.

Supplementary Problems

1. In an oblique triangle ABC, it is known that tan A = ¾, cos B = 5/13 and AB = 10. Find

a) sin C

b) side AC

c) side BC

2. A chord, AC, of a certain circle equals 13, and the angle B of the inscribed triangle ABC

is 49.35. Find the radius of the circle.

3. An observer in a balloon 1 mile high observes the angle of depression of an object on the

ground to be 3540. After ascending vertically at uniform rate for 10 minutes, he finds the

angle of depression of the same object is to be 5520. Find the rate of ascent of the

balloon in miles per hour.

4. From the top of a lighthouse 85 ft. high standing on a rock, the angle of depression of a

ship was 738 and from the bottom of the lighthouse the angle of depression was 3.

What was the height of the rock?

5. A tower 51.63 high makes an angle of 11312 with the inclined plane on which it stands.

The angle subtended by the tower at the same point down the plane from its base is

2327. How far is this point from the base?

6. A house is situated on a hillside which is inclined 2939 to the horizontal plane. A ladder

32.75 long just reaches the bottom of a window 25 from the ground. What is the

distance of the foot of the ladder from the house, measured down the slope?

7. A man standing at a point due to west of a tower 150 high found the angle of elevation of

the top of the tower to be 68. He then walked to a second point southwest of the first and

found the angle of elevation of the top of the tower to be 39. Determine how far he

walked.

8. A circle is inscribed in a ABC whose sides are a = 25, b = 33 and c = 38. How long

are tangents to the circle from vertex A? vertex B? vertex C?

9. A circle is inscribed in a triangle whose sides are 5, 12 and 13. Find the radius of the

circle.

10. The lower base of an isosceles trapezoid is 70.23. Each of the nonparallel sides is

35.18. each lower base angle is 8130. How long is each diagonal?

11. If the diameter of a circle is 32.68, find the angle at the center determined by a chord

12.9 long.

12. The area of a triangle is 50 sq. in. and the lengths of two of its sides are 20 in. and 10 in.

Find the included angle.

13. Two automobiles leave the same town at the same time. One goes north at the rate of 30

miles per hour, the other goes in the direction 47 E of N at the rate of 20 miles per hour.

How far apart are they at the end of 5 hours?

14. The acute angle between the diagonals of a parallelogram is 45. The diagonals are 8

and 12 inches. Find the lengths of the sides of the parallelogram.

Page 65: Mathematics

15. The sides of a triangle are 9, 12, and 14 and the largest angle is bisected, find the length

of the bisector of this angle and the length of the median to the shortest side.

Inverse Trigonometric Functions

1. Inverse Sine Function

y = Arcsin x if and only if x = sin y

y y

1 /2

–1

x x

-/2 /2 1

-1 -/2

y = sin x y = Arcsin x

2. Inverse Cosine Function

y = Arccos x if and only if x = cos y

y y

1

/2 x /2

–1 –1 1 x

y = sin x y = Arccos x

where, –1 x 1, 0 y principal value

3. Inverse Tangent Function

Y = Arctan x if and only if x = tan y

y y

1 /2

0

x 0 x

-1 1

-1

-/2

y = tan x y = Arctan x

where, x , -/2 < y < /2 principal value

Inverse Identities

1. Arcsin (sin ) = for -/2 /2

2. sin (Arcsin x) = x for –1 x 1

Page 66: Mathematics

3. Arccos (cos ) = for 0

4. cos (Arccos x) = x for –1 x 1

5. Arctan (tan ) = for -/2 x /2

6. tan (Arctan x) = x for all values of x

Supplementary problems

Solve the following trigonometric equations:

1. 3sinA = 2cos2A , 00 A 3600 ans: 300, 1500

2. sin2x + 2sin2 x = 1 , 0 < x < 1800 ans:300,900, 1500

3 .Tan -12x +Tan-13x = ans: 1 / 6

4. Tan-1[2sin(Cos-1x)] = 600 ans: 1 / 2

5. Arctanx + Arctan2x = Arctan3ans: -1, 1 / 2

6. Arcsin(1 – x) + Arccosx = 900 ans: 1 / 2

Solve the following problems

1. If sinA = 0.80, cosA > 0, find cotA. Ans. 0.75

2. If sin = and cos < 0, find tan . Ans. -12/5

3. If A – B = 450 and tanA = ¾ , find tanB. Ans. -1/7

4. If cot = -24 / 7, in Quadrant II, find sec . Ans. -25/24

5. If cosx = 1 / 3, find cos4x. Ans. 17/81

6. If cos = -1 / 3 and 00 < < 3600, find tan . Ans. Sqrt(2)

If sinA = 3 / 5 and A is in Quadrant II, find cos2A. ans. 7/25

If tanA = 3 / 4 , tanB = -15 / 8, A in Q III, B in Q IV, find

cos(A – B). Ans. 13/85

9. Two trains start at the same time from the same station and upon straight tracks

diverging at an angle of 68 degrees. If one train runs at 32 kph and the other at 46 kph,

how far apart are they at the end of 3 hours? Ans. 135.4km

10. The bearing of B from A is N200E; the bearing of C from B is S300E; and the bearing of A

from C is S400W. If AB = 10 m, find the area of the triangle formed by A, B and C. Ans.

13.94sq.m.

11. An airplane flying an altitude of one km directly away from a stationary observer on the

ground, has an angle of elevation of 48 degrees at a certain instant and an angle of

elevation of 20 degrees one minute later. Find the speed of the plane. Ans. 110.82kph

12. From a point on a level ground, the angle of elevation of the top of a building is observed

to be twice the angle of elevation of a window one third of the way up the building. Find

the angle of elevation of the top of the building. Ans. 60degrees

13. From the top of the tower 33 m high, the angles of depression of the top and bottom

of another tower standing on the same horizontal plane are found to be 28.930 and 53.680

respectively. Find the distance between the tops of the towers. Ans. 27.7m

14. A surveyor at a certain distance measured the angle of elevation of a cliff. He then

walked 20 m on a level ground toward the cliff. The angle of elevation from this second

station was then the complement of the former angle. The surveyor again walked 5 m

nearer to the cliff in the same line and found the angle of elevation from the third station

to be double the first angle. How high is the cliff? Ans. 22.91m

Page 67: Mathematics

15. The minute hand of a clock is 23 cm long while the hour hand is 15 cm long. The plane of

rotation of the minute hand is 5 cm above the plane of rotation of the hour hand. Find the

distance between the tips of the hands of the clock at 2:30 p.m. Ans. 30.9cm

16. A point P within an isosceles right triangle is at a distance of 6, 5 and 4 cm from the

vertices A, B and C respectively. Find the length of the hypotenuse of the right triangle if

the right angle is at C. Ans. 10.65cm

17. The angle of elevation of the top of a pole at a point 30 m from the pole is three times the

angle of elevation of the top of the same pole at a point 150 m from the pole. Find the

height of the pole. Ans. 56.7cm

18. A corner lot of land is 35 m on one street and 25 m on the other street, the angle between

two lines of the streets being 80 degrees. The other two lines of the lot are respectively

perpendicular to the lines of the streets. What is the worth of the lot at P1,000 per square

meter?

ans: P725,475.00

19. At noon, a ship A is sailing on a course eastward at the rate of 20 kph. At the same

instant, another ship B, 100 km east of ship A is sailing on a course N300W at the rate of

10 kph. How far away from each other are the ships after one hour? Ans 75.5km

20. A pole casts a shadow 15 cm long when the angle of elevation of the sun is 61 degrees.

If the pole leans 15 degrees from the vertical toward the sun, what is the length of the

pole? Ans. 54.23cm

21. Find the interior angles of a triangle whose sides are 21, 28 and 17.

ans: 48.400, 94.340, 37.260

22. The sides of a triangle ABC are a = 50 cm, b = 64 cm and c = 20 cm. Find the length of

the median drawn from B to AC. Ans. 20.64cm

23. Two stations B and C are situated on a horizontal plane 366 m apart. A balloon is directly

above a point A in the same horizontal plane as B and C. At B, the angle of elevation of

the balloon is 62 degrees and the angle at B subtended by AC is 53 degrees and at C,

the angle subtended by AB is 72 degrees. Find the height of the balloon. Ans. 799.19m

PROBABILITY

Introduction to Set Theory

Definitions

Set – a collection of objects called elements of any sort with restriction to

those objects clearly described. (Denoted by capital letters)

Element – any object that belong to a set (Denoted by small letters)

- symbol used to indicate that an element belong to a given set.

Subset – If all elements of set A are in set B or if there are elements of set B not

belonging to set A, then A is a subset of B denoted by A B.

Equal Sets – Two sets are equal if they have exactly the same elements.

A = B

Equivalent Set – Two sets are equivalent if they have equal number of

elements. A B

Size of Set

Null Set or Empty Set – a set without an element

denoted by { } or

Note: Null set is a subset of all sets

Unit Set – a set with only one element

Infinite Set – if it is impossible to list down all elements of a set

Finite Set – if it is possible to write all of its elements

Cardinal Number – refers to the number of elements of a certain set

– denoted by n(A) no. of elements of set A

Operations on Sets

Union of Sets

Page 68: Mathematics

The union of sets A and B is another set denoted by A∪B whose elements

are in A, or in B or both A and B.

In symbol, A B = {x ⋃ x A or x B}

Intersection of Sets

The intersection of sets A and B is another set denoted by A B whose⋂

elements are common to both A and B.

In symbol, A B = { x ⋂ x A and x B }

Relative Complement

The relative complement of the set B with respect to A is another set denoted

by A – B whose elements are all A but not in B.

In symbol, A – B = { x x A and x B }

Absolute Complement

The absolute complement of set A denoted by A is another set whose

elements do not belong to A but in the universal set U

In symbol A = { x x u and x A }

Product Set or Cartesian Product

The product set denoted by A x B is a set of ordered pairs (x, y) such that x is

an element of set A and y is an element of set B.

In symbol, A x B = { (x, y) x A and y B}

Note: A x B B x A

Disjoint Set

Two sets A and b are considered disjoint sets if they have no element in

common.

In symbol, A B = { } or A B = ⋂ ⋂

Venn Diagram – Pictorial representation of set relations and operations

– Introduced by John Venn, an English Logician

Number of elements in a union of Sets: COUNTING FORMULA

a.) n(A B) = n(A) + n(B) – n(A B)⋃ ⋂

b.) n(A B C) = n(A) + n(B) + n(C) – n(A B) – n(A C) – n(B C) + n(A B C)⋃ ⋃ ⋂ ⋂ ⋂ ⋂ ⋂

Sample Problems:

1. In a survey, 458 men like basketball, 385 like softball, 250 like both. How many men were

there? How many like softball only.

Solution:

In using Venn Diagram, always prioritize to indicate the number of elements

in the intersections.

u Let n(B) = numbers who like basketball

n(S) = number of men who like softball

n(B S)=number of men who like⋂

basketball and softball

n(B S) = total number of men in the survey⋃

Now,

n(B) – n(B S) = 458 – 250 = 208 like basketball only⋂

n(S) - n(B S) = 385 – 250 = ⋂ 135 like softball only Answer

n(B S) = n(B) + n(S) - n(B⋃ S)

= 458 + 385 –250

n(B S) = ⋃ 593 total Answer

Fundamental Counting Principle

If an element E, can happen in n1 ways, and for each of these another event E2 can

happen in n2 ways, and for each of these events, another event E3 can happen in n3 ways,

and so on and so forth, then all in all the events can happen simultaneously in N ways equal

to:

N = n1 n2 n3 …nk no. of ways

PERMUTATION / COMBINATION

Factorial Notation

- the factorial n, where n is any positive integer is denoted by n!

B S

208 135250

Page 69: Mathematics

Factorial n is defined as the product of positive consecutive integers from 1 to n

inclusive

That is,

n! = 1(2)(3) …(n-3)(n-2)(n-1)n

or n! = n(n-1)(n-2) …3.2.1

Observe that n! = n(n-1)!

= n(n-1)(n-2)!

By definition 0! = 1

PERMUTATION

- is arrangement of all or part of a set of objects in a definite order

- is any linear ordering of the elements of a set

CASES OF PERMUTATION

CASE 1: Permutation of n different things taken r at a time

- defined as an arrangement of r out the n objects with attention given to the order of

arrangement

nPr = n! r<n

(n-r)!

CASE 2: Permutation of n different things taken all at a time

nPr case whre r = n

nPr = nPn = n! = n! = n!

(n-n)! 0!

CASE 3: Permutation of n things not all different

Let N = number or permutations on n things taken all at a time of which p are alike, q

others are alike, r others are alike, and so on…

N = n! where p + q + r + …= n

p!q!r!…

CASE 4: Number of Permutation of n distinct objects arranged in a circle

N = (n-1)!

COMBINATION

- selection of group of objects out of a set irrespective of their order

- combination of n things taken r at a time refers to filling out r place when we have n things at

our disposal denoted by nCr

nCr = n!__

(n-r)! r!

Special Cases:

1. When r = n

nCn = n! = n! = 1

(n-n)! n! 0! n!

nCn = 1 (combination of n things taking all at a time)

2. When r = 1

nC1 = n! = n(n-1)! = n

(n-1)! 1! (n-1)!

nC1 = n (combination of n things taking one element at a time)

Other important Combination Formulas

1. nCr = nCn-r

that is nCa = nCb , if a + b = n

2. Number of combination of n things taking 1 or 2, … or n at a time

nC1 + nC2 + nC3 + … + nCn = 2n – 1

Supplementary Problems

1. In how many ways can 7 boys and 6 girls sit in a row if the boys and girls must alternate.

Ans. 3, 628, 800

2. In how many ways can 6 people be lined up to get on a car?

b.) If certain 3 persons insist on following each other

c.) If certain 2 persons refuse to follow each other how many ways are possible

Page 70: Mathematics

Ans. a.) 720 b.) 144 c.) 480

3. From the digits 0, 1, 2, 3, 4, 5, 6, 7, how many 3 digit number can be formed

b.) how many numbers are odd if digits are distinct

c.) how many numbers are even if digits are distinct

Ans. a.) 294 b.) 144 c.) 150

4. In a multiple choice test of 6 questions with 4 possible answers of which only one is

correct

a.) in how many different ways can a student check off one answer to each

question?

b.) In how many different ways can a student check off one number to each question

and get all answers wrong.

Ans. a.) 4, 096 b.) 729

5. How many triangles can be formed using the vertices of a dodecagon? Ans. 220

6. In how many ways can you invite one or more of your 5 friends in a certain show?Ans.

31

7. How many different three-digit numbers can be made with three 4’s, four 2’s and two 3’s?

Ans. 26

8. A department plays 12 softball games during a season. In how many ways can the team

end the season with 7 wins, 3 losses and 2 ties? Ans. 7920

9. How many distinct permutations are there of the letters of the word “CIRCUIT”.Ans. 1,260

10. How many numbers between 3,000 and 5,000 can be formed with the digits 7, 3, 4, 8, 2

and 5, no repetitions being allowed. Ans. 48

11. In how many ways can 8 boys and 5 girls be arranged in a circle if

a.) certain 2 boys will not be together

b.) a certain boy and girl won’t be together

c.) certain 2 girls won’t be together

d.) certain 3 girls will be together

Ans. a, b, c, = 399, 168, 000, d = 21, 772, 800

12. There are 12 ECE’s in a certain telephone company so that 3 will be assigned to

switching, 2 will be assigned in planning, 4 will be assigned to transmission and the

remaining will be assigned in some other functions. In how many ways the engineer may

be divided?

Ans. 277, 200 ways

13. A club has 25 members 4 of whom are engineers. In how many ways can a committee of

3 members be formed in order that at least one member is an engineer? Ans. 970

14. From a group of 6 women and 8 men, a committee of 5 is to be formed. In how many

ways can this be done. If each committee id to consist of a.) exactly 3 men b.) at least 4

men c.) at most 2 men. Ans. a.) 840 b.) 476 c.) 686

15. Solve for n in the equation nP4 = 30[nC5]. Ans. n = 8

PROBABILITY – measure of the likehood of occurrence of an event

Classical Definition

If an event E can happen in h ways (called the success) and fail to happen in f ways

(called failure) out of n possible equally likely ways, then the probability of occurrence is

denoted by

p(E) = h = favorable ways

h + f possible ways

The probability q that an event will fail to happen is given by

Page 71: Mathematics

q(E) = f probability of failure

h + f

Odd Ratios

1. Odds in favor

p = h

q f

2. Odds against

q = f

p h

Note: p + q = 1

Axioms of Probability

1. The probability of occurrence of a particular event is a positive real number from zero to

one 0 p 1

2. The probability of occurrence of any events in a sample space is equal to one

The probability of occurrence of certain event is equal to one P(S) = 1

3. The probability of impossible event is equal to zero P() = 0

4. The sum of probability of occurrence of each event in the sample space is equal to one.

Pi = 1

EVENTS

Independent Events

Two or more events are said to be independent if the occurrence or non-occurrence

of any of them does not affect the probabilities of the occurrence of any of the others.

Dependent Events

Two or more events are said to be dependent if the occurrence of one events affects

the probabilities of the occurrence of any of the other.

Mutually exclusive Events

Two or more events are said to be mutually exclusive if the occurrence of any one of

them excludes the occurrence of the other.

(that is, not more than one of them can happen in a single trial)

CONDITIONAL PROBABILITY

For two events E1 and E2, the probability that E2 occurs, given that E1 has occurred is

denoted by

P(E1/E2)

For compound events E1E2,

P(E1E2) = P(E1) P(E2/E1)

LAWS OF PROBABILITY

1. Multiplication Law (AND LAW)

Probability that events E1and E2 will happen both at the same time

P(E1E2) = P(E1) P(E2) for independent events

P(E1E2) = P(E1) P(E2/E1) for dependent events

So that, P(E2/E1) = P(E1 E 2)

P(E1)

2. Addition Law (OR LAW)

P(E1E2) = P(E1) + P(E2) for mutually exclusive events

P(E1E2) = P(E1) + P(E2) – P(E1E2) for not mutually exclusive

Note: P(E1E2) means probability that either E1 occur only, E2 occur only or E1 and

E2 occur both at the same time

For n mutually exclusive events

P(E1E2E3 … En) = P(E1) + P(E2) + P(E3) + … + P(En)

3. P(E’) = 1- P(A)

Probability of the complement of an event, E’

Probability in Repeated Trials – Bernoulli’s Experiment

P = nCr pr qn-r

Page 72: Mathematics

Where: P= the probability that the event will occur exactly r times in trials

p = the probability that the event will occur in a single trial

q = the probability that the event will fail to happen (probability of failure)

= 1 – p

nCr = the number of ways that the event will occur

Multinomial Distribution

If events E1, E2, E3, …, En can occur with probabilities P1, P2, P3, …, Pn respectively, then

the probability that E1, E2, E3, …, En will occur x1, x2, x3, …,xn times respectively is given by:

P = N! P1X1 P2

X2 P3X3 … Pn

Xn

x1! x2! x3! …xn!

where x1+ x2+ x3+ … +xn = N

Mathematical Expectation

If p is the probability of success in a single trial of an event, the expected number of

successes in n trials is given by :

E = nP

Supplementary Problems

1. In a family of 5 children, what is the probability that the first 3 are girls. Ans. 1/8

2. Four Electronics books and three Communications book are placed on a shelf. What is

the probability that the Communications book will be together? Ans. 0.14286

3. Determine the probability that 7 or 11 comes up in a single toss of a pair of fair dice.

Ans. 2/9

4. Tirso and Louie work independently in troubleshooting a circuit. The probability that Tirso

can fix it is 0.6 while the probability that Louie can fix it also is 0.5. What is the probability

that the circuit will be fixed? Ans. 0.8

5. During a certain war, an offensive troop released 4 bombs each of which has probability

of hitting the enemy of 0.25. If 4 bombs are released simultaneously, what is the

probability of hitting the enemy? Ans. 0.685

6. A box contains an assortment of red, blue and yellow balls. If three balls are drawn from it

at random, the probability that 2 red balls and 1 blue ball are drawn is 30% of the

probability that 2 blue balls and 1 yellow ball are drawn. Furthermore, the probability that

3 blue balls are drawn is 5/48 times the probability that one ball of each color is drawn.

However, if only two balls are drawn, the probability that one is blue and the other is a

yellow ball is 8 times the probability that both are red. How many are red balls, blue balls,

and yellow balls are there in the box.

*Author’s Hint: The number of balls are in arithmetic progression.

7. An urn contains 3 red pens and a number of blue pens. A second urn contains 5 blue

pens and an unknown number of red pens. One pen is drawn from the first urn and

placed unseen in the second urn. Now, two pens are to be drawn form the second urn. If

the odds in favor of getting 2 blue pens equals 3:5 and the odds in favor of getting pens

of different colors equal 31:29, how many are blue pens in the first urn and how many are

red pens in the second urn?

*Author’s Hint: No. of blue balls is 3 more than the no. of red balls

8. A pair of dice is thrown. If it is known that one die shows a 4, what is the probability that

a. the other dies shows a 5

b. the total of both dice is greater than 7

Ans. a.) 2/11 b.) 5/11

9. Find the probability of getting 1, 2 or 3 in four tosses of a fair die. Ans. 1/16

10. There are three shooters A,B,C. The probability that probabilities that A, B and C can hit

the target are 1/2, 1/3 and 1/4 respectively. If they are simultaneously shooting the target,

find the probability that two of them will hit the target. Ans. 0.25

Page 73: Mathematics

11. The probability that Alvin can win in a chess game whenever he plays is 20%. If he plays

5 games, find his probability of losing exactly 2 games. Ans. 32/625

12. An urn contains 5 white and 6 green balls. If 4 balls are drawn together, find the

probability that all are of the same color. Ans. 2/33

13. Out of 800 families with 5 children each, how many would you expect to have a.) 3 boys

b.) 5 girls and c.) either 2 or 3 boys? Ans. a.) 250 b.)25 c.) 500

14. A box contains a very large number of red, white, blue and yellow marbles in the ratio

4:3:2:1 respectively. Find the probability in 10 drawings.

a.) 4 red, 3 white, 2 blue and 1 yellow marble will be drawn and

b.) 8 red and 2 yellow marbles will be drawn Ans. a.) 0.0348 b.) 0.000295

16. a) How many ways can be five people be lined up to pay their electric bills?

b) If two particular persons refuse to follow each other, how many ways are possible?

Ans. a) 120 ways b) 72 ways

17. An engineering freshman must take a chemistry course, a humanities course, and a

mathematics course. If they may be select any of 2 chemistry courses, how many ways can

he arranged his program? Ans. 24 ways

18. In how many different ways can a ten- question true- false examination be answered?

Ans. 1024 ways

19. How many distinct permutations can be made from the letters of the word “mathematics”?

Ans. 4,989,600

20. How many ways can the first five players in a basketball team be filled with twelve men

who can play any of the positions?

Ans. 95,040 ways

21. How many three- digit numbers can be formed from the digits 0,1,2,3,4 and 5

a) if each digit is used only once in a given number?

b) if digits may be repeated in a given number

c) How many in (a) are odd numbers?

d) How many in (a) are even numbers?

e) How many in (b) are even numbers?

f) How many are less than 330?

g) How many are greater than 330?

Ans. a) 100 b)180 c)48 d)52e)90 f)90 g)89

22. A contractor wishes to build five houses, each different in design. In how many ways can

he place these homes on a street if two lots are on one side and three lots on the

opposite side?

Ans. 120 ways

22. In how many ways can four boys and three girls sit in a row if the boys and girls must

alternately be seated?

Ans. 288 ways

23. In how many ways can seven trees be planted in a circle? Ans. 720 ways

24. In how many ways can two mango trees, three chico trees, and two avocado trees be

arranged in a straight line if one does no distinguish between trees of the same kind?

Ans. 210 ways

25. A college team plays eight basketball games during an intramural. In how many ways can

the team end and games with four win, three losses and one tie? Ans. 280 ways

Page 74: Mathematics

26. Nine people will be shooting the rapids of Pagsanjan in three bancas that will hold 2, 4

and 5 passengers, respectively. How many ways is it possible to transport the nine

people to the falls?

Ans. 4,536 ways

27. From a group of three men and seven women, how many committees of five people are

possible?

a) with no restrictions?

b) with two men and three women?

c) with one man and four women if a certain woman must be on the committee?

Ans. a) 252 b) 105 c)60

28. From three red, four green, and five yellow bubblegums, how many selections consisting

of five bubblegums are possibleif two of each color are to be selected? Ans. 390

29. A shipment of 10 Sony Betamax video recorders contains 3 defective sets. In how many

ways can a hotel purchase 4 of these sets and receive at least 2 of the defective sets?

Ans. 70

30. A bag contains four blue, five red, and six yellow plastic chips.

a) If two chips are drawn, find the probability that both are yellow

b) If six chips are drawn, find the probability that there will be two chips of each color

c) If nine chips are drawn, find the probability that two will be red, five yellow and two blue

Ans. a) 1/7 b)180/1,001 c) 72/1,001

31. In a single throw of two dice, what is the probability of throwing not more than five?

Ans. 5/18

32. Find the probability that all five cards drawn from a deck are all hearts? Ans. 4.95 X 10 -4

33. a team of 5 students is to be chosen for a math contest. If there were ten male and eight

female students to choose from, what is the probability that three team members will be

male and two will be female? Ans. 20/51

34. A bag contains five pairs of socks. If four socks are chosen, what is the probability that

there is no complete pair taken? Ans. 8/21

35. In the game “spin-a-win” the rim of a wheel is divided in to 30 equal parts with each

marked P10, P20,...,P300. The “win” is indicated by a fixed pointer at the top. If the wheel

is spun, what is the probability that a three-digit number will be the player’s take home

winning? Ans. 7/10

36. If eight different books are arranged at random in a shelf, what is the probability that a

certain pair of books (a) will be beside each other? (b)will not be together? Ans. a) ¼

b) ¾

37. Joey prepares 3 cards for his 3 girlfriends. He addresses 3 corresponding envelopes. A

brown-out suddenly occurred and he hurriedly placed the cards in the envelope at

random. What is the probability that

(a) each card is send to its proper addressee?

(b) no card is sent to the proper address? Ans. a) 5/16 b)9/6

38. A box contains 15 red eggs and 20 white eggs. If 12 eggs are taken on random, what is

the probability that these will have an equal number of red and white eggs? Ans. 7/12

39. During a fund raising lottery, 250 tickets will sold to the freshman, of which 3 are winners.

Marissa, a freshman has 2 tickets. What is her probability of winning something? Ans.

248/ 10375

Page 75: Mathematics

40. If the probability that Nini will go to UP for a certain seminar is 1/3 and the probability that

she will go to UST that seminar is 1/4, find the probability that she will go to collage in

one of the two schools.

Ans. 7/12

41. If the probability that Ginebra, Alaska, and Shell will win the PBA open conference

championship are 1/5, 1/6, and 1/10, respectively, find the probability that one of theme

will win the title. Ans. 7/15

42. The probability that Joseph Estrada will be nominated to run fir president is ¼ , and the

probability of his election if nominated is 1/3. Find the probability (a) of his being elected

president, (b) of his being nominated and not being elected. Ans. a) 1/12 b)1/6

43. Find the probability of obtaining a 4 in each of two successive tosses of a pair of dice.

Ans. 1/1296

44. One box contains five black and three white handkerchiefs and another seven black and

five white handkerchiefs. If one handkerchief is drawn from each box, find the probability

that both will be (a) black, (b) white, (c) the same color. Ans. a) 35/96 b) 5/32

c)25/48

45. The probabilities that Marita will win the preliminary, semifinal, and final contest in singing,

are 3/8, 1/6, and ½, respectively. Failure in any contest prohibits participation in the

following one. Find the probability that she will (a) reach the final contest (b) win the final

contest. Ans. a) 1/16 b) 1/192

46. Three Physics books, five Algebra books, and two Chemistry books are on the shelf. Judd

decides to take two books and selects them at random. Find the probability that the first

book drawn will be Physics and the second Chemistry. Ans. 1/15

47. Find the probability of throwing in three tosses of a die, (a) exactly two 4’s (b) at least two

4’s.

Ans. a) 5/.72 b) 2/27

48. A bag contains three white, four red, and five green candies. Five withdrawals of one

candy each are made, and the candy is replaced after each. Find the probability that all

five will be red. Ans.1/243

49. If the probability that Alaska basketball team will win the PBA Conference Championship

is 2/3, find the probability that it will win exactly three championships in 5 years.

Ans. 80/243

50. Six Algebra books, four Physics books, and two Chemistry books are on the table. If a

book is removed and replaced, then another is removed and replaced, and so on until six

removals and replacements have been made, find the probability that an Algebra book

was removed and replaced (a) three times (b) at least three times. Ans. 16/243

51. If the probability that Imelda will be elected to office is 2/3, find the probability that she will

be elected for successive terms and then defeated on the fifth term. Ans. 16/243

52. the probability of an event happening exactly twice in four trials is 18 times the probability

of it happening exactly five times in six trials, find the probability that it will happen in one

trial. Ans. 1/3

53. If the probability that an event will happen exactly three times in five trials is equal to the

probability that it will happen exactly two times in six trials, find the probability that it will

happen in one trial. Ans. 0.451

54. How many permutations can be formed from the letters of the word “constitution”? Ans.

9,979,000

Page 76: Mathematics

55. How many four- place numbers can be written using the digits from 1 to 9? Ans. 3,024

56. Find in if P(n,3)= 6 C(n,5) Ans. n=8

57. Two dice are rolled. Find the probability that the sum of the two dice is greater than 10.

Ans. 1/12

58. A pair of dice is thrown. Find the probability of having a 7 or 11. Ans. 2/9

59. A pair of dice is thrown. If it is known that one die shows a 4, what is the probability that

the other die shows a 5? Ans. 2/11

60. Nine tickets, numbered to 1 to 9, are in the box. If two tickets are drawn at random,

determine the probability that both are odd. Ans. 5/18

61. A committee of three is to be chosen from a group of 5 men and 4 women. If the

selections is made at random, find the probability that two are men. Ans. 10/21

62. Three balls are drawn from box containing 5 red, 8 black, and 4 white balls. Determine

the probability that all are white. Ans. 1/170

63. A bag contains 9 balls numbered 1 to 9. Two balls are drawn at random. Find the

probability that one is even and the other is odd. Ans.5/9

64. How many cars can be given license plates having 5 digits numbers using the digits

1,2,3,4 & 5 with no digit repeated in any license plate? Ans. 120

65. A committee of 4 is selected by lot from a group of 6 men and 4 women. What will be the

probability that will consist of exactly 2 men and 2 women? Ans. 3/7

66. There are 52 tickets in lottery in which there is a first and a second prize. What is the

probability of a man drawing a prize if he owns 5 tickets? Ans. 0.18367

67. There are three candidates for A, B, and C for mayor of a certain town. If the odds are 7:5

that candidate A will win and those of B are 1:3, what is the probability that candidate C

will win? Ans. 1/6

68. A coin is biased so that the head is twice as likely to occur as tail. If the coin is tossed 3

times, what is the probability of getting

a) 2 tails and 1 head? Ans. 2/9 b) at least 2 heads? Ans. 20/27

69. Three man are running for public office. C candidates A & B are given about the same

chance of winning. But candidate C is given twice the chance of either A and B. Find the

probability that:

a) C wins b) A does not win

Ans. a) ¾ b)3/4

70. A player sinks 50% of all his shots. What is the probability that he will make exactly 3 of

his next 4 shots? Ans. 25%

Page 77: Mathematics